Cost & Management Accounting-munotes

Page 1

1
MODULE -I
Unit-1
MARGINAL COSTING AND ABSORPTION
COSTING
Unit Structure :
1.0 Learning objectives
1.1 Absorption Costing
1.2 Limitations of absorption costing
1.3 Marginal Costing
1.4 Marginal v/s Absorption costing
1.5 Contribution analysis
1.6 Solved Illustrations
1.7 Questions
1.0 LEARNING OBJECTIVES
After studying this chapter, the student should able to
understand -
Meaning of absorption costing
The distinction between marginal costing and absorption costing
The meaning of the terms -breakeven po int, margin of safety,
p/v ration, angle of incidence and cost indifference point.
How to prepare profit statements based on marginal costing
and absorption costing.
the assumptions underlying CVP analysis
How to calculate break -even points for multi -produ ct
situations.
1.1 ABSORPTION COSTING
Absorption costing is principles whereby fixed as well as
variable costs are allocated to cost units and total overheads are
absorbed according to activity level. It is the practice of charging all
costs irrespective of fixed and variable and direct and indirect
expenses are charged. It is a simple and fundamental method of
asserting the cost of a product or service. This method is familiar
since many companies still follow this approach for pricing
decisions. This th e oldest and widely used system. This method is
also called as ‘cost plus’ costing.munotes.in

Page 2

2
1.2 LIMITATIONS OF ABSORPTION COSTING
Absorption costing suffers from the following limitations.
In practice this method employs highly arbitrary method of
apportionment o fo v e r h e a d .T h i sr e d u c e st h ep r a c t i c a lu t i l i t y
of cost data for control purposes.
Under absorption costing, fixed cost relating to closing stock
is carried forward to the next year. Similarly, fixed cost
relating to opening stock is charged to current year instead
of previous year. Thus under this method, all the fixed cost
is not charged against the revenue of the year in which they
are incurred. It is unsound practice.
Under absorption costing collection and presentation of cost
data is not very useful fo rd e c i s i o nm a k i n g ,b e c a u s ep r o c e s s
of assigning product cost a reasonable share of fixed
overhead obscures cost volume profit relationship.
Under absorption costing, behavioral pattern of costs is not
highlighted and thus many situations, which can be util ized
under marginal costing, are likely to go unnoticed in
absorption costing.
The complaint is sometimes made that absorption costing
often deals only with production costs and ignores selling
and administration costs.
The decision maker needs to know the costs that will vary as
ar e s u l to fh i sd e c i s i o n ,a n dt h ec o s t st h a tw i l lr e m a i n
unchanged. Absorption costing does not provide a
convenient basis for making such calculations. Its main
purpose is to provide cost information for stock valuation and
the me asurement of reported profits
1.3 MARGINAL COSTING
The term’ Marginal Cost’ is defined as the amount at any
given volume of output by which aggregate costs are changed if the
volume of output is increased or decreased by one unit. It is a
variable cost o f one unit of a product or a service i.e. a cost which
would be avoided if that unit was not produced or provided.
1.3.1 Definition and Meaning:
Marginal costing is a principle whereby variable costs are
charged to cost units and fixed costs attributable to the relevant
period is written off in full against the contribution for that period.
Marginal Costing is the ascertainment of marginal cost and themunotes.in

Page 3

3
effect on profit of changes in volume or type of output by
differentiating between fixed costs and variab le cost.
CIMA defines marginal as “the accounting system in which variable
cost are charged to the cost units and fixed costs of the period are
written off in full against the aggregate contribution.
Marginal Costing is not a distinct method of costing li ke job
costing or process costing. It is a technique which provides
presentation of cost data in such a way that true cost volume profit
relationship is revealed. Under this technique, it is presumed that
costs can be divided in two categories, i.e., fixed cost and variable
cost. Fixed cost is charged to contribution of the period in which it
is incurred and is considered period cost.
1.3.2 Features of marginal costing:
a.Costs are divided into two categories, i.e. fixed costs and
variable costs.
b.Fixed cost is considered period cost and remains out of
consideration for determination of product cost and value of
inventories.
c.Prices are determined with reference to marginal cost and
contribution margin.
d.Profitability of department and products is determined wit h
reference to their contribution margin.
e.In presentation of cost data, display of contribution assumes
dominant role.
f.Closing stock is valued on marginal cost
1.3.3 Advantages of Marginal costing
a.It avoids the complications of over or under absorption of fixed
cost by excluding it from cost of production.
b.The technique provides useful data for managerial decision
making.
c.By not carrying forward fixed cost from period to period, it
facilitates cost comparison.
d.The impact of profit on sales fluctuations are clearly shown
under marginal costing.
e.The technique is flexible in the sense it can be used along with
other techniques such as budgetary control and standard
costing.
f.It establishes a clear relationship between cost, sales and
volume of put and break even analysis which shows the
effect of increasing an decreasing production activity on
theprofitability of the company.munotes.in

Page 4

4
g.It provides useful data for the management in determination of
policies regarding future
h.te production and sales.
i.Stock of work in pr ogress and finished goods are valued at
marginal cost, which is uniform.
1.3.4 Limitations:
a.The segregation of semi variable costs often poses a
problem
b.Closing stock of work in progress and finished goods are
understated which is not acceptable to tax aut horities.
c.With the change technology and owing to automation of
industries, it results in more fixed cost. Marginal costing fails to
reflect the exact change because of adoption of new
technology.
d.It does not provide any yardstick to exercise control. So a n
effective means of control cannot be exercised.
e.The technique is not suitable under cost plus contract
because of technique ignores fixed cost in calculating total
cost.
f.Variable cost per unit remains constant only in the short run
but not in the long ru n.
g.Cost comparison of two jobs will be difficult. Though
marginal costing may be same for both the jobs.
h.When sales are based on marginal cost or marginal cost
with some contribution, it may result in losses or low profit.
1.4 MARGINAL V/S ABSORPTION COST ING
Absorption Costing Marginal costing
1.All costs are charged to the
cost of production1. Only variable cost is charged
tocost of production. Fixed
costs are recovered from
contribution.
2. Stock of work in -progress and
finished goods are valued at full
ortotal cost. Fixed cost is
carried over from one period to
another period which distorts
cost comparison.2. Stock of work in progress and
finished goods are valued at
marginal cost. This facilitates
cost comparison.munotes.in

Page 5

5
3. The difference between sale s
and total cost constitute profit.3. The excess of sales revenue
over variable cost is known as
contribution when fixed cost
is deducted from contribution, it
results in profit.
4. The apportionment of fixed
costs on an arbitrary basis gives
rise to und er or over absorption
of overheads4. As only variable costs are
charged to products, it does not
give rise to over or under
absorption of overheads.
5. Costs are classified according
to functional basis such as
production cost, administration
cost, sell ing and distribution
cost.5. Costs are classified according
to variability.
1.5 CONTRIBUTION ANALYSIS
1.5.1 Contribution
Contribution is the excess of selling price over variable
costs. It is known as contribution because it contributes towards
recov ery of the fixed costs and profits. B y equation, the concept of
contribution can be stated as follows:
Contribution = Sales -Variable cost
Or
Contribution = F + P
1.5.2 Cost Volume profit (CVP) Analysis:
It is an important tool of profit planning. It p rovides
information about the following matters:
i.The behavior of cot in relation to volume
ii.Volume of production or sales, where the business will break
even
iii.Sensitivity of profits due to variation in output
iv.Amount of profit for a projected sales volume
v.Quantity of production and sales for a target profit level
Cost volume profit analysis may therefore be defined as a
managerial tool showing the relationship between various
ingredients of profit planning viz, cost selling price and volume of
activity, e tcmunotes.in

Page 6

6
Profit –Volume (P/V) Ratio a)Contribution=Salesb)Differences in Profits=Differences in sales
i)P/V RatioContribution= 100Sales
ii)P/V Ration = 100% --Marginal Cost%
Profit at a given sales volume
Contribution = Sales x P/V Ratio
Profit = Contribution -Fixed Cost
Profit at a given sales level = (Sales Revenue x P/v Ratio) -Fixed
cost.
1.5.3 Breakeven Point:
The point which breaks the total cost and the selling price
evenly to show the level of outp ut or sales at which there shall be
neither profit nor loss, is regarded as break even point. At this point,
the income of the business exactly equals its expenditure. If
production is enhanced beyond this level, profit shall accrue to the
business, and it is decreased from this level, loss shall be suffered
by the business.
Breakeven point (in units)Fixed Cost=Contributioon per unitBreak –even Point (in Rs.)Fixed Cost=× S a l e sContribution per unit
1.5.4Margin of safety:
Total sales minus the sales at breakeven point is known as the
margin of safety
Thus, the formula is:
Margin of Safety = Total Sales -Break even sales
Margin of Safety can also be computed according to the
following formula :
Margin of safetyNet profit=P/V Ratio
Margin of safety can also be exp ressed as a percentage of
sales :
Margin of Safety= ×100Total Salesmunotes.in

Page 7

7
Check Your Progress:
1)Define the following terms.
a)Absorption Costing
b)Marginal costing
c)Contribution
d)Break Even Point
e)Margin of Safety
2)Give Formulas
a)Contribution
b)Profit Volume Ratio
c)Profit at a given sales levels
d)Break Even point in units
e)Margin of safety.
1.6 SOLVED ILLUSTRATIONS
Illustration -1:
Prepare Income statements under Absorption Costing and
under Marginal costing from the following information relating to the
year2 0 0 1 -02:
Opening Stock =1,000 units valued at Rs. 70,000
including variable cost of Rs. 50 per unit.
Fixed cost =Rs. 1, 20,000
Variable cost =Rs. 60 per unit
Production =10,000 units
Sales =7,000 units @ Rs. 100 unit
Stock is valued on the basis o fF I F Omunotes.in

Page 8

8
Solution:
INCOME STATEMENT (Under Absorption Costing)
Rs. Rs.
Sales (7,000 units @ Rs. 100 per unit) 7,00,000
Less : Cost of Goods Manufactured :
Variable cost (10,000 unit @ Rs. 60 per unit) 6,00,000
(Rs. 1,20,000) 1,20,000
Fixed cost (10,000 units = Rs. 12 per unit) 7,20,000
70,000
Add :Value of Opening Stock 7,90,000
Less :Value of Closing Stock (4,000 units @ Rs. 72per unit)2,88,000
5,02,000
Profit 1,98,000
INCOME STATEMENT (Under Marginal Costi ng)
Rs. Rs.
Sales 7,00,000
Variable cost 6,00,000
Add: Value of Opening Stock (1,000 units @ Rs. 50
per unit)50,000
6,50,000
Less : Value of Closing Stock (4,000 units @ Rs. 60
per unit)2,40,000
4,10,000
Contribution 2,90,000
Less:Fixed Cost 1,20,000
Profit 1,70,000
Illustration -2
Your Company has a production capacity of 12,500 units
and normal capacity utilisation is 80%. Opening inventory of
finished goods on 1 -1-1999 was 1,000 units. During the year
ending 31 -12-1999, it produced 11,000 units while it sold only
10,000 units.munotes.in

Page 9

9
Standard variable cost per unit is Rs, 6.50 and standard
fixed factory cost per unit is Rs. 1.50. Total fixed selling and
administration overhead amounted to Rs. 10,000. The
company sells its produ ct at Rs. 10 per unit.
Prepare Income Statements under Absorption Costing and
Marginal Costing. Explain the reasons for difference in profit,
ifany.
Solution:
INCOME STATEMENT (Absorption Costing)
Rs. Rs.
Sales (10,000 units @ Rs. 10) 1,00,000Variable factory cost (11,000 units @ Rs.
6.50)71,500
Fixed factory cost (11,000 units @ Rs. 1.50) 16,500
88,000
8,000
Add : Opening stock (1,000 units @ Rs. 8) 96,000
16,000
Less : Closing stok (2,000 units @ Rs. 8) 80,000
Les
s:Over-absorption (1,000 units @ Rs. 1.50) 1,500
78,500
Add : Selling and administration overhead 10,000
Total cost 88,500
Profit 11,500
INCOME STATEMENT (Marginal Costing)
Rs. Rs.
Sales (10,000 units @ Rs. 10) 1,00,000
Variable cost (1 1,000 units @ Rs. 6.50) 71,500
Add : Opening Stock (1,000 units @ Rs. 6.50) 6,500
78,000
Less : Closing Stok (2,000 units @ Rs. 6.50) 13,000
Variable cost of Manufacture 65,000
Contribution 35,000
Less : Fixed cost –Factory (10,000 x Rs. 1 .50) 15,000
Selling and Administration 10,000
25,000
Profit 10,000munotes.in

Page 10

10
The difference in profits Rs. 1,500 (i.e. Rs. 11,500 –Rs.
10,000) as arrived at under absorption costing and marginal costing
is due to the element of fixed factory cost includ ed in the valuation
of opening stock and closing stock as shown below:
Opening Stock Closing Stock
Rs. Rs.
Absorption Costing 8,000 16,000
Marginal Costing 6,500 13,000
1,500 3,000
Net Difference = Rs. 3,000 –Rs. 1,500 Rs.1,500
Illustration 3 :
If the Budgeted output is 80,000 units, Fixed cost is Rs.
4,00,000, Selling price per unit is Rs. 20 and variable cost per unit
is Rs. 10, find out BEP sales, BEP in units, P/V ratio and indicate
the margin of safety.
Solution :
Rs. Per U nit
Selling Price 20
Less : Variable Cost 10
Contribution 10
P/V RatioContribution 10= = 0.5Sales 20
Break Even Sales (Rs.)Fixed Cost=P/V Ratio. 4,00,000=0.5Rs=Rs. 8,00,000
Fixed Cost=Contribution Per Unit. 4,00,000=.10RsRs=40,000 Units
Margin of Safety Sales =Budgeted Output –Break Even Sales
Margin of Safety (Units) =80,000 –40,000 = 40,000 Units
Margin of Safety Sales (Rs.) =Margin of Safety Units x Selling Price
Unit
=40,000 x 10
=Rs. 4,00,000
Illustr ation -4:
(a)From the following information calculate :
(a)Break –Even Point.
(b)P/V Rationmunotes.in

Page 11

11
(c)Profit
(d)Profit at 75% capacity,
(e)Profit at 100% capacity
(1)Budgeted Sales Rs. 2,00,000 (80% capacity)
(2)Direct Materials 30% of Sales.
(3)Direct labor 20% on sales.
(4)Variabl e Overheads (Factory) 10% on sales.
(5)Variable Overheads (Administration) 15% of sales.
(6)Fixed Cost Rs. 30,000
Solution :
Activity
Level
Particulars 75% Rs. 80%
Rs.100%
Rs.
Sales 1,87,5000 2,00,000 2,50,000
Less : Variable Cost :
Direct Mat erial (30%) 56,250 60,000 75,000
Direct Labour (20%) 37,500 40,000 50,000
Factory Overheads (10%) 18,750 20,000 25,000
Administration Overheads
(15%)28,125 30,000 37,500
Total Variable Expenses 1,40,625 1,50,000 1,87,500
Contribution 46,875 50,000 62,500
Less : Fixed Cost 30,000 30,000 30,000
(3) Profit 16,875 20,000 32,500
(1) BEP Fixed Cost 30,000 30,000 30,000
(in Rs.) P/V Ratio 0.25 0.25 0.25
1,20,000 1,20,000 1,20,000Contribution46,87550,00062,500(2) P/V Ratio Sales 1,87,500 2,00,000 2,50,000
=0 . 2 5 =0 . 2 5 =0 . 2 5
Illustration -5:
Company X and Company Y, both under the same
management, makes and sells the same type of product. This
budgeted Profit and Loss Accounts for January –June, 2005, are
as under :munotes.in

Page 12

12
Company
`X’Company
`Y’
Particulars Rs. Rs. Rs. Rs.
Sales 6,00,000 6,00,000
Less :Variable Cost4,80,000 4,00,000
Fixed Cost 60,000 5,40,000 1,40,000 5,40,000
Profit 60,000 60,000
You are required to :
(i)Calculate the Break -Even Point for each company.
(ii)Calculate t he sales volume at which each of the two
companies with profit of Rs. 20,000.
(iii)Calculate margin of Safety for both the companies.
Marginal Cost Sheet
Particulars X YSales6,00,0006,00,000Less : Variable Cost 4,80,000 4,00,000Contribution1,20,0002,00,000Less : Fixed Cost 60,000 1,40,000
Profit 60,000 60,000
munotes.in

Page 13

13
Illustration -6.
(a)X Ltd. has earned contribution of Rs. 2,00,000 and net profit of
Rs. 1,50,000 on sales of Rs. 8,00,000. What is its margin of
safety?
(b)If margin of safety is Rs. 2,40,000 (40% of sales) and P/V Ratio
is 30% of AB Ltd., calculate its
(i) Break even sales and (ii) Amount of profit on sales of Rs.
9,00,000.
(c)Ac o m p a n ys e l l si t sp r o d u c ta tR s .1 5p e ru n i t .I na period, if
it produces and sells 8,000 units, in incurs a loss of Rs. 5 per
unit. If the volume is raised to 20,000 units, it earns a profit of
Rs. 4 per unit. Calculate break -even point both in terms of
rupees as well as in units.
(d)A company earned a profit of Rs. 30,000 during the year 1994 -
95. If the marginal co st and selling price of a product are
Rs. 8 and Rs. 10 per unit respectively, find out the amount of
`Margin of Safety’.
(e)The profit volume (P/V) ration of B B & Co. dealing in
precision instruments is 50% and the margin of safety is 40%.
You are requi red to work out the break -even point and the
net profit if the sale volume is Rs. 50 lakhs.
(f)Comment on the economic soundness of the following firms :
Firm A Firm B
Current Sales Volume 3,00,000 3,00,000
Break Even Sales Volume 2,00,000 2,00,000
Margin of Safety 1,00,000 1,00,000
Fixed Cost 1,00,000 60,000munotes.in

Page 14

14
(g)A company has a P/V Ratio of 40 per cent. By what percentage
must sales be increased to offset :
(i)10 per cent reduction in selling price and
(ii)20 per cent reduction in selling price
Solution :
(c)Average cost at 8,000 units volume
=S e l l i n gp r i c ep e ru n i t~l o s sc o m p o n e n tp e ru n i t=R s .1 5~R s .5
=R s .2 0 Average cost at 20,000 units volume = Rs. 15
–Rs. 4 = Rs. 11 Total cost at 8,000 units volume = Rs.
8,000 x Rs. 20 = Rs. 1,60,000 Total cost at 20,000 units
volume = Rs. 11 x 20,000 = Rs. 2,20,000munotes.in

Page 15

15
Alternative Solution :
We know the following relationship :
munotes.in

Page 16

16
Rs. Rs.
Current Sales Volume 3,00,000 3,00,000
Contribution on Current Sales Volume 1,50,000 90,000
(i.e. Sal es x P/V Ratio) (3,00,000 x 50%) (3,00,000 x 30%)
Less : Fixed cost 1,00,000 60,000
Profit 50,000 30,000
Comment : Firm A is more sound as compared to Firm B because
it gives excess profit of Rs. 20,000 (i.e. Rs. 50,000 –Rs. 30,000).
It is beause of high er P/V ratio of 50%. Higer the P/V ratio,
better it is. Firm A will start earning profit @ 50% on sales after
B.E.P. whereas firm B will earn profit @ 30% on sales in excess of
break even sales.
(g) Suppose selling price per unit is Re. 1 and units sold a re 100.munotes.in

Page 17

17
Illustration -7:
A company has annual fixed costs of Rs. 14,00,000. In 2001
sales amounted to Rs, 60,00,000 as compared with Rs.
45,00,000 in 2000 and profit in 2001 was Rs. 4,20,000 higher than
in 2000 :
(i)At what level of sales does the compa ny break -even?
(ii)Determine profit or loss on a present sales volume of Rs.
80,00,000.
(iii)If there is reduction in selling price in 2002 by 10% and the
company desires to earn the same profit as in 2001, what would
be the required sales volume?
Solution :
munotes.in

Page 18

18
= Rs. 8,40,000
1.7 EXERCISE
1.7.1 Simple Questions
1.Define cost -volume -analysis.
2.State any four objectives of cost volume profit analysis.
3.State any four assumptions of cost volume profit analysis.
4.State any four limitations of cost volum e profit analysis.
5.What is Contribution ? How is it different from profit ?
6.Give Marginal Costing Equation.
7.Give three ways by which P/V Ratio can be improved.
8.What is Margin of Safety? How can it be improved?
9.Why are P/V Ratio and Marginal of Safety calcu lated?
10.Distinguish BE charts from P/V charts.
11. Write a note on Cash Break -even chart.
1.7.2 Objective questions
From the following choose the most appropriate answer:
i)Contribution margin is also known as
a)Variable cost
b)Gross Profit
c)Net Income
ii)Period cost means
a)Variable cost
b)Fixed cost
c)Prime cost
iii)The break even point is the point at which:
a)There is no profit no loss
b) Contribution margin is equal to total fixed cost:
c) Total revenue is equal to total cost:
d) All of the abovemunotes.in

Page 19

19
iv) Production cost under marginal costing include
(a)Prime cost only
(b)Prime cost and variable overhead
(c)Price cost and fixed overhead
(d)Price cost, variable overhead and fixed overhead
v) One of the primary difference between marginal costing and
absorption co sting is regarding the treatment of :
(a)Direct material (c) Fixed overhead
(b)Variable overhead (d) Overhead costs
vi) Period costs are :
(a)Variable costs (c) Prime cost
(b)Fixed cost (d) Overhead cost
vii) Absorption costing differs from marginal costing in the
(a)Fact that standard costs can be used with absorption costing
but not with marginal costing.
(b)Amount of fixed costs that will be incurred
(c)Kind of activities for which each can be use
(d)Amount of costs assigned to individual units of
products.
viii)To obtain the break -even point in rupee sales value, total
fixed costs are divided by :
(a)Variable cost per unit
(b)Contribution margin per unit
(c)Fixed cost per unit
(d)Profit / volume ration.
ix) The break -even point is the point at which
(a)There is no profit no loss
(b)Contributio n margin is equal to total fixed cost,
(c)Total revenue is equal to total cost
(d)All of the above.
x) Margin of safety is referred to as :
(a)Excess of actual sales over fixed expenses
(b)Excess of actual sales over variable expenses
(c)Excess of actual sales over break -even sales
(d)Excess of budgeted sales over fixed costs.
Answer : i (b), ii(a), iii (a), iv (b), v(c), vi (c), vii (d), viii(d), ix (d), x(c).munotes.in

Page 20

20
1.7.3 Essay type questions:
1)What do you mean by differential costs and incremental
revenue?
2)State the managerial d ecision which can be taken with the
help of Differential cost Analysis?
3)Explain the importance of the marginal cost technique in
managerial decision making.
4)In the context of cost volume profit analysis, what is meant
any limiting factor? Discuss its utili ty.
5)Explain briefly the circumstances under which selling prices
below marginal cost may be justified.
6)How does cost volume profit analysis help control of cost?
7)“Cost Volume Profit analysis is helpful for profit planning”.
Explain.
8)What is `analysis of ma rgin of contribution’ ? Discuss the
need for it.
9)Define marginal costing, What are the features of marginal
costing?
10)“Cost -volume profit analysis is a very useful technique to
management for cost control, profit planning and decision
making”. Explain.
11) In the context of cost -volume profit analysis, what is meant by
limiting factor? Discuss its utility.
12) Define cost -volume profit analysis and explain its main
features and useful contribution to the management in
decision making.
13 ) How does cost -volume profit analysis help control of cost?
1.7.4 Practical Problems
Illustration 1:
You are supplied with the information relating to sales and
costs of sales of a manufacturing company. your are required to
find out :
1) a) P. V. Ratio.
b)Break even Point .
c)Margin of safety in 2002.
d)Profit when sales are Rs. 1, 20,000.
e)Sales required earning a profit of Rs. 75,000munotes.in

Page 21

21
2) Calculate the revised P. V. ratio, break even point in each of the
following cases :
a)Decrease of 10% in selling price.
b)Increase of 10% in var iable costs.
c)Increase of sales volume to 4000 units and increase in fixed
costs by Rs. 40,000.
d)Increase of Rs. 18,000 in fixed costs.
e)Increase of 20% in selling price and increase of Rs. 8,000 in
fixed costs.
3) The sales and cost of sales during the two years were as follows:
Year Sales Rs. Costs of Sales Rs. Units
2001 ...... ... ...
2002 ...... ... ...6,00,000
7,50,0005,60,000
6,80,0002,400
3,000
(M.Com ., Apr. 03, Adapted)
(Ans: P/V Ratio 20%, Fixed Cost -Rs. 80, 000)
Illustration 2 :
You are given the following information for the next year.
Year Units
Sales (10,000 units) ...... ... ... ... 1,20,000
Variable Cost ...... ... ... ... 48,000
Fixed Cost ...... ... ... ... 60,000
1)Find out the P. V. Ratio, Bre ak-even point and the margin of
safety.
2)Evaluate the effect of following on P. V. Ratio, Break -even
point and the margin of safety.
a)10% increase in Variable Cost.
b)10% decrease in Variable Cost.
c)10% increase in Fixed Cost.
d)10% decrease in Fixed Cost.
e)10% in crease in Physical Sales Volume.
f)10% decrease in Physical Sales Volume.
g)5% increase in Selling Price.munotes.in

Page 22

22
h)5% decrease in Selling Price.
i)10% increase in Selling Price and 10% decrease in Physical
Sales Volume.
j)5% decrease in Selling Price and 10% increase in
Physical Sales Volume.
(M.Com ., Oct 96, Adapted)
(Ans: P/V Ratio 60%, BEP -Rs.1,00,000, MOS -Rs. 20,000)
Illustration 3:
AB Ltd. and LM Ltd. are manufacturing the same product. The
Profit & Loss details are as under :
Parti cularsABLtd.
Rs.LM Ltd.
Rs.
Sales ...... ... ... ... 10,00,000 10,00,000
Less : Variable Cost ...... ... ... ... 4,00,000 6,00,000
...... ... ... ... 6,00,000 4,00,000
Less : Fixed Cost ...... ... ... ... 3,00,000 1,00,000
Profit ...... ... .. .. . . 3,00,000 3,00,000
You are required to:
1)Calculate Contribution / Sales ratio for each company.
2)Calculate BEP for each company.
3)Profits of each company if sales increase by 20%.
4)Profits of each company if sales decrease by 20%.
5) Comment on the prof itability of both companies.
(M.Com .A p r0 4 ,A d a p t e d )
(Ans: P/V Ratio AB Ltd. 60%, LM Ltd. 40% BEP -AB Ltd. Rs.
5,00,000, LM.Ltd -Rs. 2,50,000)
Illustration 4:
The Vijaya Electronics Co. furnishes you the following income
information of the year 1995.
Particulars First Half Second Half
Sales ...... ... ... ...
Profit ...... ... ... ...4,05,000
10,8005,13,000
32,400munotes.in

Page 23

23
From the above table you are required to compute the
following assuming that the fixed cost remains the sam ei nb o t ht h e
periods.
a)P/V Ratio
b)Fixed Cost
c)Break -even point
d)Variable Cost for first and second half of the year
e)The amount of Profit or Loss where sales are Rs.
3,24,000. f) The amount of sales required to earn a profit of Rs. 54,000.
(M.Com .A p r0 8 ,A d a p t e d )
(Ans: P/V Ratio 20%, Fixed Cost Rs. 1, 40,000, BEP -Rs. 7,
02,000)
Illustration 5:
National Plastic Ltd. manufacturing chairs provides the
following information:
Fixed cost Rs. 50,000 for the year
Variable cost Rs. 20 per chair
Capacity Rs. 2,000 chairs per year
Selling price Rs. 70 per chair
From the above mentioned information:
i)Find the Breakeven point
ii)Find the number of chairs to be sold to get a profit
ofRs. 30,000
iii)Find out Breakeven point and sales if the sellin g price changes
to Rs. 60 per chair.
iv)If the company can manufacture 600 chairs more per year with
an additional fixed cost of Rs. 2,000, what should be the
selling price to maintain profit per chair as at (ii) above?
(M.Com . Oct 04, Adapted)
(Ans: P/V Ratio 71.43% BEP -1,000 Chairs)
Illustration 6 :
Sunil Ltd. had prepared the following budget estimates for
the year 2004 :
Sales Units Rs. 15,000
Fixed Expenses Rs. 34,000
Sales Value Rs. 1, 50,000
Variable Costs Rs. 6 per unitmunotes.in

Page 24

24
You are required to:
i)Find out the P/V Ratio, Break Even Point and Margin of Safety.
ii)Calculate the revised P/V Ratio, Break Even Point and Margin
of Safety in each of the following cases :
a)Decrease of 10% in the selling price
b)Increase of 10% in the variable costs
c)Increase of sales volume by 2,000 units
d)Increase of ` 6,000 in fixed costs.
(M.Com . Oct 05, Adapted)
(Ans: P/V Ratio 40%, BEP -Rs.85, 000, MOS -Rs. 65,000)
munotes.in

Page 25

25
Unit-2
MANAGERIAL DECISIONS -I
Unit Structure :
2.0 Learning Objectives
2.1 Introduction
2.2 Long –term and Short -term decisions
2.3 Application s of Marginal Costing
2.4 Solved Problems
2.4 Illustration
2.5 Questions
2.0 LEERING OBJECTIVES
After Studying this chapter, you should be able to:
Explain distinction between a) relevant cost and irrelevant
cost’s) marginal cost and differential cost c) breakeven point
and cost indifference point; d) relevant cost and opportunity cost
and g) traceable cost and common cost.
Explain the importance of quali tative factors in decision
making
Construct the statements of relevant costs and relevant
revenues for such Problem as:
A) Deleting a segment; b) special selling price decisions;
c) make or buy decision and d)accepting or rejecting an
export order.
Expla in and consider the impact of opportunity cost, shadow
price or incremental opportunity cost and imputed cost on
decision making.
Distinguish between situation of decision making (i.e. choice
among the alternatives) and performance evaluation (i.e.
evaluat ion of managerial performance with reference to
overall contribution to companies’ profit.)
Explain the short term decision making and long term
decision making
Advise the management the best course of action after
proper evaluation of all the available in formationmunotes.in

Page 26

26
2.1 INTRODUCTION
Decision making involves choice between alternatives.
Many quantitative and qualitative factors have to be taken into
account in decision making. The term cost is very elusive; it has
different meanings in different situations .Ac o s ta c c o u n t a n t
examines each situation in depth to decide the kind of cost
concepts to be used and plays an imp9rtant role in decision
making by making precise and relevant data available to
management. In cost studies, a cost accountant should always
consider four points for decision makings; (i) he must establish
why a choice is necessary (ii) he must separately analyze
each available alternatives,(iii) specific effort should be made
to determine how every alternative alters or influences decision
makers choice, and (iv)choice of a particular course of action
from among the alternatives. Decision making involves
prediction, which cannot change the past, but it is expected to
influence the future.
2.2 LONG –TERM AND SHORT -TERM DECISIONS
Decision maki ng involves two types of decisions i.e. long
term decisions and short term operating decisions. The long term
decisions force management to look beyond the current year. Time
value of money and return on investment are major considerations
in long term dec isions. Short run operating decisions involves the
selection of alternatives that can be implemented within a one year
period. These short run operating decisions involve many special
non recurring decisions such as: i) make or buy: ii)sell or process
further; iii) accept or reject an order and countless other decisions.
2.3 APPLICATION S OF MARGINAL COSTING
The technique of marginal costing is largely use in the
managerial decision making process. The application of marginal
costing I the day to day deci sion making process are as follows.
2.3.1 Make or Buy decision:
Very often management is confronted with the problem of
deciding whether to buy a component or product from an outside
source or to manufacture the same if it is economical as compared
to the price quoted by a supplier. In deciding the absorption costing
would mislead. If the decision is to buy from an external source
the price quoted by the supplier should be less than marginal cost.
If the decision is to make within the organization, the cos to f
production should include all additional cost such as depreciation
on new plant interest on capita, etc., If this cost of production is lessmunotes.in

Page 27

27
than the quotation price, it should be decided making the product
rather than procure it from an external sour ce.
Factors that influence make or buy decision:
In a make or buy decision the following cost and non cost
factors must be considered specifically.
a) Cost factors
1.Availability of plant facility.
2.Quality and type of item which affects the production
sched ule
3.The space required for the production of item.
4.Any special machinery or equipment required.
5.Any transportation involved due to the location of production
6.Cost of acquiring special know how required for the item.
b) Replacing existing machinery with ne w machinery
Sometimes with a view to derive maximum efficiency an
existing plant may have to be replaced by a new one. Again the
guiding factors mentioned earlier will help in such decision making
process.
Items of differential costs
i)Capital equipment and associated costs, viz., interest,
depreciation., etc.,
ii)Loss on sale of old equipment
iii)Increased in fixed overhead costs.
Items of differential benefits
i)Saving in operating costs.
ii)Increased volume and value of production
iii)Realizable value of old machin e
iv)Tax benefits, if any.
c)Alternative use of plant or productive facility -
To take advantage of alternative use of production facility or
alternative use of plant it is necessary to know the contribution
margin. That alternative which yields highest contrib ution margin
shall be selected.
d)Product Mix, Profit planning and profit maximization -
Companies manufacturing varieties of products often have to
decide which product mi8xc is more profitable. That product mix
which gives maximum contribution is to be con sidered as best
product mixes. Similarly, profit planning is often considered so as
to earn reasonable profit if not maximum profit. The profit planning
is affected by factors such as i) volume of output, ii) Product mix,
iii)costs to be incurred, iv)Price st ob ec h a r g e da n ds oo n .M a r g i n a l
costing techniques guide the management in this regard.munotes.in

Page 28

28
e)Avoidable and unavoidable cost
Avoidable costs are those which can be eliminated if a
particular product or department, with which they are directly
related, is di scontinued. Unavoidable cost is that cost which will not
be eliminated with the discontinuation of a product or department.
f)Relevant Cost and Irrelevant Cost
A cost that is relevant to a decision is called relevant cost.
Past costs are not generally relev ant costs because they are sunk
costs or costs already incurred. Thus the book value of an asset or
depreciation charged in accounts in respect of an asset is not
relevant cost. On the other hand, the fall in the resale value of an
asset as a result of usi ng it, as also the running expenses incurred
to make use of the asset are relevant costs.
g)Profitability of the department o r products;
The preparation of a departmental profit and loss account
under marginal costing is useful in determining which dep artment is
making profit and which department is incurring a loss. This
enables the management to decide whether a particular
department, must continue operation or it should be eliminated.
The decision is taken by referring to the contribution made or los s
incurred by the department or product.
2.3.2 Selling at or below marginal cost:
Some time it may become necessary to sell the goods at a
price below the marginal cost some such situations are as follows:
a)Where materials are of perishable nature
b)Where l arge quantities of stock are accumulated and whose
market prices have fallen. This will save the carrying cost of
stocks.
c)In order to popularize a new product.
d)In order to increase sales of those products having higher
margin or profits.
If the selling pr ice is below the total cost but above the
marginal cost, the contribution will leave on under recovering of
fixed cost. If the selling price fixed is equal to marginal cost, there
will be a loss which is equal to fixed cost. However, where the
selling pric ei sf i x e di sl e s s e rt h a nt h em a r g i n a lc o s t ,t h el o s sw i l lb e
greater than fixed cost.
2.3.3 Determination of selling price and volume of output :
The determination of selling price and volume of output is
based on differential costing. The difference is total cost due to
difference in sales volume is known as differential costing. The
increase in sales volume is known as incremental revenue. The
analysis of differential cost and incremental revenue helps in
determining selling price which will yield the o ptimum profit. Somunotes.in

Page 29

29
long as incremental revenue is more than the differential cost it is
advantages to increase the output. But as soon as incremental
revenue equals the differential cost further increase in output is not
advantages. Different cost analysis thus helps to determine the
selling price and the level of activity which are expected to yield the
highest profit.
2.3.4 Shut Down or Continue:
A business is sometimes confronted with the problem of
suspending its business operations for a temporary peri od or
permanently closing down. Permanent closure of the business is a
very drastic decision and should be carried out only in extreme
circumstances.
Temporary shut down
The following items of costs and benefi ts should be
considered while deciding about t he temporary shutdown of plant.
Items of cost
i)Effect on fixed overhead costs.
ii)Packing and storing of plant and equipment costs.
iii)Setting up costs.
IV)Loss of goodwill/market.
v)Lay off or retrenchment compensation to workers
Check Your Progress:
1)Why the goods are sold at or below the Marginal Cost?
2)Distinguish between
a)Long Term Decisions and short Term Decisions.
b)Relevant Cost and Irrelevant cost
c)Avoidable and unavoidable cost
2.4 EXERCISE
Answer in Brief
1.Comment –Pricing decisions ma yb eb a s e do nP e r c e n t a g eo f
profit on total cost
2.Comment –Pricing decisions may be based on percentage of
profit on selling price.munotes.in

Page 30

30
3.Enumerate any two limitations of Marginal Costing
4.Comment –major limitation of Marginal Costing s that it is
difficult to sep arate fixed and variable costs.
5.Comment -Marginal Costing is not applicable to Contract
Costing.
6.Comment profit volume ratio ignores price changes.
7.Define -opportunity Cost.
8.Define -Replacement Cost
9.Define -Normal Cost
10.Define –Differential Cost
11.Define Avoidabl e cost
12.Define -Unavoidable
Cost
13.Define -differential Costing
14.14 Define Differential Cost Analysis.
15.15 Define Differential Costing
Select the correct answer in each of the following:
1.Measurable value of an alternative use of resources is
a)Sunk Cost c)Oppo rtunity cost
b) Imputed cost d) Differential cost
2.The decision maker should consider, in case of limiting factor to
maximize the Profit
a) Sales b) Contribution
c) Variable cost d) Fixed cost
3.In make or buy decision
a)Only marginal cost is relevant
b)Only fixed cost is relevant
c)Total cost is relevant
d)None of these
4.Ideal product mix is decided in terms of
a) Sales b) Variable cost
c) Total cost d) Marginal cost
5.Ac o s ti n c u r r e di nt h ep a s ta n dh e n c ei r r e l e v a n tf o rc u r r e n t
decisions making is
a) Fixed cost b) Direct cost
d) Discretionary cost d) Sunk cost
6.A cost that cannot be changed by any decision made now is
a) Sunk cost b) Opportunity cost
c) Indirect cost d) mixed cost
7. A shut down point is the point at which
a)Marginal cost an d purchase price should be considered
b)Contribution is less than fixed costmunotes.in

Page 31

31
c)Contribution is equal to fixed cost
d)None of these
8.In a decision situation which one is the cost not likely to
contain a variable cost component
a) Material b) Labour
c) Overhe ad d) Direct expenses
9.In a situation when the decision is to be taken about acceptance
or rejection of special orders where there is a sufficient idle
capacity which one is not relevant for decision making’
a)Absorption cost b)Variable cost
c)Differential cost d) Incremental cost
Theory Questions
1.What do you mean by limiting factor? How does the
management elect the most profitable mix the presence of a
limiting factor?
2.The effect of price reduction is always to reduce p/v ratio, to
raise the BEP nd to shorten the margin of safety. Explain and
illustrate your views with appropriate illustrations.
3.How does marginal cost differ from total cost? In what
circumstances, if any, may it be to the advantage of
manufacturer to sell some of its products at price :
a)below total cost
b)below marginal cost.
4.What is “cost and profit”? Bring out its importance.
5.“Profit -Volume analysis” is a technique of analyzing the costs
and profits at various levels of volume’. Explain how such
analysis helps management.
munotes.in

Page 32

32Unit-3
MANAGERIAL DECISIONS -II
Unit Structure :
3.0 Objectives
3.1 Solved Problems
3.2 Exercise
3.0 OBJECTIVES
After studying the unit the students will be able to solve the
problems on managerial decision making.
3.1SOLVED PROBLEMS
Illustration: 1
Ac o m p a n yw h i c hs e l l sf o u rp r o d u c t s ,s o m eo ft h e m
unprofitable proposes discontinuing the sales of one of them. The
following information is available regarding its income, cost and
activities for a year.
Products
A B C D
Rs. Rs. Rs. Rs.Sales3,00,0005,00,0002,50,0004,50,000Cost of sales at
purchase price2,00,000 4,50,000 2,10,000 2,25,000Area of storage (sq. ft.)50,00040,00080,00030,000No. of parcels sent 1,00,000 1,50,000 75,000 1,75,000
No. of invoices sent 80,000 1,40,000 60,000 1,20,000munotes.in

Page 33

33Its overhead cost and basis of allocation are :
Fixed costs Rs. Basis of
allocationRent and insurance30,000Sq. ft.Depreciation 10,000 Parcel
Salesman’s salaries and
expenses60,000 Sales volume
Administrative wages and
salaries50,000 No. of invoices
Variable costs
Packing, wages and
materials20 p. per parcel
Commissions 4 % of sales
Stationery 10 p. per invoice
You are required to:
a)Prepare a profit and loss statement showing percentage profit or
loss to sales for each product.
b)Compare the profit in the company discontinues sales of
product B with the profit if it discontinues product C. [I.C.W.A.,
Inter]
Solution:
(a) Profit and Loss Statement:
A B C D TotalProduct
Rs. Rs. Rs. Rs. Rs.
Sales (A) 3,00,000 5,00,000 2,50,000 4,50,000 15,00,000
Variable costs :
Cost of sales at2,00,000 4,50,000 2,10,000 2,25,000 10,85,000
Purchase Price
Commission @ 4% of
sales12,000 20,000 10,000 18,000 60,000
Packing, wages &
materials
@2 0p .p e rp a r c el 20,000 30,000 15,000 35,000 1,10,000
Stationery @ 10 p. per
invoice8,000 14,000 6,000 12,000 40,000
Total variable costs (B) 2,40,000 5,14,000 2,41,000 2,90,000 12,85,000
Contribution (A) –(B) 60,000 (-) 9,000 1,60,000 2,15,000
14,000munotes.in

Page 34

34Fixed Costs :
Rent and insurance
@1 5p .p e rs q .f t . 7,500 6,000 12,000 4,500 30,000
Depreciation @ 2 p. per
parcel2,000 3,000 1,500 3,500 10,000
Salesmen’s salaries &
expenses
@ 4 p. per Re. of sales 12,000 20,000 10,000 18,000 60,000
Administr ative wages &
Salaries @ 12.5 p. per
invoice10,000 17,500 7,500 15,000 50,000
Total Fixed Cost ( Y ) 31,500 46,500 31,000 41,000 1,50,000
Profit & Loss ( X ) –(Y) 28,500 (-) (-)1,19,000 65,000
60,500 22,000
Percentage of Profit or
loss on sales9.5 (-)1 2 . 1 (-)8 . 8 26.4 4.3
(b) If either product B or product C is discontinued then the
result will be as follows:
ContributionProduct B if
discontinued
Rs.Product C if
discontinued
Rs.
Product A 60,000 60,000
Product B -- --
Product C 9,000 (-)1 4 , 0 0 0
Product D 1,60,000 1,60,000
Total Contribution 2,29,000 2,06,000
Less : Fixed Costs 1,50,000 1,50,000
Total Profit Rs. 79,000 Rs. 56,000
Thus, if product B is discontinued the total profit may rise
toRs. 79,000 whereas if product C is discontinued the total profit
may fall to Rs. 56,000.
Illustration -2
Ac o m p a n yp r o d u c e sa n ds e l l sf o u rt y p e so fd o l l sf o r
children. It also produces and sales a set of dress kit for the dolls.
The company has worked out the following estimates for the next
year.munotes.in

Page 35

35Doll Estimated Standard Standard Estimated sale
demand Material Labour price per unit
cost Cost (Rs.) (Rs.)
(Rs.)
A 50,000 20 15 60
B 40,000 25 15 80
C 35,000 32 18 100
D 30,000 50 20 120
Dress 2,00,000 15 5 50
Kit
To enco urage the sale of dress kits, a discount of 20 % in its
price is offered it it were to be purchased along with the doll. It is
expected that all the customers buying dolls will also buy the
dress kit.
The company’s factory has effective capacity of 2, 00, 000
labour hours per annum on a single shift basis and it produces all
the products on that basis the labour hour rate is Rs. 15.
Overtime of labour has to be paid at double the normal rate.
Variable cost works out to 40% of direct labour cost
fixed costs are Rs. 30 lakhs per annum.
There will be no inventory at the end of the year.
You are draw a conservative estimate of the year’s
profitability.
[C.A., Inter]
Solution :
Statement of conservative estimate of the year’s profitability
Doll A Doll B Doll C Doll D Dress Kit
Estimated Demand (Units) 50,000 40,000 35,000 30,000 2,00,000
Rs. Rs. Rs. Rs. Rs.
Selling Price per unit (A) 60 80 100 120 50
Marginal cost per unit
Material Cost 20 25 32 50 15
Labour Cost 15 15 18 20 5
Variable Cost
(40% of labour cost) 6 6 7.20 8 2
Total marginal cost : (B) 41 46 57.20 78 22
Contribution per unit: (C) 19 34 57.20 78 22
= [(A) –(B)]munotes.in

Page 36

36Total contribution on 9,50,000 13,60,000 14,98,000 12,60,000 56,00,000
Estimated demand (50,000 x (40,000 x (35,000 x (30,000 x (2,00,000
Rs. 19) Rs. 34) Rs.42.80 Rs. 42) xR s .2 8 )
Less : Discount on dress
kits
Net contribution--
9,50,000--
13,60,000--
14,98,000--
12,60,00015,50,000
40,50,000
Total net contribution 91,81,000
(Rs.)
Less:Overti me Premium 5,79,990
(Rs.)(38,666 hrs x Rs. 15)
Less : Fixed Cost (Rs.) 30,00,000
Profit (Rs.) 55,38,000
Total labour hours required to meet estimated demand
of four types of dolls and their dress kit.
Doll
(a)Estimated
demand
(Unit s)Std. labour time
P.U.
(Std. labour cost ÷15)Total labourhours
(d) = (b) –
(b) (c) (c)
A 50,000 1 hr. 50,000.00
B 40,000 1 hr. 40,000.00
C 35,000 1.2 hrs 42,000.00
D 30,000 1.3333 hrs. 40,000.00
Dress kit 2,00,000 0.33333 hrs. 66,666.00
Total labour hours to meet estimated demand 2,38,666.00
Since the total available hours are only 20,000, therefore
38,666 hours will be utilized by employing the labour on overtime
basis.
** Total discount on the sale of dress kit.
Out of 2, 00,000 dress kit s, 1, 55,000 were sold along with
four types of dolls. Each unit of sale of dress kit along with a unit
ofdoll is entitled for a discount of 20 % of Rs. 50 i.e., Rs. 10. The total
discount amount on the sale of 1,55,000 dress kit comes to
Rs. 15,50,000.
Illustration :3
The following particulars are extracted from the records of
ELLORA SALES LTD.
Direct wages per hour is Rs. 5.munotes.in

Page 37

37Particulars Product A Product B
Sales (per unit) Rs. 100 Rs. 120
Consumption of material 2 kgs 3k g s
Material cost Rs. 10 Rs. 15
Direct wage cost Rs. 15 Rs. 10
Direct expenses Rs. 5 Rs. 6
Machine hours used 3h r s 2h r s .
Overhead expenses
Fixed Rs. 5 Rs. 10
Variable Rs. 15 Rs. 20
(a)Comment on the profitability of each product (both use the
same raw material) when (i) Total Sales pote ntial in units is
limited; (ii) Total sales potential in value is limited; (iii) Raw
material is in short supply; and iv) Production capacity (in
term of machine hours) is the limiting factor.
(b)Assuming raw material as the key factor, availability of which is
10,000 kg and maximum sales potential of each product being
3,500 units, find out the product mix which will yield the
maximum profit.
[I.C.W.A., Inter]
Solution :
(a) Marginal Cost Statement
Product
A B
Rs / Per
UnitRs / Per
UnitSelling Price100120Direct Materials 10 15
Direct Wages 15 10
Direct Expenses 5 6
Variable Overheads 15 20Marginal Cost4551Contribution margin 55 69P/V ratio55 %57.5 %Contribution per kg. of material 27.5 23
Contribution per machine hour 18.3 34.5
Comments:
(1)When total sales potential in units is a limiting factor, B is more
profitable as it is making a larger contribution margin per unit
as compared to A.
(2)When total sales potential in value is a limiting factor, still B is
more profitable as its P / Vr a t i oi sm o r et h a nt h a to fA .munotes.in

Page 38

38(3)When raw material is in short supply, A is more profitable as
its contribution per kg of material is more than that of product
B.
(4)When production capacity is limited, B is more profitable as it
makes larger contribution per machine hour than A.
(Note: Best position is reached when contribution per unit of key
factor is maximum)
(b) When raw material is a key factor, A is more profitable to
produce as its contribution per kg of material is higher than B.
If 3,500 units of A are manufactured, total material
consumption will be 7,000 kg (i.e. 35,00 x 2 kg). The balance
3,000 kg of material can be used to manufacture 1,000 units
(3000 kg ÷ 3) of B. The total contribution by this product mix
will be :
Contribution
Product A, 3,500 units @ Rs. 55 each Rs. 1, 92,500
Product B, 1,000 units @ Rs. 69 each Rs. 69,000
Total Rs. 2, 61,500
This sales -mix would give maximum contribution, therefore
maximum profit. Profit figure cannot be calculated as total fixed cost
is not given in the question.
Illustration: 4
A company produces a single product which is sold by it
presently in the domestic market at Rs. 75 per unit. The present
production and sales is 40,000 units per month representing 50 %
of the capacity available. The cost dat ao ft h ep r o d u c ta r ea su n d e r :
Variable costs per unit Rs. 50
Fixed costs per month Rs. 10 lakhs
To improve the profitability, the management has 3
proposals on hand as under :
(a) to accept an export supply order for 30,000 units per month at
a reduced price of Rs. 60 per units, incurring additional variable
costs of Rs. 5 per unit towards export, packing, duties etc.
(b)to increase the domestic market sales by selling to a
domestic chain stores 30,000 units at Rs. 55 per unit,
retaining the existing sales at the existing price.munotes.in

Page 39

39(c)to reduce the selling price for the increased domestic sales as
advised by the sales department as under :
Produce selling price per unit by Increase in sales expectedRs.(in units)5 10,000
8 30,000
11 35,000
Prepare a tabl et op r e s e n tt h er e s u l t so ft h ea b o v ep r o p o s a l s
and give your comments and advice on the proposals.
[I.C.W.A., Intermediate]
Solution :
Proposal (a)
Present +
export
LevelorderProposal (b)
Present +
export
LevelorderProposal (c)
Price reduction over
present levels
I or IIor III
Selling
price perunit (Rs.)7560
75 5570 67 64Less :VariablecostPer unit50555050505050(Rs.)Contributionper unit25 5 25 5 20 17 14(Rs.)Sales(inunits)40,000 30,000 40,000 30,000 50,000 70,000 75,000Contribution(Rs. In lakh) 10 1.5 10 1.5 10 11.9 10.5
Total
Contribution 11.5 11.5 10 11.9 10.5
(Rs. Inlakhs)Note: Fixed cost has been excluded as they would remain the
same under all alternatives.
Comment & Advise:
1)Proposal to sell 70,000 units (under proposal ‘C’ II) at a reduced
price of Rs. 67 is most profitable, as it would give maximum
contribution of Rs. 11 .9 lakhs.munotes.in

Page 40

402)Between proposal (a) and (b) there is no difference in
profitability. However, it is advisable to accept to accept export
order as there is no danger of competition in the domestic
market. It may happen that domestic chain stores may starts
selling at lower price.
3)There is a profitability of going wrong in estimating sale demand
atreduced prices; we may not be able to achieve sales of
70,000 units at reduced price of Rs. 67. Therefore, it is
advisable to go for export order.
Illustration -5
Sports Specialists Ltd. are famous for specialized
manufacture of quality ches boards sets. Presently, the company is
working below chess boards sets in the national market at Rs. 150
per unit. During April, 1994, 600 units were sold which is the regular
sales volume for each month all through the year. The unit cost of
production is :
Direct Ma terials Rs. 60
Direct Labour Rs. 30
Factory Overhead Rs. 30
Selling and administration Overhead Rs. 15
The company has received an export order on 20 -4-1994 for
supply of 600 units to be dispatched by 30 -6-1994. However, the
order stipulates the price per unit at Rs. 100 only. The cost analysis
indicated that the cost of direct material and direct labour that are to
be incurred on the export order would be same amount per unit as
the regular line of production. However an amount of Rs. 2,000 will
have to b e incurred on special packing, labeling get up etc. No
additional factory selling or administrative overhead costs would be
incurred in executing the export order since the firms is operating
below normal capacity.
Using differential cost analysis method, prepare the income
statement to show whether the acceptance of the export order
would be profitable to the company. Assumptions and comments if
any may be given separately.
[I.C.W.A., Intermediate]munotes.in

Page 41

41Solution :
Existing
Position
without
export
orderExpo rt
order
differential
figuresProposed
position
with export
orderUnits for 2 months (nos.)1,200Rs.Rs.Rs.Selling price per unit 150 100 --
Sales (A) 1,80,000 60,000 2,40,000
Direct Material@ Rs. 60 p.u. 72,000 36,000 1,08,000
Direct Labour @ Rs. 30 p.u. 36,000 18,000 54,000
Factory Overheads 36,000* -- 36,000
Special packing, labeling etc. -- 2,000 2,000
Selling & Admin. Overheads
@ Rs. 15 p.u. (for 2 months) 18,000 -- 18,000
Total Cost (B) 1,62,000 56,000 2,18,000
Profit ( A –B) 18,000 4,000 22,000
600 units p.m. x Rs. 30 x 2 months = Rs. 36,000
Assumptions & Comments :
1)As there is no change in factory, selling or administrative
overheads costs, these overheads has been taken as fixed.
Hence, they are irrelevant to the decision of accepting export
order.
2)The company can manufacture 600 units for export order in May
and June 1994 as it has a spare capacity of 800 units in two
months.
Conclusion:
The export order should be accepted as it would give
additional profit of Rs. 4000.
Illustration: 6
X Ltd. has two factories, one at Lucknow and another at
Pune producing 7,200 tons and 10,800 tonnes of a product against
the maximum production capacity of 9,000 and 11,880 tons
respectively at Lucknow and Pune.
10% of the raw material intr oduced is lost in the production
process. The maximum quantity of raw material available locally is
6,000 and 13,000 tons at Rs. 720 and Rs. 729 per tones at
Lucknow and Pune respectively. For the additional needs a supplier
of Bhopal is ready to supply ra w material at a factory site at Rs. 792
ton.munotes.in

Page 42

42Other variable costs of the production process are as 22.32
lakhs and Rs. 32.94 lakhs and fixed costs are Rs. 18 lakhs and Rs.
24.84 lakhs respectively for Lucknow and Pune factory.
The output is sold at a sel ling price of Rs. 1,450 and Rs.
1,460.
You are required to compute the cost per tone and net profit
earned in respect of each factory.
Can you suggest any other alternative production plan for
both the factories without any change in present total output of
1,80,000 tons whereby the company may earn optimum profit.
[C.A. Final]
Solution :
Statement of cost per tone and net profit earned in respect of
each factory
Lucknow Pune
Present Production (tones) 7,200
Rs. Rs.
Cost of raw material (Rs. In l akhs) 59.04 87.48
(Refer to working note 1)
Other variable costs (Rs. In lakhs) 22.32 32.94
Fixed costs (Rs. In lakhs) 18.00 24.84
Total cost (Rs. In lakhs) 99.36 145.26
Cost per ton(Rs.):total cost/production
in tones1,380 1,345
Selling price (Rs .) per tone 1,450 1,460
Net profit per ton (Rs.) : (S.P. –cost per
ton)70 115
Total net profit (Rs. In lakhs) Rs.(70 x 7,200 (Rs. 115 x
tons) 10,800 tons)
Total profit of the company (Rs. 5.04 lakhs + Rs. 12.42 lakhs)
= Rs. 17.46 lakhs
Computation of contribution per ton of outputmunotes.in

Page 43

43Lucknow Pune
Maximum production capacity (tones) 9,000 11,880
Present production (tones) 7,200 10,800
Cost per ton of output : Rs. Rs.
Cost per ton of output manufactured
from
Locally purchased raw material 800 810
(Refer to working note 2)
Cost per ton of output manufactured
from
Material purchased from Bhopal 880 880
(Refer to working note 3)
Other variable cost (Rs.) 310 305
(Rs.22.32 (Rs. 32.94
lakhs + lakhs + 10,800
7,200tons) tons)
Total variable cost
(when material is purchased locally)1,110 1,115
Total variable cost
(when material is purchased from1,190 1,185
Bhopal)
Selling Price per ton (Rs.) 1,450 1,460
Contribution per ton of output : (S –V) 340 345
(Locally purchased raw material) (1,450 – (1,460 –
1,110) 1,115)
Contribution per ton of output (S –V) 260 275
(when material was purchased from (1,450 – (1,460 –
Bhopal) 1,190) 1,185)
The priority to produce 18,000 of total output de pends on
contribution per ton from
Contribution
per unitPriorityPune factory (local purchase of raw material)345ILucknow factory (local purchase of raw
material)340 II
Pune factory (raw material purchased from 275 III
Bhopal)
Lucknow factory (raw material purchased from 260 IV
Bhopal)
Suggested alternative production plan to earn optimal profitmunotes.in

Page 44

44Output (in tones) Production
priorityRaw
material
input (in
tones)Lucknow Pune Total
I 11,700 tons 13,000 --11,700 11,700
II 5,400 ton s 6,000 5,400 -- 5,400
III (11800 –
11700) 200 180 180
180 tons
IV 720 tons
(balancing
figure)
(18,000 – 800 720 -- 720
17,280 tons)
20,000 6,120 11,880 18,000
Working Note :
Lucknow Pune1Present production output(formal)7,200 10,800
Total raw material required for
Present production (tons) 8,000 12,000
[7,200 x 100 ÷ 90] [10,800 x 100 ÷ 90]
Raw material procured locally(tones)6,00012,000Raw material procured from
Bhopal 2,000 --
Cost of raw material
purchased
Locally and from Bhopal 59.04 87.48
(Rs. In lakhs) (Rs. 720 x 6,000+
Rs. 792 x 2,000)(12,000 x Rs. 729)2Cost per ton of outputManufactured from locally 800 810
Purchased raw material (in
Rs.)[Rs.720 x 100÷90] [Rs.72 9x1 0 0 ÷ 9 0 ]
3)Cost per ton of outputManufactured from material880880Purchased from Bhopal (in
Rs.)[Rs. 792 x 100÷90]
Illustration -7munotes.in

Page 45

45ZED Ltd. manufacturing two products P and Q and sells
them at Rs. 215 and Rs. 320 per unit respectively. The variable
costs per unit are as under:
Particulars Product P
Rs.Product Q
Rs.
Raw Materials :
Material –X 22.00 28.00
Material –Y 8.00 32.00
Direct wages (Rs. 6 per labour hour)
Department –A 36.00 54.00
Department –B 18.00 36.00
Depart ment –c 54.00 --
Department –D -- 72.00
Variable overheads 23.00 14.30
The company procures raw materials against import license.
The company operates at single shift a day of 8 hours for 300 days
in a year. The number of workmen engaged is 30, 16, 1 8a n d2 4i n
department A, B, C and D respectively. Neither the workers are
subject to transfer from one department to another nor is any new
recruitment possible at present. Fixed costs are Rs. 12,000 per
month.
You are required to find out the following :
a)the product mix to yield maximum profit.
b)The most profitable product if only one product is to be
manufactured. Whether the answer will differ if license to import
raw material is released only for Rs. 1,80,000
[C.A., Final]
Solution:
Product wise contri bution per direct labour hourmunotes.in

Page 46

46Product P
Rs.Product Q
Rs.Selling price per unit215320Total Raw material cost per unit(Rs. 22 x Rs. 8) 30
(Rs. 28 + Rs. 32) 60
Direct Wages per unit
(Rs. 36 + Rs. 18 + Rs. 54) 108
(Rs. 54 + Rs. 36 + Rs. 7 2) 162Variable overheads per unit2314.30Total variable cost per unit 160 236.30
Contribution per unit (Sp –Vc) 54 83.70
Direct labour hours per unit 18 27
Contribution per direct labour hour
(contribution per DLH per unit) 3 3.10
Through the c ontribution per direct labour hour of production
Q is better but there is a limitation that workers in each department
can neither be interchanged nor newly recruited, hence due to this
following two alternatives are possible and company will have this
following two alternatives are possible and company will have to
choose between the two.
Alternative –I:Procuring 4,800 units of product Q and utilizing the
remaining available hours of labour for making units of product P as
number of hour in department D are only 57,600 sufficient to
produce only 4,800 units of product Q
Alternative –II :Producing 4,800 units of product P and utilizing
the remaining available hours of labour for making units of product
Qa sn u m b e ro fh o u r si nd e p a r t m e n tCa r eo n l y4 3 , 200 sufficient to
produce only 4,800 units of product P.
Statement of product mix under alternative Imunotes.in

Page 47

47Department Available
HoursHours
required for
4,800 units
of QRemaining
hoursHrs /
units of
product
PUnits of
product
P
(a) (b) c=( a ) –
(b)(d) (e)=(c)
/(d)
A
B
C
D72,000 WN
1
38,400
43,200
57,60043,200
28,800
--
57,60028,800
9,600
43,200
NIL6W N 239--4,800
3,200
4,800
--
The above table shows that out of the available hours under
alternative 1; 4,800 units of product Q and 3,200 units of Product P
can be made. This would result in 9,600 idle hours in department A
and 14,400 idle hours in Department D.
Statement of product mix under alternative I I
Department Available
HoursHours
required
for 4,800
units of PRemaining
hoursHrs /
units of
product
QUnits of
product P
(a) (b) c=( a ) –(b) (d) (e)=(c) /(d)
A
B
C
D72,000
38,400
43,200
57,60028,800
14,400
43,200
--43,200
24,000
--
57,6009W N2
6
--
124,800
4,000
--
4,800
The above table shows that out of the available hours under
alternative II; 4,800 units of product P and 4,000 units of product Q
can be made. This would result in 7,200 idle hours in department A
and 9,600 idle hours in department D
Profit statement under above alternatives.
Product First alternative Second alte rnative
Unit Contribution
P.U. (Rs.)Amount
(Rs.)Unit Contributi
on P.U.(Rs.)Amount
(Rs.)
P 3,200 54.00 1,72,800 4,800 54.00 2,59,200
Q 4,800 83.70 4,01,760 4,000 83.70 3,34,800
Total contribution 5,74,660 5,94,000
Less : fixed cost P.A. 1,44,000 1,44,000
PROFIT 4,30,560 4,50,000
Comments, although contribution per direct labour hour is
more in case of product Q, it is not advisable to produce 4,800 unitsmunotes.in

Page 48

48of Q first as it would result in more idle hours in some departments,
viz., department A and D as compared to that of second alternative.
It is, therefore, second alternative has given higher profit figure.
Hence second alternative is the most profitable product mix.
Statement of most profitable product if only one product is to
be manufactur ed.
Product P
Rs.Product Q
Rs.
Contribution per unit (Rs.) : (a)
Maximum possible output (in units) : (B)
Total contribution = (A) x (B)54.00
4,800
2,59,20083.70
4,800
4,01,760
Product Q is to be preferred as it would give more contribution.
State ment of most profitable product if only one product is to
be manufactured and license to import the raw material is only
worth Rs. 1, 80,000.
Product P Product QRaw material required P.U. (Rs.)3060Permissible output (in units) out ofimportedMaterial of Rs. 1,80,000 6,000 3,000
Maximum output possible inthe
available hours4,800 4,800
Output possible keeping in view the
availability
Of imported material and labourhours (units)4,800 3,000
Contribution per unit (Rs.) 54 83.70
Contributio np e rr u p e e so fm a t e r i a l 1.80 1.395
Total contribution : (Rs.) 2,59,200 2,51,100
(4,800 units (3,000 units
x Rs. 54.00) x Rs. 83.70)
Product P is to be preferred (i.e. answer differs) because it gives
higher contribution per rupee of material, which is a limiting factor.
Working notes :
1) Computation of total Labour hours available:munotes.in

Page 49

49Departments No. or Days Hrs / Total Hours
(a) workmen (c) days (e) = (b) x (c)(b)(d)x( d )A30300872,000B 16 300 8 38,400
C 18 300 8 43,200
D 24 300 8 57,000
2) Computation of hours required per unit of each product
First alternative Second alternative Department
Wages
(Rs.)
(a)Wages /
Hr
(Rs.)
(b)Hrs
(c) =
(a) + (b)Wages
(Rs.)
(d)Wages/H r(Rs.)
(e)Hrs
(f) =
(d) + (e)
A
B
C
D36
18
54
--6
6
6
--6
3
9
--54
36
--
726
6
--
69
6
--
12
Total hours per unit 18 27
Illustration : 8
A multi product company has the following costs and output
data for the last year.
Particulars ProductxYZSales mix 40 % 35 % 25 %
Rs. Rs. Rs.
Selling price 20 25 30
Variable cost per unit 10 15 18
Total fixed cost 1,50,000
Total sles 5,00,000
The company proposes to replace product Z by product S.
Estimated cost and output data are :
Sales mix 50 % 30 % 20 %
Selling price 20 25 28
Variabl e cost per unit 10 15 14
Total fixed costs 1,50,000
Total sales 5,00,000
Analyze the proposed change and suggest what decision the
company should take.munotes.in

Page 50

50[I.C.W.A., Inter]
Solution :
(1) Computation of Present Profit and BEP
2) Computation of Pro posed Profit and BEP
A comparison of the present situation and the proposed
situation shows that if product Z is replaced by product S, profit
would increase by Rs. 15,000 and breakeven point will reduce by
Rs. 21760/ -. This change is beneficial and th erefore product Z may
be dropped, provided all other relevant factors remain constant.
Illustration : 9. (Make or Buy)
Auto Parts Ltd. has an annual production of 90,000 units for
a motor component. The component’s cost structure is as below :munotes.in

Page 51

51
a)The pur chase manager has an offer from a supplier who is
willing to supply the component at Rs. 5.40. should the
component be purchased and production stopped ?
b)Assume the resources now used for this components
manufacture are to be used to product another new pr oduct for
which the selling price is 485.
In the latter case the material price will be Rs. 200 per units
90,000 units of this product can be produced on the same cost
basis above for labour and expenses. Discuss whether it would be
advisable to divert th e resources to manufacture the new products
on the footing that the component presently being produced would
instead of being produced, be purchased from the market.
[C.A. Inter]
Solution :
Statement showing the variable cost and purchase cost of
componen t. Used by Auto Parts Ltd.
Fixed expenses not being affected, it is evident from the
above statement that if the component is purchased from the
outside supplier, the company will have to spend Rs. 45 per unit
more and on 90,000 units the company will have to spend Rs.
40,50,000 more. Therefore, the company should not stop the
production of the component.
b)The following statement shows the cost implications of the
proposal to divert the available facilities for a new product.munotes.in

Page 52

52Statement showing the c ontribution per unit if the existing
resources are used for the production of another new product.
Thus, if the company diverts its resources for the production
of another new product, it will benefit by Rs. 15, i.e. Rs. 60 –45 per
unit. On 90,000 uni ts the company will save Rs. 13,50,000.
Therefore, it is advisable to divert the resources to manufacture.
The new product and the component presently being produced
should be purchased the market. This is also brought out by the
following figures :
Illustration : 10. (Export proposals)
Vinayak Ltd. opening at 75 % level of activity produces and
sells two products A and B. the cost sheets of the two products are
as under :
munotes.in

Page 53

53Factory overheads are absorbed on the basis of machine
hour which is limiting (key) factory. The machine hour rate is Rs. 2
per hour.
The company receives an offer from Canada for the
purchase of product A at a price of Rs. 17.50 per unit.
Alternatively, the company has another offer from the Middle
East for the purchase of produc t B at a price of Rs. 15.50 per unit.
On both the cases, a special packing charge of 50 p. per unit
has to be borne by the company.
The company can accept either of the two export order and
in either case the company can supply such quantities as may be
possible to be produced by utilizing the balance of 25% of its
capacity.
You are required to prepare:
a)A statement showing the economics of the two export proposals
giving your recommendations as to which proposal should be
accepted.
b)A statement showing th e overall profitability of the company
offer incorporating the export proposal recommended by you.
[C. A. Inter]
Solution:
(i) Economics of the two Export Proposalsmunotes.in

Page 54

54
Since machine hour is the limiting (Key) factor, the
contribution should be linked wit h the machine hours. This has
been worked out as follows:
Machine hour per unit 2.5 hour 1. 5h o u r
Contribution per machine hour Rs. 1.92 Rs. 2.13
Product B yield a better contribution per machine hour. The
order from the Middle East should therefore b e accepted as
compared to the Canadian offer.
Maximum hours at 100% activity; 2100 ÷ 75 x 100 = 2,8000 hours
Capacity hours available for export 2,800 –2100 = 700 hours
(ii) Statement of overall profitabilitymunotes.in

Page 55

55
Working notes:
1)Number of Units o fB:
Sales in the home market 400
Export market 700 / hr / 1.5 467
Total 867
2)Sales value of B :
400 units in the home market @ Rs. 19/ -Rs. 7,600
467 units for export @ Rs. 15.50 Rs. 7,239
Total Rs. 14,839
Illustration: 11. ( Decision about mechanization)
The present output details of a manufacturing department
are as follows :
Average output per week 48,000 units from 160 employees
Saleable value of output Rs. 6,00,000
Contribution made by output towards fixed expenses and
profit Rs. 2,40,000
The board of directors plans to introduce more
mechanization into the department at capital cost of Rs. 1,60,000.
The effect of this will be to reduce the number of employees to 120,
and increasing the output per individual employee by 60 %. To
provide the necessary incentive to achieve the increased output,
the Board intends to offer a 1 % increase on the price work rate of
Re. 1 per unit for every 2 % increase in average individual output
achieved.
[C.A. Inter]
Solution :munotes.in

Page 56

56Working No tes :
1)Present average output employee and total future expected
output per week
Present average output per employee per week
Total future expected output per week
=Total Number of future employees
[present output + 60 % of present output per em ployee]
=120 employees (300 units + 60 % x 300 units)
=57,60 units
2)Present and proposed price work rate:
Present price work rate =Re. 1.00 per unit
Proposed price work rate =Present Price work Rate + 30 %
xRe. 1
=R e .1 . 0 0+0 . 3 0
=R e .1 . 3 0p e ru n i t
present sale price per unit.
3)Present and proposal sales price per unit present sale price per
unit.
Present Sale price per unit = Rs. 12.50
(Rs. 60,000 ÷ 48,000 units)
Proposed sale price per unit = Rs. 12.00
(Rs. 12.50 –4% x Rs. 12.50)
4)Present marginal cost (excluding wages per unit) :
Present sales value –fixed expenses & profit –contribution
towards present wages
Present output (units)
=Rs. 6,00,000 –Rs. 2,40,000 –Rs. 48,000
=Rs. 6.50 per unit
48,000 units
State ment of extra weekly contribution (information resulting
from the proposed change of mechanization meant for board’s
evaluation)munotes.in

Page 57

57
Evaluation: Since the mechanization has resulted in the increase
of contribution to the extent of Rs. 1,920 per week, there fore the
proposed change should be accepted.
Illustration: 12. (Limited factor decision)
A company producing products ‘PIE’ n ‘SIGMA’ using a
single production process, has the following cost data
PIE SIGMA
Selling price per unit (Rs.) 20 30
Variab le cost per unit (Rs.) 11 16
Machine hours required per unit production (hrs) 1 2
Market limitation (units) 1lakh 2.5lakh
Total machine hours available –4l a k h s
Fixed cost per annum –Rs. 26 lakhs
Considering the limiting factors of machine hours and market
limitations, you are required to
a)Indicate the best combination of products to give optimum
contribution,
b)Show the additional machinery requirement to be augmented on
rental basis at an annual rent of Rs. 1.5 lakhs per machine to
provide additi onal capacity of 30,000 hours per machine;
(d)Change in number to be if the annual rental charges reduce to
Rs. 1,25,000 per machine
[I.C.W.A., Inter]
Solution :munotes.in

Page 58

58
a)Statement showing best combination of Product Mix towards
optimum contribution
b)Computation of Requirement of Additional Machines on Rental
Basis
munotes.in

Page 59

59Thus, the net contribution will stand reduced by Rs. 10,000
in case seven machines are hired. It is, therefore, better to hire only
six machines.
C)In case the annual rental c harges are reduced to Rs. 1,25,000,
the position will be as under;
Net contribution from 7th Machine = 20,000 hrs. x Rs. 7 –Rs. 1,25,000
=R s .1 5 , 0 0 0
Thus, the seventh machine will also give additional
contribution of Rs. 15,000.
Hence, in all seven machines may be taken on rent.
3.2 EXERCISE
Practical problems
Illustration 1:
From the following date you are required to present.
1)The marginal cost of product x and y and the contribution per
unit.
2)The total contribution and profits resul ting from each of the
suggested sales mixtures.
Variable expenses 100% of direct wages per product.
Fixed expenses (total) 800
Sales Price X 20.50 and
Y 14.50
Suggested sales mixes:
(M.Com. Oct. 97, adapted)
(Ans.: Contribution per Unit : Prod uct X -Rs. 4, Product Y -Rs. 2)munotes.in

Page 60

60Illustration 2 :
A manufacturer of packing cases makes three main types -
Deluxe, Luxury and Economy. Overheads are incurred on the
basis of labour hours. Wages are paid at Re. 1.00 per hour.
Estimates for the cases show t he following :
The manufacturer felt that he would be well advised to
discontinue producing the Delux and Economy cases even though
it would mean that some of production facilities would remain
unused. He cannot increase the sale of Luxury cases. It h as been
ascertained that 60% of the overheads is fixed.
You are required to advise the manufacturer.
(M. Com ., Mar. 98, adapted)
(Ans.: Contribution per Unit: Deluxe -Rs. 5.20, Luxury -Rs. 6.60,
Economy -Rs. 5.40)
Illustratio n3:
Suyash Ltd. is considering launching a new monthly
magazine at a selling price of 10 per copy. Sales of the magazine
are expected to be 5,00,000 copies per month but it is possible that
the actual sales could differ quite significantly from this esti mate.
Two different methods of producing the magazines are being
considered and neither would involve any additional capital
expenditure. The estimated production cost for each of the two
methods or manufacture, together with the additional marketing an d
distribution costs of selling the new magazine, are given below :munotes.in

Page 61

61
It may be assumed that the fixed cost element of the semi -
variable cost will remain constant throughout the range of activity
shown. The company sells a magazine covering related topic st o
those that will be included in the new publication, and
consequently, it is anticipated that sales of this existing magazine
will be adversely affected. It is estimated that for every ten copies
sold of the new publication, sales of the existing magaz ine are as
shown below : -
Sales 2,20,000 copies per month
Selling Price Rs. 8.50 per copy
Variable Costs Rs. 3.50 per copy
Specific Fixed costs Rs. 8,00,000 per month
You are required to calculate for each production method :
1)The net increase in company profit which will result from the
introduction of the new magazine, at each of the following levels
of activity :
5,00,000 4,00,000 6,00,000 copies per month.
2)The amount by which sales volume of the new magazine could
decline from the antic ipated 5,00,000 copies p.m., before the
company makes no additional profit from the introduction of the
new publication.
And also briefly identify any conclusions which may be
drawn from your calculations.
(M.Com., Oct. 01, adapted)
Illustration 4 :
From the following data, which product would you
recommend to be manufactured in a factory, time being the key
factor :
Per unit of Product A B
Direct Material 24 14
Direct Labour (Re. 1 per hr) 2 13
Variable Overhead (2 per hr) 4 6
Selling Pri ce 100 110
Standard time to produce 2hrs 3hrs
(M.Com .M a r .0 4 ,a d a p t e d )munotes.in

Page 62

62(Ans.: Contribution per Standard Hour: Product A -Rs. 35, Product
B-Rs. 25.67)
Illustration 5 :
From the following information you are required t o:
a)Calculate and present the marginal product cost and
contribution per unit.
b)State which of the alternative sales mixes you would
recommend to management? and Why?
Fixed overheads are Rs. 75,000 and variable overheads are 150%
of direct wages.
Alternative Sales Mix
1)2500 units of product X and 2500 units of product Y.
2)Nil units of product X and 4000 units of Product Y.
3)4000 units of Product X and 1000 units of Product Y.
(M.Com .M a r9 6 ,a d a p t e d )
(Ans.: Contr ibution per Unit: Product X -Rs. 20, Product Y -Rs. 40)
Illustration 6 :
Ap e nm a n u f a c t u r e rm a k e sa na v e r a g en e tp r o f i to fR s .
25.00 per pen on a selling price of Rs. 143.00 by producing and
selling 60,000 pens, or 60% of the potential capacity. His cos to f
sales is :
During the current year he intends to produce the same
number of pens but anticipates that his fixed charges will go up by
by 10% while rates of direct labour and direct material will increasemunotes.in

Page 63

63by 8% and 6% respectively. But he has no op tion of increasing the
selling price. Under this situation, he obtains an offer for a further
20% of his capacity. What minimum price will you recommend for
acceptance to ensure the manufacturer an overall profit of Rs.
16,73,000.
(M.Com .M a r .2 0 0 0 ,a d a p t e d )
(Ans.: Selling Price per Unit of 80% capacity: Rs.135.49)
Illustration 7 :
The price structure of an electric Fan made by the Vijaya
Electric Company Ltd. is as follows :
This cost is based on manufacture of 1,00,000 fans p.a. The
company expects that due to competition, they will have to reduce
the selling price. However, they want to keep the total profit infact.
You are required to prepare a statement showing the position, if
1)Selling price is reduced by 10% and
2)Selling price is reduced by 20%.
(M.Com . Oct. 2000, adapted)
(Ans.: P/V Ratio: Option I -44.44%, Option II -37.5%)
Illustration 7 :
Modern Chair Manufacturing Company received an offer to
sell 25,000 outdoor patro chairs to Easy Life Corporation. Modern
Chair Manufacturing Company produces 4,00,000 chairs annually
by operating at 80% of full capacity. Regular selling price for this
type of chairs is 33. the chairs required are similar to those
currently being produced by Mode rn Chair Manufacturing
Company.
Budgeted annual production costs and other expenses are
as follows :munotes.in

Page 64

64
The company wants to earn a minimum profit of Rupee one
per chair and no selling expenses will be incurred for special order
transaction. Assume tha tn o r m a lo p e r a t i o n sw i l ln o tb ea f f e c t e db y
the special order and that regular sales volume for the year is
4,00,000 chairs as initially planned. You are required:
What should be the minimum rice to be quoted by Modern
Chair Manufacturing Company?
Prepar e an income statement showing the position of the
company without special order, for special order and with special
order.
(M.Com . Oct. 2004, adapted)
(Ans.: Minimum Price to be quoted Rs. 18.75)
Illustration 8 :
A manufacturer has planned his level of operation at 50% of
his plant capacity of 30,000 units. His expenses are estimated as
follows, if 50% of the plant capacity is utilized.
i)Direct Materials Rs. 8,280
ii)Direct Materials Rs. 11,160
iii)Variable and Ot her Manufacturing Expenses Rs. 3,960
iv)Total Fixed Expenses irrespective of Capacity Rs. 6,000
utilization
The expected selling price in the domestic market is Rs. 2
per unit. Recently the manufacturer has received a trade enquiry
from an Overseas Or ganisation interested in purchasing 6,000 units
at a price of Rs. 1.45 per unit.
As a Professional Management Accountant, what should be
your suggestion regarding acceptance or rejection of the offer?
Support you suggestion with suitable quantitative inf ormation.
(M.Com . Oct. 2006, adapted)
(Ans.: Total Contribution: Present position Rs. 6,600, Proposed
Offer Rs. minus 600)
Illustration 9:munotes.in

Page 65

65X Ltd. manufactured and sold 14,000 units and 18,000 units
in the first year and the se cond year respectively. The selling price
per unit was Rs. 100 in both the years. In the first year it suffered a
loss of 20,000 and in the second year earned profit of Rs. 20,000.
calculate the following :
a)The amount of fixed cost
b)The BEP in units and in Sales Value
c)Profit when 26,000 units are sold
d)The number of units to be sold to earn post -tax profit of `
30,000. Tax rate is 40%.
X Ltd. estimates that its sales will be ` Nil in the next year. The
competitor has made an offer that it would buy the products of X
Ltd. at present selling price less 10% with a condition that X Ltd.
should purchase competitor’s product equal to double of the units
purchased by the competitor from X Ltd. The competitor’s product
selling price is Rs. 90 and fetches contribution of Rs. 25 per unit. If
the competitor’s offer is accepted, calculate:
a)BEP in units purchased and sold by X Ltd.
b)No. of units to be purchased and sold to earn profit of Rs.
40,000
(M.Com .M a r2 0 0 7 ,a d a p t e d )
(Ans.: Contribution per Unit: Rs. 10, Fixed Cost Rs. 1, 60,000)
Illustration 10 :
Following relevant data of a firm is given :
Activity Levels (tons)
50,000
tons60,000
tons70,000
tons80,000
tons
Variable Cost ...... ...
(`in thousands)
Semi -Variable ...... ...
cost (`inthousands)
Fixed Cost ...... ...
(`in thousands)
Total Cost ...... ...
(`in thousands)5,000
1,500
2,500
9,0006,000
1,600
2,500
10,1007,000
1,650
3,000
11,6508,000
1,700
3,000
12,700
The fixed costs follow step -graph pattern as is clear from the
above and the semi -variable costs change at uniform rate between
the above given activity levels. Given that the firm operates 55,000
tons level at present –munotes.in

Page 66

661)Calculate the additional / increm ental costs if it manufactures
additional (a) 10,000 tons (b) 15,000 tons.
2)Advise whether the firm should accept ‘any one’ of the following
additional (special) export market offers and if Yes, ‘which one’
should it accept :
i)for 10,000 tons at a sell ing price of Rs. 125/ -per ton.
ii)for 15,000 tons at a selling price of Rs. 150/ -per ton.
(M.Com .A p r .2 0 0 9 ,a d a p t e d )
(Ans.: Additional Incremental Cost in thousands -10,000 Tons Rs.
1,075, 15,000 Tons Rs. 2,100 )

munotes.in

Page 67

67MODULE -II
Unit-4
STANDARD COSTING -I
Unit structure :
4.0 Objectives
4.1 Introduction
4.2 Standard Costing
4.3 Standard Cost
4.4 Benefits of Standard Costing
4.5 Fixation of Standards
4.6 Analysis of Variances
4.7 Material Cost Var iances
4.8 Labour Cost Variances
4.9 Overhead Cost Variances
4.10 Sales Variances
4.11 Limitations of Standard Costing
4.12 Exercises.
4.0 OBJECTIVES
After Studying this unit, you would be able to:
Understand the terms Standard Cost and Standard Costing
Understand the process of setting the standards for various
elements of costs.
Understand how standard costing operates.
Explain the benefits of standard costing
Calculate the material, labour, overhead and Sales Variances
Understand the use of sta ndard costing for cost reduction
4.1 INTRODUCTION
Managers are constantly comparing their product cost
with the budgets. The reasons for deviations are constantly
analyzed and responsibilities are promptly fixed. Thus, “What a
product should have Coast ed” is a question of great concern to the
management for improvement of cost performance. Standard
costing is a managerial device to determine efficiency andmunotes.in

Page 68

68effectiveness of cost performance. Standard costing explains the
difference between actual profit and profit as per standard relating
to the operating period. It also helps to explain the variances
according to their causes and responsibilities.
4.2 STANDARD COSTING
Standard costing is a control technique which compares
standard costs and revenues wi th actual result to obtain variances
which are used to stimulate improved performance. Use of
standard costing is not confined to industries having repetitive
processes and homogeneous product only. This technique has
established the advantages of its use in industries having non
repetitive processes like manufacture of automobile, turbines,
boilers and heavy electrical equipment.
4.3 STANDARD COST
Standard Cost is a scientifically pre -determined cost, which
is arrived at assuming a particular level of ef ficiency in utilization of
material labour and indirect services. CIMA defines standard cost
as “a Standard expressed in money. It is built up from an
assessment of the value of cost elements. Its main uses are
providing bases for performance measurement, Control by
exception reporting, valuing stock and establishing selling prices”.
reveals a very useful information for cost control.
Standard cost is like a model which provides basis of
comparison for actual cost.
This comparison of actual cost with stan dard cost control.
4.4 BENEFITS OF STANDARD COSTING
The benefits of standard costing are as follows:
1)Use of standard costing leads to optimum utilization of men,
materials and resources.
2)Its use provides a yardstick for comparison of actual cost
performance.
3)Only distinct deviations are reported to management. Thus, it
helps application of the principle of “management by exception”
4)It is very useful to management in discharging functions, like
planning control, decision –making and price fix ation.
5)It creates an atmosphere of cost consciousness.munotes.in

Page 69

696)It motivates workers to strive for accomplishment of defined
targets. It precipitates an attitude that is conducive to efficiency.
7)It highlights areas, where probe promise improvement.
8)Itsintroduction leads to simplification of procedures and
standardization of products.
9)Its introduction enables the management to reduce time
required for preparation of reports for pricing, control or
quotation purposes.
10)Its use enables to find out th e cost of finished goods
immediately after completion.
11)If standard costing is used, stock ledgers can be kept in terms of
quantities only. This eliminates much clerical effort in pricing,
balancing and posting on stores ledgers cards.
12)Its use may e ncourage action for cost reduction.
4.5 SETTING OF STANDARDS
Determination of standards for various elements of cost is an
exercise that requires skill, imagination and experience. For setting
standards, routines and process of working conditions are
thoroughly studied and motion studies are conducted and different
tests are carried out to ensure that standards are realistic and
conform to management’s view of efficient t operations and relevant
expenditure. The job of setting the standards is done by a g roup,
which is represented by Engineering Department, production
Department, Purchase Department, personnel Department and
Cost Accounts Department, Setting of standards can Cost
Accounts Department. Setting of standards can be divided in two
categories:
i)Determination of quantity standards; and
ii)Determination of price standards.
Quantity standards are pre -determined expressing in
physical terms the relationship between a unit produced and
resources consumed. Price standards are pre -determined
measures expressing in money terms the cost per unit of resources
consumed. Quantity standards are developed by representatives of
Engineering Department in liaison with representatives of purchase
Department. Wage rate standards are developed by personnel
Department. Accounts department works in advisory capacity
supplying the information based on historical costing.munotes.in

Page 70

704.6 ANALYSIS OF VARIANCES
The comparison of actual performance with standard
performance reveals the variance. A variance represents a
deviati on of the actual result from the standard result. There can be
cost variance, profit variances, sales value and operational and
planning variances. Whether a variance is favorable or unfavorable
is ultimately determined with reference to is impact on profi t. For
example a variance will be adverse, if the actual cost exceeds the
standard cost or vice versa. Profit variance will be favorable if
actual profit exceeds standard profit or vice versa. Variance
analysis is an exercise, which involves efforts to iso late the causes
of variance in order to report to management those situations which
can be corrected and control by timely action . The extent to which
the causes are established, depends upon the amount of time effort
and money, that a company is willing to spend in accumulating data
as that variance occur. In variance analysis a point is reached
where incremental information is not worth its incremental cost.
This point indicates the limit of variance analysis and it is
determined by judgment in the light of individual circumstances.
Variance analysis must be devised to suit the conditions prevailing
within a particular enterprise. Analysis of variances must be
followed by intelligent and factual interpretation. Computation,
classification and reporting of variances is a vital feature of
standard costing.
4.7 MATERIAL COST VARIANCE
It represents the different between actual cost of material
used and standard cost of material specified for output achieved.
Material cost variance arises due to variation in prices and usage of
materials. The following formula is used to find out material cost
variance:
Material cost variance = Standard cost of Material used –Actual
cost of material
Materials cost variances are as follows: -
a)Material Price Variance: It is that part of material cost variance
which due to the different between the actual price paid and
standard price specific for the material. It is determined as
follows:
Material price variance = Actual quantity (Standard price –Actual
price)munotes.in

Page 71

71b)Materi al usage variance: It is that part of material cost variance
which due to different between the actual quantity used and
standard quantity specific for output . This indicated whether or
not material was properly unitized, It is determined as follows
Material usage variance = standard price ( standard quantity –
actual quantity)
c)Material Mix variance: It is that part of material usage variance
which is due to different between the actual composition of mix and
standard composition mixing the different types of material. It is
determined as follows:
Material mix variance = standard price (revised standard quantity –
actual quantity)
d)Material yield variance: It is that portion of material usage
variance which is due to different between the actual yie ld optance
and standard yield specific. It determined as follows:
Material yield variance = standard price (actual output –standard
output)
Illustration 1
Yes Ltd manufacture a single product the standard cost of which is
as follows :
Material A 60% @ Rs20/ per Kg.
Material B 40% @ Rs10/ per Kg.
Normal lost is 20% of input . Due to shortage of material A the
standard mix was changed. Actual result for January 2011 were as
follows :
Material A –105 Kg. @ Rs20/ per kG
Material B –95 Kg @ Rs9/ per K g.
Input 200 kg
Loss 35 Kg
Output 165 Kg
Calculate A Material cost variance B Material price variance C
Material usage variance D material mix variance E material yield
variance.
Solution A Material Cost Variance is = Standard cost of Material
used –Actual cost of materialmunotes.in

Page 72

72Rs 3300 –Rs2955 = Rs345F
Actual cost of material used
A-105kg @ Rs20 = Rs2100
B–95 kg @ Rs9 = Rs 855
Total = Rs2955
Standard cost of material = A 123.75 @ Rs20 = Rs2475
B8 2 . 5 @ Rs 10 = Rs825
Total = Rs3300
123.75 = 120 X 165/160 82.5 = 80X 165/160
BMaterial price variance = Actual quantity (Standard price –Actual
price)
A=1 0 5 ( 2 0 -20) = 0
B=9 5( 1 0 -9) =9 5 F
Total =9 5 F
C Material usage variance = standard price (standard qua ntity –
actual quantity)
A=2 0 X(495/4 -10 5 )=3 7 5F
B=1 0( 1 6 5 / 2 -95)=1 2 5A
Total =2 5 0F
DMaterial Mix variance = standard price (revised standard quantity
-actual quantity)
A=2 0 ( 1 2 0 -105) =3 0 0F
B=1 0( 8 0 -95) =1 5 0A
Total=1 5 0F
EMaterial yi eld variance = standard price ( actual output –standard
output) .
20 (165 -160) = 100 F
Standard price of output
Material A= 60 x 20 = Rs. 1200
Material B= 40 x 10 = Rs. 400
Total = 100 = Rs. 1600
Loss 20
Outpu t 80
Standard Cost Price = Rs. 1600/80 = Rs. 20
Check Your Progress:
1) Define the following terms
(a)Standard Cost
(b)Material Price Variances
(c)Material Mix Variances
(d)Material yield variancesmunotes.in

Page 73

732)Give formulas of the following
a)Material cost variances
b)Material Price Variances
c)Material Mix Variances
d)Material yield variances
4.8 LABOUR COST VARIANCE
Labour cost variance is different between the actual wages
paid and standard wages specific wages for the production. It is
calculated as follows:
Labour cost variance = Standard labour cost –Actual labour cost
STD Hours X STD Rate –Actual Hours X Actual rate.
Following are the labour cost variance.
A)Labour rate variance: It is that portion of labour cost variance
which due to the different betw een the actual labour rate and
standard labour rate specific. It determined has follows :
Labour rate variance = Actual hours ( standard rate –Actual rate )
B)Labour efficiency variance: It is that part of labour cost
variance which due to the differen t between the actual hours paid
and standard hours allowed for output achieved. It determined as
follows :
Labour efficiency variance = standard rate (standard hours –actual
hours)
C)Labour mix variance: Production may be completed if labour is
mixed a ccording standard proportion. Standard mix may not be
adhered to under sum circumstances and substitute will have to
made. It is determined as follows: -
Labour mix variance = standard rate (actual labour mix –revised
standard labour mix)
D)Labour yiel d variance: It is that part of labour efficiency
variance which is due to different between actual output and
standard output of workers specific . It is determined as follows: -
Labour yield variance = average standard labour hours rate (actual
production –standard production on actual hour)
Check Your Progress:
1)Give the formulas of the following
a)Labour Cost variancesmunotes.in

Page 74

74b)Labour rateVariances
c)Labour effi ciency Variances
d)Labour mix variances
e)Labour yield variances
2)Explain the following terms
a)Labourcost variances
b)Labour rate Variances
c)Labour effi ciency Variances
d)Labour mix variances
e)Material yield variances
Illustration 2
Standard labour hours and rate for production of one unit of
article A is given below:
Actual Data
Articles Produces 1000 units
Calculated
1.1 Labour cost variance
2.Labour rate variance
3.Labour efficiency variance
4.labour mix variance
5.labour yield variance
Solution:
1)Labour cost variance = Standard labour cost –Actual labour
cost
Skilled = STD Hours XS T DR a t e –Actual Hours X Actual rate.
5000 X1 .5 –4500 X 2 = 7500 -9000= Rs 1500 Amunotes.in

Page 75

75Unskilled = 8000 X 0.5 –10000 X 0.45 = 4000 -4500 = Rs 500 A
Semi Skilled = 4000 X 0.75 -4200X0.75 = 3000 -3150 = Rs 150 A
1500 + 500 + 150 = Rs2150 A
4.9 OVERHEAD COST VARIANCE
It is different between standard overhead cost and actual
overhead cost of producing goods . There are two types of
overhead variance:
A)Variable Overhead Variance
B)Fixed Overhead Variance
A) Variable Overhead Variance :It is different between standard
variable overhead cost and actual variable overhead cost. It
determined as follows:munotes.in

Page 76

76Variable overhead cost variance = Standard variable overhead cost
–actual variable overhead cost.
Variable overhead cost is again divi dend into two parts
1)Variable overhead expenditure variance: It is that portion of
variable overhead variance which arrives due to difference between
actual overhead and standard variable overhead appropriate to the
level activity. It determined as follo ws:
Variable Overhead Expenditure variance = standard variable
overhead at actual level –actual variable overhead
2)Variable overhead efficiently variance : It is the difference
between actual hours work at standard variable overhead rate and
standard variable overhead for production. It is determined as
follows :
Variable Overhead Efficiency Variance = standard variable
overhead –actual variable overhead for production.
Check your progress :
1)Explain the following terms :
a)Variable Overhead cost Variance
b)Variable Overheads expenditure Variance
c)Variable Overheads efficiency Variance
2)Give the formulas of the following :
a)Variable Overheads cost Variance
b)Variable Overheads expenditure Variance
c)Variable Overheads efficiency Va riance
Illustration 3
Following information is obtained from Wise Ltd
Budgeted production for the period 600 units
Budgeted variable overhead Rs 15600/ -
Standard Time for one unit 20 hours
Actual Production for the period 500 units
Actual Variable overh ead Rs 14000/ -
Actual Hours Worked 9000 Hrs
Calculate:
A Variable overhead expenditure variance
B variable overhead efficiency variance
C Variable Overhead variancemunotes.in

Page 77

77Solution:
A VO Expenditure variance = standard variable overhead at actual
level –actual variable overhead
=1 1 7 0 0 -14000 = 2300 A
SVOH = 15000/1 2000 X 9000 = 11700
B VO Efficiency Variance = standard variable overhead –actual
variable overhead for production.
=1 3 0 0 0 -11700= 1300 F
C Variable overhead cost variance = Standard variabl eo v e r h e a d
cost –actual variable overhead cost.
=1 3 0 0 0 -14000= 1000 A
4.10 FIXED OVERHEAD COST VARIANCE
It represents the difference between actual fixed overhead
incurred and standard cost of fixed overhead absorbed. It
determined as follows:
Fixed o verhead Cost variance = Standard cost of fixed overhead –
actual fixed overhead
Fixed overhead variance is follows:
A. Fixed overhead volume variance -It that part of fixed
overhead variance which due different between actual fixed
overhead incurred and standard allowance for fixed overhead.
Fixed overhead volume variance = standard rate X actual
output -budgeted overhead
B. Fixed overhead expenditure variance -It that part of fixed
overhead variance which is due different between actual fixed
overhe ad incurred and budgeted fixed overhead.
Fixed overhead expenditure variance = budgeted fixed
overhead –actual fixed overhead.
C. Fixed overhead Calendar Variance -It is that part of fixed
overhead volume variance which is due to different between
budg eted fixed overhead and fixed overhead for dates available
during the period at standard rate.
Fixed Overhead Calendar variance = standard rate ( actual
quantity –Standard quantity)munotes.in

Page 78

78D. Fixed overhead efficiency variance: It that portion of volume
varian ce which reflected increase or reduced output arising from
efficiency being above or below standard.
Fixed overhead efficiency variance = standard rate (actual
production –standard production)
E. Fixed overhead capacity variance: It is that part of fixe d
overhead variance which arrives due to different between capacity
utilized and avai lable capacity.
Fixed overhead capacity variance = Standard rate (revised
budgeted -budgeted unit)
Check your progress :
1)Fill in the blanks :
a.-------------- repre sents the difference between actual fixed
overhead incurred and standard cost of fixed overhead
absorbed.
b)The different between actual fixed overhead incurred and
standard allowance for fixed overhead means ----------------- .
c)The different between actual fixed overhead incurred and
budgeted fixed overhead means ---------------- .
d)--------------- variance is the portion of volume variance.
e)--------------- variance arrives due to different between capacity
utilized and available capacity.
f)--------------- variance reflects increase or reduced output arising
from efficiency being above or below standard.
g)Fixed overhead calendar variance is the different between ------
and------------ available during the period at standard rate.
Illustr ation 4
From the following data calculate overhead variance :
There was an increase of 5% in capacity.munotes.in

Page 79

79Solution:
1.Total overhead variance = Actual output in standard rate –
Actual overhead
16000 X 5 –(30500+ 47000)
Rs 2500 F
2.Variable Ov erhead Variance = Standard rate X Actual output –
Actual Overhead
3X1 6 0 0 0 –47000 = 1000F
3.Fixed Overhead Variance = Standard rate X Actual Output –
Actual Overhead
2X1 6 0 0 0 –30500 = Rs 1500F
4.Fixed overhead volume Variance = Standard Rat e X Actual
Output –Budgeted Overhead
2X1 6 0 0 0 -30000 = 2000 F
5.Fixed overhead Expenditure Variance = Budgeted Fixed –
Actual Fixed overhead
Rs 30000 -Rs 30500 = Rs 500A
6.Fixed Overhead Capacity Variance = Standard Rate (Revised
Budgeted Uni ts–Budgeted Units)
Rs 2 (15750 -15000) = 2 X750 Rs 1500 F
7.Fixed overhead calendar variance = Standard Rate (Actual
Quantity –Standard Quantity)
2 (1 5750/25 X2) = Rs1 260 X 2 Rs 2520 F
8.Fixed overhead efficiency variance = Standard Rate ( Actual
Production –Standard Production)
=2( 1 6 0 0 0 -17 0 1 0 )=R s2 0 2 0A
Note: Standard production = 15000 units
+ Increase Due Capacity increase = 750
+_ Increased production for 2 days = 1260
Total = 17010
4.11 SALES VARIANCE
Sales vari ance is difference between the actual value
achieved in a given period and budgeted value of sales. Sales
value variance are useful for sales managers in determined the
effect of changes in different factors on sales value. The following
are Types of varia nce.munotes.in

Page 80

80A)Sales value variance -It is the different between the actual
sales value reliable and the standard value of sale as per the
Budget.
Sale value variance = Actual Quantity X Actual price -Standard
quantity X Standard Price
B)Sales price variance -It is that portion of the total sales value
variance which due to different between actual sales price realised
and budgeted sales price
Sales price variance = Actual Quantity ( Actual price –
standard price)
C)Sales volume variance -It is that port ion of total sales value
variance which due to difference between the standard value the
actual sales effected and standard value of sales as per the budget.
Sales volume variance = standard price (Actual quantity –
Standard quantity)
D)Sales Mix varian ce-It is that portion of sales volume variance
which due the difference between the standard value of actual
sales effected and Actual value of Sales.
Sales mixed variance = Standard Price ( Actual proportion –
revised standard mix actual sales)
E) Sal es quantity variance -It is that portion of sales volume
variance which due tot the difference between standard value of
actual sales effected and standard values of sales as per the
budged.
Sale quantity variance = standard price ( revised standard mix
–standard mix)
Check Your Progress :
1)Give formulas.
a)Sales value variances
b)Sales price variances
c)Sales volume variances
d)Sales mix variances
e)Sales quantity variances
2)Fill in the blanks.
a.Sales variance is difference between th e actual value achieved
in a given period and --------------------- .munotes.in

Page 81

81b.----------------- is the different between the actual sales value
reliable and the standard value of sale as per the Budget.
c.Sales price variance is the different between -------- -------- and
budgeted sales price.
d.---------------- variance is the difference between the standard
value the actual sales effected and standard value of sales as
per the budget.
e.Sales mix variance is the difference between the ---------------
andActual value of sales.
Illustration 5
From the following information about sales calculate:
A) Total sales variance B) Sales price variance C) Sales volume
variance D) Sales mix variance E) Sales quantity variance
STANDARD
ACTUAL
Solution :
munotes.in

Page 82

82
4.12EXERCISE
1)What standard costing ?Explain the advantages and limitations
of costing .
2)What is meant by standard cost? How standard cost is
determined?
3)Choose the right answer with your reasoning: -
1)Material cost variance arises due to v ariation in price and
--------------- of materials.
(a) Quality (b) Quantity (c) Volume (d) Delivery
2)Idle time variance is due to difference between labour hours
applied and labour hours ---------------- .
(a) Utilised (b) Supplied (c) Unutilised ( d) Underutilised
3)Fixed overhead capacity variance arises due to difference
between capacity utilized and ---------------- capacity.
(a) Spare (b) Excess (c) Fixed (d) Planned
4)Sales mixvariance is due to the difference between standard
value of a ctual sales and actual value of sales -----.
(a) Realised (b) Effected (c) Margin (d) Volume
5)Standard cost is a specifically --------------- cost.
(a) Pre -determined (b) Estimated (c) Planned (d) Average
(Ans -1-b,2 –a,3 –d,4 –a,5 –a)
munotes.in

Page 83

83Unit-5
STANDARD COSTING -II
Unit structure :
5.0 Objectives
5.1 Solved Problems
5.2 Exercise
5.0 OBJECTIVES
After studying this unit the students will be able to solve the
problems on variance analyses.
5.1 SOLVED PROBLEMS
Illust ration 1
The standard cost of a certain chemical mixture is as follows:
Material Cost per tonn (Rs.)
I (40%) 20
II (60%) 30
A standard loss of 10% is expected in production.
For a period, the actual consumption data was as follows:
Material Cost per tonn (Rs.)
I (180 tonnes) 18
II (220 tonnes) 34
The actual weight produced was 364 tonnes.
Calculate the Material cost variance, Material price variance
and Material quantity variance.
(M.Com. April 2011)munotes.in

Page 84

84Solution :
Actual cost of Material used ( M1)
Standard cost of Material used (M2)
Standard cost of material, if it has been instandard
proportions (M3)
Adding (i) and (ii) we get the value of M3
M3 = Rs. 3,200 + Rs. 7,200 = Rs. 10,400
Standard cost of output. (M4)
Letu sf i n do u tt h e standard cost, when input is 100 tonn.
It means for output of 90 tonnes standard cost will be Rs. 2,600
Standard cost of actual output of 364 tonnes will be
munotes.in

Page 85

85Variance
1.Material price variance =M1–M2
=10,720 –10,200 = Rs. 500 (A)
2.Material Mix variance =M2–M3
=10,200 –10,400 = Rs. 200 (F)
3.Material Yield variance =M3–M4
=10,400 –10,516 = Rs. 116 ( F)
4.Material cost variance =M1–M4
=10,720 –10,516 = Rs. 204 (A)
Alternatively it can be found out as follows:
=Material price variance + Material Mix variance + Material
Yield variance
=Rs. 520 (A) + Rs. 200 (F) + Rs. 116 (F) = Rs. 204 (A)
5. Material Usages variance = M2 –M4
=1 0 , 2 0 0 –10,516 = Rs. 316 (F)
Alternatively it can be found out as follows:
=Material Mix variance + Material Yield variance
=Rs. 200 (F) + Rs. 116 (F) = Rs. 316 (F)
Illustration 2
The budgeted and the actual sales for a Quarter ended 31st
March, 2012 in respect of three products are given below:
Calculate:
1.Sales Value Variance
2.Sales Price Variance
3.Sales Volume Variance
4.Sales Mix Variance
5.Sales Quantity Variance
(M.Com. April 2012)munotes.in

Page 86

86Solution :
munotes.in

Page 87

87
Illustration 3:
From the following information calculate Labour Rate
Variance, Labour Cost Variance and Labour Efficiency Variance:
Standard Data for 20 units:
Standard Hours 60 Hours
Standard Rate Rs. 21 per unit
Actual Data:
Actual Production 15,000 units
Actual Hours Worked 46,500 Hours
Actual Rate Rs. 6.80 per Hour
(M. Com April 2012)
Solution :
From the given information :
Standard Hours per unit =Standards hrs.
Standar du n i t s
=60 hrs.
20 units
=3h o u r sp e ru n i t
Standard Rate per hour =Standard rate per unit
Standard hours per unit
=Rs. 21
3h r s
=Rs. 7 per hour.
Given Data
Variances:
1)Labour cost variance =(SH X SR) –(AH X AR)
=(45,000 X7 ) –(46,500 X 6.80)
=3,15,000 –3,16,200
=1,200 (A)munotes.in

Page 88

882)Labour rate variance =AH ( SR –AR )
=46,500 ( 7.00 –6.80)
=9,300 (F)
3)Labour efficiency variance = SR (SH –AH)
=7( 4 5 , 0 0 0 –46,500)
=7(-1,500)
=1 0 , 5 0 0( A )
Illustration 4:
AB Costing Ltd. manufactures a costing the standard
specification are as under :
The actual date for a period as follows :
a)Output obtained 120 Kgs.
b)Input of raw material :
Raw material P. . . . . 112 Kgs. at actual price of Rs. 85 per Kg.
Raw material S. . . . . . 28 Kgs. at actual price of Rs. 15 per Kg.
Calcu late the material cost variances.
Solution :
The calculate the material cost variances, proceed as follows :
a)For the output obtained, find out how much raw mater ial
should have been used.
b)In this case, the output obtained was . . . . 120 Kgs.
c)Hence as per standard specified, the total input should have
been only 120 Kgs. and the break up should have been as
follows and the Std. Cost per unit of output.
Raw material P. . . . 90 Kgs. @ Std. Price of Rs. 30 = Rs.2,700
S... . .30 Kgs. @ Std. Price of Rs. 20 = Rs. 600
120 Kgs. Rs. 3,800munotes.in

Page 89

89The standard cost the material in the product is Rs. 27.50 per kg.
d)Now let us construct the standard and acutal data for 120
Kgs. of output :
The material cost variance can now be calculated :
Table
According to the definitions, we have
a)Price Variance
=(Actual quantity @ Actual price) –(Actual Quantity @ Std. Price)
=(Col. A –Col B)
=Rs. (440 –8920)
=Rs. 420 advance as actual cost is more than the Std. cost.
b)Usage variance
=(Actual quantity @ Std. Price) –(Std. Quantity @ Std. Price)
=(Col. B –Col. C)
=Rs. 8920 –8800
=620 Advers e as std. cost of actual is more than the std. cost of
standard allowance.
c)Total variance = Price variance + Usage variance
= Rs. 420 A + Rs. 620 A
= Rs. 1040 adverse
The actual yeild obtained was 120 Kgs.
Hence we should have 120 Kgs. of materia l at standard mix.
That is to say, the consumption ofPand S should have
been.
Standard consumption for 120 kgs. yield:
Raw material P 80 Kgs.
S 40 Kgs.
120 Kgs.munotes.in

Page 90

90But the actual consumption was 140 kgs. yielding 120 kgs.
hence, we compare :
a)The a ctual consumption @ std. mix gave yield of 120 Kgs.
b)The standard allowed for a yield of 120 Kgs.
Table
The yield variance is given by the difference between:
Rs.
a)Std. cost of actual consumption that yielded 120 Kgs. =3 , 8 5 0
b)Std. cost of allowed consumption that yielded 120 Kgs. =3 , 8 5 0
Yield variance unfavorable =5 5 0
Illustration 5
For product A, F.O.G. is a compound made up of three raw
materials F, O and G, in the standard proportion given below. The
standard price per Kg . is also given. During January 1st week,
actual input was for 100 mix and the other data are as under –
munotes.in

Page 91

91Table
Suggested Solution :
Total Variance = Price Variance + Mix Variance + Yield Variance
(A-D) (A-B) (B-C) (C-D)
Rs.(2210 -2178) =Rs.(2210 -2150) +Rs.(2150 -2250) +Rs.(2250 -2178)
82 A Rs.61 A + Rs. 100 F + Rs. 70 A
Note: -
a)The actual yield was 43.56 kgs
Which is equal to 90% as per standard.
b)Hence, the actual in put should be
Table for Example No.2
We can summaries the above calculations in the various
tables and list them as shown below. In fact for examination work
this table can be quickly written up and variances obtained.
munotes.in

Page 92

92
Hence the standard quantit y (that should have been used)
for each raw material and the standard cost:
For Self Examination
1)What do you understand by:
a)Price Variance
b)Usage Variance
c)Mix Variance
d)Yield Variance
By assuming certain data work out each of these va riance.
2)What is the relationship between usage variance, mix variance
and yield variance?
3)From the following data calculate the material cost variances
and check your answer with those given belowmunotes.in

Page 93

93
The above quantity represents the standard mix f or 80 mixes.
The actual mix for 80 mixes during a period was.
Illustration 6
A contract work was estimated to be performed in a week of
40 hours by a labour force constituted as follows at the hourly rates
given below.
The Standard Mix of Labour gr ade :
40 Men at Rs.0.60 per hour
20 Women at Rs.0.30 per hour1070boys at Rs. 0.30 per hourmunotes.in

Page 94

94Standard Mix of Labour grade is as follows.
Men 40/70
Women 20/70
Boys 10/70
But however, as sufficient men were not available, the actu al
composition of Labour force employed and their rates of pay the job
as under.
50 Men at Rs.0.60 per hour
20 Women at Rs.0.45 per hour 10 Women at Rs.0.30 per hour 15
Boys at Rs.0.32 per hour
And the work was completed in 50 hours and each of them was
paid for the 50 hours.
Calculate the loss or gain on the Labour Cost of the contract and
analyse the variance.
Answer
The first step is to be prepare a Standard Labour Cost of the job ....
See Table A.
The second step is to prepare the Actual Cost incur red………..
See Table B.
Table A
Standard Labour Cost Rs.
Table B
The Actual composition of the Labour Cost
Hence from (A) and (B) the total Labour Cost Variance is Rs.840
Adverse i.e. Rs.1400 –Rs.1740 = Rs.340 it is adverse because the
Actual Cost i s more than the Standard Cost.
Now, let us analyse the Labour Cost variance under –
a)Wage Rate Variance
b)Time Usage Variancemunotes.in

Page 95

95Thereafter analyse the Time Variance under –
c)Labour Mixture Variance
d)Labour Efficiency Variance
Wages Rate Variance Ca lculation
In the third step we determine the wage rate variance which
is the First Variance that should be calculated.
The wage rate variance analyse is prepared by comparing
the data of (a) and (b) given below.
a)The Actual Wages paid i.e. The Actual t ime worked X the Actual
rate paid.
b)The wages that should have been paid for the Actual Hours at
the Standard rate specified.
The formula : The Actual time worked X Standard rate of Wage.
From table B. We have the data for Actual Wages @ Actual rates
paid.
Table B
2. We prepare table C to calculate the wages that should have
been paid for the Actual Hours if the Rate per Hour had been as
per the Standard.
Table C
munotes.in

Page 96

96Now comparing the wages in Table B and Table C we find that –
a)The Actual Hours a re the same in both the cases.
b)But in Table B we have used the Actual rate and in Table C we
have used the Standard Rate.
Therefore, we get the Variance arising due to wage difference only
and this known as Wage Rate Variance.
Calculation of Wage Rate Variance
The Variance is Adverse because the Actual Wages is more than
the Standard Wages. Usage of Time Variance.
We determine this variance in the fourth step.
This is arrived at by comparing (c) and (d).
The standard time @ Standard Wage rate, an d
(d) The Actual time @ Standard Wage rate.
The calculation already shown in Table A and C respectively are
reproduced here for easy reference.
Table D
The difference between Rs.1400 (Standard Hours X Std. Rate and
Rs.1725) (Actual Hours X Standard R ate) = Rs.325 Adverse
represents the Variance due to usage of Time.
As the (Actual Hrs. X Std. Rate) is greater than the Standard
Wages (Standard Hours X Std. Rate) the variance is Adverse.munotes.in

Page 97

97CheckThe Total Labour Wage Rate Usage ofCost Variance Variance Time            Rs.34C (A) =Rs.15 (A) + Rs.325 (A)
Note
After you have carried out this primary variance analysis we can
proceed to analyse the usage of Time Variance into its further
components, namely,
i)Labour Mix Variance
ii)Efficiency Variance
Further analysis brings out an y difference arising because of –
a)The actual mix of Labour employed being different from the
Standard “Mix Variance”
“Mix Variance”
B)Any difference on account of the Actual efficiency being different
from the Standard efficiency of 100% called “Effi ciency Variance”
(A) Mix Variance :
This variance, if any, can be established by comparing
e)The Actual mix, of the Actual Hours worked and
f)the Standard Mix of the Actual Hours worked.
Table E
Example : 3750 x 40/70 = 2143 etc.
From the above table you will observe that Col. C.
represents the Actual Hours worked by the labour employed
whereas col. C represents the mix of Hours as specified in the
Standard Mix giving rise Mix Variance in terms of Value.munotes.in

Page 98

98The Mix Variance is reflected by the two figures Col E -F
Actual Hrs. @ Actual Mix Rs. 1725
Actual Hrs. @ Standard Mix Rs. 1875
Mix Variance in Col. G Rs. 150 F
As Standard Mix is more than the Actual Mix, the Variance is
Favorable.
B) Efficiency Variance :
The work “Efficiency” Connotes the sp eed of working or
doing job.
If a work is done in a shortes time than the specified time we
say the efficiency is more than 100%, if not then the efficiency is
less than 100% and if completed exactuly within the
Specified/Standard time, then the efficienc yi s1 0 0 %
Using these criteria, we compare -
i)the standard time @ Standard Mix to complete the work and
ii)the Actual time W Standard Mix taken to do the work. The Table
below gives the comparative Hours @ Standard Mix.
Table F
Col. A gives the Actual Hours Q Standard Mix and Col. B
shows the Standard Hours allowed for the job also at Std. Mix.
Hence, by reducing the Actual Hours of 3750 hours @ Standard
Mix we have made the figures in Col. A and B comparable.
The Efficiency Variance is Reflec ted by -
i)The Actual Wages of Standard Mix @ Std. Rate Rs. 1875
ii)The Standard wages at Standard Mix @ Std. Rate Rs. 1400
The Efficiency Variance is unfavorable by Rs. 475 A
If the Actual workers employed had been as per Standard
mix, then we would have had to pay Rs. 1875 @ Std. rate.
When actually the standard Hrs. specified @ Std. Mix @ std.
rate the wages would have come to Rs. 1400.munotes.in

Page 99

99In other words, even if Std. rate and Std mix of workers had
been used, the Wages was more by Rs. 475 on acc ount of more
hours taken.
Check
Time usage variance = Mix. of Labour variance + Effiency of
Labour Variance
825 A = 150 F + 475 A
Summary
The above Variances can be tabulated as shown below; This
will simplify extracting Variances, specially in Exa m. Hall.
Note : After you segregate the wage rate variance the Labour
hours should be evaluated only with the standard Rate of Wages
for calculating the other Variances.
Illustration 7
The Following Standards have been set for doing a job.
A)The Team should consist of :
a)10 Men of “A” grade each one working for 40 hours @
Rs.0.80 per hour.
b)10 Men of “B” grade each one working for 40 hours @
Rs.0.60 per hourmunotes.in

Page 100

100c)5 Men of “c” grade each one working for 40 hours @ Rs.0.50
per hour.
d)The S tandard output Expected is 1000 units.
B)During a week in May, the Actual Team Composed of :
a)15 men of “A” grade each one worked for 40 hours @
Rs.1.00 per hour.
b)10 Men of “B” grade each one worked for 40 hours @
Rs.0.50 per hour.
c)2M e no f“ C ” grade each one worked for 40 hours @ Rs.0.35
per hour.
And the Actual Production was 96 units.
Work out the Labour Cost Variances.
Solution :
The Standard and the Actual Labour Cost is as follows.
Observe that the Standards are specified for 100 units since only
96 units are produced, we have to calculate the Variance from the
output of 96 units.
A)Total Labour Cost Variance :
Standard Cost of Producing the 96 units Rs.688.60
(See note below)
The Actual Cost the job Rs.828.00
Total Variance o n Labour Cost Rs.194.40 Adverse
Analysis of Total Labour Costmunotes.in

Page 101

101(A) First we should calculated Rate of Pay Variance
Table 1 –Wage Rate Variance
C)The Next Variance is the Time Usage Variance
From the output, we have been given that only 96 units w ere
produced and let us calculate the Standard Time that should have
been take to Produce the 96 units, is 960 Hours .
We should compare this time with the Actual time taken at
Standard rate. Hence Tabulating these we have
Standard and Actual time taken for 96 units
Table 2
From the two tables (1) and (2) we can reconcile the Total Labour
Cost Variance shown in (A).
Total Labour Cost Variance = Rate of Pay Variance + Time
Usage Variance
Rs.194.40 Adverse =Rs. 68.00 Adverse
Rs.126.40 Advers e
Rs.194.40
Further Analysis of Time Usage Variancemunotes.in

Page 102

102The time usage Variance is a composite variance of :
i)Mix of Labour Variance
ii)Efficiency of Labour Variance
The Mix variance is calculated by reference :
a)The Actual Mix of Labour used at St andard Rate.
b)The Standard Mix of Labour that should have been used at
Standard Rates.
Both the Mixes are to be considered with reference to the
Actual Hours worked
Mix Variance
Table 3
Efficiency Variance
The last of the Variance is to indicate th e time Efficiency
Variance –That is the difference between –
a)The time that should have been taken at Standard Rate for the
Actual Output achieved at Std. Mix.
b)And the Actual Time taken at Std. Mix calculated at the
Standard Rate.
We revise the Act ual Mix to Standard Mix of Actual Input tabulate
the result as shown.
Hence, whenever, A gang of working of difference categories are
employed and the Output is different from the Standard expected.
The Variance calculation will be as above.munotes.in

Page 103

103We can no w summaries all the Variance as follows.
TAB-1 Rate of pay Variance 60.00 Adverse
TAB –3 Mix of Labour Variance 47 Adverse
TAB –4 Efficiency of Labour Variance 79.20 Adverse
TAB –2 Time Usage Variance 196.40 126.40 Adverse
Total Labour Cost Va riance 194.40
Summary
We can tabulate the Variance Calculated above in the table
given below.
Check
Actual Cost of Production is more than the Std. allowed. Hence
Adverse.
Illustration 8
Example II
A contract work was estimated to be performed i naw e e ko f
40 hours by a labour force constituted as follows at the hourly rates
given below.
The Standard Mix of Labour grade :
40 Men at Rs.0.60 per hour
20 Women at Rs.0.30 per hour1070boys at Rs. 0.30 per hourmunotes.in

Page 104

104Standard Mix of Labour grade is as follows.
Men 40/70
Women 20/70
Boys 10/70
But however, as sufficient men were not available, the actual
composition of Labour force employed and their rates of pay the job
as under.
50 Men at Rs.0.60 per hour
20 Women at Rs.0.45 pe rh o u r
10 Women at Rs.0.30 per hour
15 Boys at Rs.0.32 per hour
And the work was completed in 50 hours and each of them was
paid for the 50 hours.
Calculate the loss or gain on the Labour Cost of the contract and
analyse the variance.
Answer
The first s tep is to be prepare a Standard Labour Cost of the job ….
See Table A.
The second step is to prepare the Actual Cost incurred …….
See Table B.
Table A
Standard Labour Cost Rs.
Table B
The Actual composition of the Labour Cost
Hence from (A) and (B) the total Labour Cost Variance is
Rs.840 Adverse i.e. Rs.1400 –Rs.1740 = Rs.340 it is adverse
because the Actual Cost is more than the Standard Cost.munotes.in

Page 105

105Now, let us analyse the Labour Cost variance under –
a)Wage Rate Variance
b)Time Usage Variance
Ther eafter analyse the Time Variance under –
c)Labour Mixture Variance
d)Labour Efficiency Variance
Wages Rate Variance Calculation
In the third step we determine the wage rate variance which
is the First Variance that should be calculated.
The wage r ate variance analyse is prepared by comparing
the data of (a) and (b) given below.
a)The Actual Wages paid i.e. The Actual time worked X the Actual
rate paid.
b)The wages that should have been paid for the Actual Hours at
the Standard rate specified.
The formula : The Actual time worked X Standard rate of Wage.
From table B. We have the data for Actual Wages @ Actual rates
paid.
Table B
2.We prepare table C to calculate the wages that should have
been paid for the Actual Hours if the Rate per Hour had been as
per the Standard.
Table C
munotes.in

Page 106

106Now comparing the wages in Table B and Table C we find that –
a)The Actual Hours are the same in both the cases.
b)But in Table B we have used the Actual rate and in Table C we
have used the Standard Rate.
There fore, we get the Variance arising due to wage difference only
and this known as Wage Rate Variance.
Calculation of Wage Rate Variance
The Variance is Adverse because the Actual Wages is more than
the Standard Wages. Usage of Time Variance.
We determin e this variance in the fourth step.
This is arrived at by comparing (c) and (d).
The standard time @ Standard Wage rate, and
(d) The Actual time @ Standard Wage rate.
The calculation already shown in Table A and C respectively are
reproduced here for e asy reference.
Table D
The difference between Rs.1400 (Standard Hours X Std. Rate and
Rs.1725) (Actual Hours X Standard Rate) = Rs.325 Adverse
represents the Variance due to usage of Time.
As the (Actual Hrs. X Std. Rate) is greater than the Standard
Wages (Standard Hours X Std. Rate) the variance is Adverse.munotes.in

Page 107

107CheckThe Total Labour Wage Rate Usage ofCost Variance Variance time            Rs.34C (A) =Rs. 15 (A) + Rs.325 (A)
Note
After you have carried out this primary variance analysis we
can proceed to analyse the usage of Time Variance into its further
components, namely,
i)Labour Mix Variance
ii)Efficiency Variance
Further analysis brings out any difference arising because of –
a)The actual mix of Labour employed being different from the
Standard “Mix Variance”
“Mix Vari ance”
B)Any difference on account of the Actual efficiency being different
from the Standard efficiency of 100% called “Efficiency Variance”
(A) Mix Variance :
This variance, if any, can be established by comparing
e)The Actual mix, of the Actual Hou rs worked and
f)the Standard Mix of the Actual Hours worked.
Table E
Example : 3750 x 40/70 = 2143 etc.
From the above table you will observe that Col. C.
represents the Actual Hours worked by the labour employed
whereas col. C represents the mix of Hours as specified in the
Standard Mix giving rise Mix Variance in terms of Value.munotes.in

Page 108

108The Mix Variance is reflected by the two figures Col E -F
Actual Hrs. @ Actual Mix Rs. 1725
Actual Hrs. @ Standard Mix Rs. 1875
Mix Variance in Col. G Rs. 150 F
As Standar d Mix is more than the Actual Mix, the Variance is
Favorable.
B) Efficiency Variance :
The work “Efficiency” Connotes the speed of working or
doing job.
If a work is done in a shortes time than the specified time we
say the efficiency is more than 100%, if not then the efficiency is
less than 100% and if completed exactuly within the
Specified/Standard time, then the efficiency is 100%
Using these criteria, we compare -
i)the standard time @ Standard Mix to complete the work and
ii)the Actual time W St andard Mix taken to do the work. The Table
below gives the comparative Hours @ Standard Mix.
Table F
Col. A gives the Actual Hours Q Standard Mix and Col. B
shows the Standard Hours allowed for the job also at Std. Mix.
Hence, by reducing the Actu al Hours of 3750 hours @ Standard
Mix we have made the figures in Col. A and B comparable.
The Efficiency Variance is Reflected by -
i)The Actual Wages of Standard Mix @ Std. Rate Rs. 1875
ii)The Standard wages at Standard Mix @ Std. Rate Rs. 1400
TheEfficiency Variance is unfavorable by Rs. 475 A
If the Actual workers employed had been as per Standard
mix, then we would have had to pay Rs. 1875 @ Std. rate.
When actually the standard Hrs. specified @ Std. Mix @ std.
rate the wages would have co me to Rs. 1400.munotes.in

Page 109

109In other words, even if Std. rate and Std mix of workers had
been used, the Wages was more by Rs. 475 on account of more
hours taken.
Check
Time usage variance = Mix. of Labour variance + Effiency of
Labour Variance
825 A = 150 F + 475 A
Summary
The above Variances can be tabulated as shown below; This
will simplify extracting Variances, specially in Exam. Hall.
Note : After you segregate the wage rate variance the Labour hours
should be evaluated only with the standard Rate of Wages for
calculating the other Variances.
5.2 EXERCISE
1.The standard cost of Mix is as under :
8T o n n e so fm a t e r i a lA@4 0p e rt o n e s 12
Tonnes of Material B @ 60 per tonnes
Standard yield is 90% of output . Actual cost for a period is as
under :
10tonnes of Material A @ 30 per tones
20 tonnes of Material B @ 68 per tonnes.munotes.in

Page 110

110Actual yield is 25.5 tonnes. calcu late A Material cost
variance B material usage variance C Material price variance D
material mix variance E material yield variance .
(Ans: a) Rs.129 A b) Rs.169 A c) Rs. 60 A d) Rs.40 A e) Rs.29 A)
2.The following figures are extracted from the books of company
Particulars Budget Actual
Output in units 6000 6500
Hours 3000 3300
Overhead cost -Fixed Rs1200 Rs1250
Overhead cost -Variable Rs6000 Rs6650
No. of Days 25 27
Compute and analysis the overhead variance
Ans.) Overhead cost variance Rs .100 A b) Variable OH Cost variance
Rs.150 A c) Fixed OH cost Variance Rs .50 F, FO Expenditure Rs .
50 A, FO Volume Rs .100 F, FO Efficiency Rs .4F, FO Calendar Rs .96
F, FO Capacity Rs .100 F.
5.The following information is given regarding standard
composition and standard rates of gang of workers
Standard Composition Standard hourly rates10 menRs 0.6255w o m e n Rs 0.400
5B o y s Rs 0.350
Accordingly to given specifications a week consists of 40 hours
and standard output for week is 1000 units . In a particular week gang
consisted of 13 men, 4 women and 3 boys and actual wages paid are
as follows :
Men @ Rs0.6 per hour, Women @ Rs0.425 per hour and boys @
Rs0.325 per hour. Two hours were lost in the week due to abnormal idle
time . Actual production was 960 units in the week . Find out A Labour
cost variance B labour rate variance C labour mix variance D labour
yield variance and E labour ef ficiency variance
Ans A Rs35 A , Rs 12 F , Rs 31 A , Rs4 F and Rs47 A
3. From the following information calculate
A–sales Variance B Sales price Variance C sales volume
variance D sales mix variance E sales quantity variancemunotes.in

Page 111

111
Product STD units STD Rate s
RsActual
UnitsActual
Rates Rs
A 5000 5 6000 6
B 4000 6 5000 5
C 3000 7 4000 8
Ans.AR s .5000 F B Rs .500 F C Rs .17500 F D Rs .2300 F
ER s .18000F
Illustration 1:
Alco Ltd. follows Standard Costing System. The standard material
costs fo r9 0u n i t sa r ea su n d e r :
Units Rate `
M1 60 4.00 240
M2 40 6.00 240
100 480
Less : Scrap 10 3.00 30
90 450
During February 2004, the company manufactured 5,400
units. The actual material cost was as under:
M1 4,000 units `20,000
M2 2,000 unit s`11,000
Scrap realized @ `2.50 p.u.
Calculate the variances in as much detail as possible.
(M.Com . Oct. 03, adapted)
(Ans.: MCV -Rs. 4,000 A, MPV -3,000 A, MMV -800 F, MYV -1,800
A, MUV -1,000 A)
Illustration 2 :
The standa rd material cost for 100 kg of Chemical D is made up of :
Chemical A = 30 kg @ `4 per kg
Chemical B = 40 kg @ `5 per kg
Chemical C = 80 kg @ `6 per kg
A batch of 500 kg of Chemical D was produced from a mix of :
Chemical A = 140 kg at a cost of `588
Chemical B = 220 kg at a cost of `1056
Chemical C = 440 kg at a cost of `2860
How do the yield, mix and the price factors contribute to the
variance in the actual cost per 100 kg of Chemical D over the
standard cost?
(M.Com .Oct. 03, adapted)
(Ans.: MCV -Rs. 4,000 A, MPV -3,000 A, MMV -800 F, MYV -1,800
A, MUV -1,000 A)munotes.in

Page 112

112Illustration 3 :
Standard Material for 100Kg. chemical A is give below :
Actual production is 2000 units of chemical A and actual material
usage is as fo llows :
Calculate :
a) Material Cost Variance b)Material Price Variance
c) Material Mix Variance d)Material Yield Variance
e) Material Usage Variance
(M.Com .M a r .9 6 ,a d a p t e d )
(Ans.: MCV -Rs. 790 A, MPV -590 A, MMV -528 F ,M Y V -728 A,
MUV -200 A)
Illustration 4 :
A manufacturing company uses the following standard mix of
their compound in one batch of 100 kgs of its production line:
50 kgs of material X at the standard price of Rs 2.
30 kgs of material Y at the standard price of Rs 3.
20 kgs of material Z at the standard price of Rs 4.
The actual mix for a batch of 120 kgs was as follows :
60 kgs of material X at the price of Rs 3.
40 kgs of material Y at the price of Rs 2.5.
10 kgs of material Z at the price of Rs 3.
Calculate the different material variances.
(M. Com .A p r i l0 9 ,a d a p t e d )
(Ans.: MCV -Rs. 40 A, MPV -30 A, MMV -17 F, MYV -27 A, MUV -
10 A)munotes.in

Page 113

113Illustration 5:
A gang of workers usually consists of 10 skilled, 5 semi -
skilled and 5 unskilled labour in a factory. They are paid at standard
hourly rates of ` 5.00, ` 3.20 and ` 2.80 respectively. In a normal
working week of 40 hours, the gang is expected to produce 1,000
units of output. In a certain week, the gang consisted of 13 ski lled,
4 semi -skilled and 3 unskilled labour. Actual wages were paid at the
rates of 4.80; 3.40 and 2.60 respectively. Two hours were lost due
to abnormal idle time and 960 units of output were produced.
You are required to calculate :
i) Labour cost varia nce ii) Labour rate variance iii) Labour idle time
variance iv) Labour efficiency variance v) Labour mix variance
(M.Com .M a r .0 6 ,a d a p t e d )
(Ans.: LCV -Rs. 280 A, LRV -96 F, LEV -32 F, MITV -160 F, LMV -
248 A)
Illustration 6:
From the following records of the Navi Mumbai
Manufacturing Company you are required to compute material and
labour variance.
(Oct. 96, adapted)
(Ans.: MCV -Rs. 30,000 A, MPV -10,000 A, MUV -20,000 A, LCV -
Rs. 9,000 A, LRV -5,000 A, LEV -4,000 A)
Illustration 7:
The following details relating to a product are made available to
you:munotes.in

Page 114

114Standard Cost per Unit :
Material 50 kg @ Rs 40 per kg
Labour 400 hours @ Rs 1 per hour
Actual Cost :
Material 4,900 kg @ Rs 42 per kg
Labour 39,600 hours @ Rs 1 per h our
Actual Production 100 units
Your are required to calculate :i)Material Cost Varianceii)Material Price Varianceiii)Material Usage Variance iv) Labour Cost Variance
v)Labour Rate Variance vi) Labour Efficiency Variance
(M.Com . Oct. 2000, adapted)
(Ans.: MCV -Rs. 5,800 A, MPV -9,800 A, MUV -4,000 F, LCV -Rs.
3,500 A, LRV -3,960 A, LEV -400 F)
Illustration 8:
The following standards have been set to manufacture a product :
Direct Materials :
2u n i t so fAa tR s4 . 0 0 per unit 8.00
3u n i t so fBa tR s3 . 0 0p e ru n i t 9.0015 units of C at Re. 1.00 per unit15.0032.00Direct labour 3 hrs. @ Rs 8 per hour24.00Total Standard Price Cost 56.00
The company Mangesh Ltd. manufactured and sold 6,000
units of the product during the year.
Direct material costs were as follows :
12,500 units of A at Rs 4.40 per unit
18,000 units of B t Rs 2.80 per unit
88,500 units of C at Rs 1.20 per unit
The company worked 17,500 direct labour hours during the
year. For 2.500 of thes e hours, the company paid Rs 12 per hour
while for the remaining the wages were paid at the standard rate.
Calculate materials price ans usage variances and also labour rate
and efficiency variances.
(M.Com . Oct. 01, adapted)
(Ans.: MCV -Rs. 19,600 A, MPV -19,100 A, MUV -500 A, MYV -
1,600 F, MMV -2,100 A, LCV -Rs. 6,000 A, LRV -10,000 A, LEV -
4000 F)munotes.in

Page 115

115Illustration 9:
The following standards have been set to manufacture a product :
Direct Materials :
4u n i t so fAa tR s4 . 0 0p e ru n i t 16.00
6u n i t so fBa tR s3 . 0 0p e ru n i t 18.0030 units of C at Re. 1.00 per unit30.0064.00Direct labour 6 hrs. @ Rs 8 per hour48.00Total Standard Price Cost 112.00
The company Mangesh Ltd. manufactured and sold 6,000
units of the product during the year.
Direct material costs were as follows :
25,000 units of A at Rs 4.40 per unit
36,000 units of B t Rs 2.80 per unit
1,77,000 units of C at Rs 1.20 per unit
The company worked 35,000 direct labour hours during the
year. For 5,000 of these hours, the company paid Rs 12 per hour
while for the remaining the wages were paid at the standard rate.
Calculate materials price ans usage variances and also labour rate
and efficiency variances.
(M.Com .M a r .2 0 0 3 ,a d a p t e d )
(Ans.: MPV -38,200 A, MUV -1,000 A, LRV -20,000 A, LEV -8,000
F)
Illustration 10:
Ketan Chemical Co. gives you the following standard and
actual data of Chemical No. 1456.
munotes.in

Page 116

116You are required to calculate :
a) Material Cost Variance b) Material Price Vari ance
c) Material Yield Variance d) Labour Cost Variance
e) Labour Rate Variance f) Labour Gang Variance
(M.Com .M a r . 0 2 ,a d a p t e d )
(Ans.: MCV -Rs. 790 F, MPV -90 F, MYV -700 F, LCV -Rs. 600 F,
LRV-3,000 A, LGV -NIL)
Illustratio n1 1 :
(M.Com .M a r2 0 0 1 ,a d a p t e d )
(Ans.: MCV -Rs. 750 A, MPV -900 F, MUV -1,650 A, MYV -1,821 A,
MMV -171 F, LCV -Rs. 184 A, LRV -84 A, LEV -100 A, LYV -35 A,
LMV -65 A)
Illustration 12:
The following information has been obta ined from the
records of a manufacturing organization using the Standard Costing
System for the month of March, 2006.
You are required to calculate the following Overhead Variances:
i)Variable Overhead Variance
ii)Fixed Overhead Variance
a)Expendit ure c)Efficiency Variance
b)Volume Variance d)Calendar Variance
iii)Also prepare Reconciliation Statement for the same.
(M.Com . Oct. 06, adapted)munotes.in

Page 117

117(Ans. -VOHCV -600 A, FOHCV -1,000 A, FOHVV -2,000 A,
FOHExp.V -1,000 F, FOHEf fi.V-4,000 A, FOHCV -2,000 F)
Illustration 13:
The following figures are extracted from the books of a
company:
You are required to calculate the following overheads variances :
i)Overhead Cost Variance
ii)Variable Overhead Cost
iii)Fixed Overhea d Cost Variance
iv)Fixed Overhead Volume Variance
v)Fixed Overhead Efficiency Variance
vi)Fixed Overhead Capacity Variance.
(M.Com .M a r .2 0 0 0 ,a d a p t e d )
(Ans. -VOHCV -600 A, FOHCV -50 F, FOHVV -2,000 A, FOHExp.V -
1,000 F, FOHE ffi.V-4,000 A, FOHCV -2,000 F)
Illustration 14:
Budgeted income from sales on 500 tonnes was 5,00,000
per month of M/s ABC and Co. in April, 2004 actual sales were 550
tonnes with a sale value of 495,000.
You are required to calculate sales Variances
(M.Com .M a r .0 5 ,a d a p t e d )
(Ans.: S Value V -5,000 F, SPV -55,000 F, S Volume V -50,000 A)
Illustration 15:
The cost accountant of Lagoon Ltd. found to this surprise
that the actual profit for the period ending 31st December, 2004
was the same as budget despite realizing 10% more than the
budgeted selling prices. The following facts and figures are
available :munotes.in

Page 118

118
You are required to assist the Cost Accountant in preparing
the necessary explanations as to why the profit remained th e same
despite an increase in sales.
(M.Com . Oct. 2005, adapted)
(Ans.: S Value V -32,50,000 A, SPV -7,50,000 A, S Volume V -
25,00,000 A)
Illustration 16:
A Company’s Budgeted Sales of product A are 40,000 units
at standard se lling price of Rs 10 per unit and product B 60,000
units at standard selling price of Rs 12 per unit.
Actual Sales of Product A are 70,000 units at 14 per unit and
Product B 50,000 units at 8 per unit.
You are required to calculate :
a)Sales Value Varia nce d)Sales Mix Variance
b)Sales Price Variance e)Sales Quantity Variance
c)Sales Volume Variance
(M.Com .M a r .0 7 ,a d a p t e d )
(Ans.: S Value V -2,60,000 A, SPV -80,000 A, S Volume V -
1,80,000 A, SQV -2,24,000 A, SMV -44,000 F )


munotes.in

Page 119

119MODULE -III
Unit-6
BUDGETARY CONTROL -I
Unit structure :
6.0 Objectives
6.1 Introduction
6.2 Budgets
6.3 Difference between Forecast and Budget
6.4 Budgetary control
6.5 Zero base Budget
6.6 Performance Budgeting
6.7 Functional Budgets
6.8 Capital Expenditure Budget
6.9 Exercises
6.0 OBJECTIVES
After studying this topic you will be able:
To understand the basic concepts of budget and budgetary
control
To understand various types of budgets
To understand the preparation of various types of budgets
To understand the benefits of budgetary control
To understand the limitations of budgetary control
6.1 INTRODUCTION
Every organization has well defined objectives, which are to
be achieved in a particular span of time. It is of paramount
importance that there should be systematic efforts to bring them
into reality. As a part of these efforts, it is necessary to formulate a
policy and it is reflected in the manpower planning budget as well
as other relevant budgets. Thus the policy to be purs ued in future
for the purpose of achieving well -defined objectives is reflected in
the budget.
6.2BUDGETS
Budget has been defined by CIMA U. K. as, ‘A financial and
or quantitative statement prepared prior to a defined period of time,munotes.in

Page 120

120of the policy to be pursued during that period for the purpose of
achieving a given objective.’
A budget is a statement that is always prepared prior to a
defined period of time. This means that budget is always prepared
for future period and not for the past. For example ,ab u d g e tf o rt h e
year 2011 -12 regarding the sales will be prepared in the year 2011 -
12. another important point is that the time for which it is prepared
is certain. Thus a budget may be prepared for the next 3 years / 1
year / 6 months/ 1 month or even for a week, but the point is that
the time frame for which it is prepared is certain. It cannot be
prepared for indefinite period of time.
Budget is prepared either in quantitative details or monetary
details or both. This means that budget will show the planning in
terms of rupees or in quantity or both. For example, a production
budget will show the production target in number of units and when
the target units are multiplied by the anticipated production cost, it
will be a production cost budget that i s expressed interms of
money. Similarly purchase budget is prepared in quantity to show
the anticipated purchase in the next year and when the quantity is
multiplied by the expected price per unit, it will become purchase
cost budget. Some budgets are prep ared only in monetary terms,
for example, cash budget, capital expenditure budget etc.
6.3 DIFFERENCE BETWEEN FORECAST AND
BUDGET
Forecast Budget
1. Forecast is merely an
estimate of what is likely to
happen. It is a statement of
probable events which a re likely
to happen under anticipated
conditions during a specified
period of time.1. Budget shows the policy and
programme to be followed in a
period under planned
conditions.
2. Forecasts, being statements
of future events, do not connote
any sense of control.2. A budget is a tool of control
since it represents actions
which can be shaped according
to will so that it can be suited to
the conditions which may or
may not happen.
3. Forecasting is a preliminary
step for budgeting. It ends with
the for ecast of likely events.3. It begins when forecasting
ends. Forecasts are converted
into bedget.munotes.in

Page 121

1214. Forecasts are wider in scope
and it can be made in those
spheres, also where budgets
cannot interfere.4. Budgets have limited scope.
It can be made of phenomenon
capable of being expressed
quantitatively.
6.4BUDGETARY CONTROL
Budgetary control is actually a means of control in which the
actual results are compared with the budgeted results so that
appropriate action may be taken with regard to any d eviations
between the two. Budgetary control has the following stages.
A.Developing Budgets:
The first stage in budgetary control is developing various
budgets. It will be necessary to identify the budget centers in the
organization and budgets will ha ve to develop for each one of them.
Thus budgets are developed for functions like purchase, sale,
production, manpower planning as well as for cash, capital
expenditure, machine hours, labor hours and so on. Utmost care
should be taken while developing the budgets. The factors affecting
the planning should be studied carefully and budgets should be
developed after a thorough study of the same.
B.Recording Actual Performance:
There should be a proper system of recording the actual
performance achieved. T his will facilitate the comparison between
the budget and the actual. An efficient accounting and cost
accounting system will help to record the actual performance
effectively.
C.Comparison of Budgeted and Actual Performance:
One of the most important aspects of budgetary control is
the comparison between the budgeted and the actual performance.
The objective of such comparison is to find out the deviation
between the two and provide the base for taking corrective action.
D.Corrective Action:
Taking appropriate corrective action on the basis of the
comparison between the budgeted and actual results is the
essence of budgeting. A budget is always prepared for future and
hence there may be a variation between the budgeted results and
actual results. Th ere is a need for investigation of the same and
take appropriate action so that the deviations will not repeat in the
future. Responsibilities can be fixed on proper persons so that they
can be held responsible for any such deviations.munotes.in

Page 122

1226.5ZERO BASE BUDG ET
Zero Base Budgeting is method of budgeting whereby all
activities are revaluated each time budget is formulated and every
item of expenditure in the budget is fully justified. Thus the Zero
Base Budgeting involves from scratch or zero.
Zero Base B udgeting actually emerged in the late 1960s as
an attempt to overcome the limitations of incremental budgeting.
This approach requires that all activities are justified and prioritized
before decisions are taken relating to the amount of resources
allocate d to each activity. In incremental budgeting or traditional
budgeting, previous year’s figures are taken as base and based on
the same the budgeted figures for the next year are worked out.
Thus the previous year is taken as the base for preparation of the
budget. However the main limitation of this system of budgeting is
that as activity is continued in the future only because it is being
continues in the past. Hence in Zero Base Budgeting, the beginning
is made from scratch and each activity and function is reviewed
thoroughly before sanctioning the same and all expenditures are
analyzed and sanctioned only if they are justified.
Besides adopting a ‘Zero Base’ approach, the Zero Base
Budgeting also focuses on programs or activities instead of
functional departments based on line items, which is a feature of
traditional budgeting. It is an extension of program budgeting. In
program budgeting, programs are identified and goals are
developed for the organization for the particular program. By
inserting deci sion packages in the system and ranking the
packages, the analysis is strengthened and priorities are
determined.
Applications of Zero Base Budgeting:
The following stages/ steps are involved in the application of
Zero Base Budgeting.
1.Each separate activity of the organization is identified and is
called as a decision package. Decision package is actually
nothing but a document that identifies and describes an activity
in such a manner that it can be evaluated by the management
and rank against other activities competing for limited resources
and decide whether to sanction the same or not.
2.It should be ensured that each decision package is justified in
the sense it should be ascertained whether the package is
consisted with the goal of the organiza tion or not.munotes.in

Page 123

1233.If the package is consisted with the overall objectives of the
organization, the cost of minimum efforts required to sustain the
decision should be determined.
4.Alternatives for each decision package are considered in order
to select bett er and cheaper options.
5.Based on the cost and benefit analysis a particular decision
package should be selected and resources are allocated to the
selected package.
ZBB was first introduced by Peter A. Pyhrr, a staff control
manager at Texas Instrume nts Corporation, U.S.A. He developed
this technique and implemented it for the first time during the year
1969 -70 in Texas in the private sector and popularized its wider
use. He wrote an article on ZBB in Harvard Business Review and
later wrote a book on the same. The ZBB concept was first applied
in the State of Georgia, U.S.A. when Mr. Jimmy Carter was the
Governor of the State. Later after becoming the President of U.S.A.
Mr. Jimmy Carter introduced and implemented the ZBB in the
country in the year 198 7. ZBB has a wide application in the
Government Departments but also in the private sector in a variety
of business. In India, the ZBB was applied in the State of
Maharashtra in 80s and early 90s. Benefits from ZBB can be
summarized as follows.
i.ZBB fac ilitates review of various activities right from the scratch
and a detailed cost benefit study is conducted for each activity.
Thus an activity is continued only if the cost benefit study is
favorable. This ensures that an activity will not be continued
merely because it was conducted in the previous year.
ii.A detailed cost benefit analysis result in efficient allocation of
resources and consequently wastages and obsolescence is
eliminated.
iii.A lot of brainstorming is required for evaluating cost and
benefits arising from an activity and this results into generation
of new ideas and also a sense o involvement of the staff.
iv.ZBB facilitates improvement in communication and co ordination
amongst the staff.
v.Awareness amongst the managers about the in put costs is
created which helps the organization to become cost conscious.
vi.An exhaustive documentation is necessary for the
implementation of this system and it automatically leads to
record building.
The following are the limitations of Zero Base Bu dgeting.munotes.in

Page 124

124i.It is very detailed procedure and naturally is time consuming
and lot of paper work is involved in the same.
ii.Cost involved in preparation and implementation of this system
is very high.
iii.Morale of staff may be very low as they might fee lt h r e a t e n e di f
a particular activity is discontinued.
iv.Ranking of activities and decision -making may become
subjective at times.
v.It may not advisable to apply this method when there are non
financial considerations, such as ethical and social respon sibility
because this dictate rejecting a budget claim on low ranking
projects.
6.6PERFORMANCE BUDGETING
It is budgetary system where the input costs are the
performance i.e. the end results. This budgeting is used extensively
in the Government and Publ ic Sector Undertakings. It is essentially
a projection of the Government activities and expenditure thereon
for the budget period. This budgeting starts with the broad
classification of expenditure according to functions such as
education, health, irrigati on, Social welfare etc. Each of functions is
then classified into programs into activities or projects. The main
features of performance budgeting are as follows.
i.Classification into functions, programs or activities
ii.Specification of objectives for each p rogram
iii.Establishing suitable methods for measurement of work as far
as possible
iv.Fixation of work targets for each program.
Objectives of each program are ascertained clearly and then
the resources are applied after specifying them clearly. The results
expected from such activities are also laid down. Annual, quarterly
and monthly targets are determined for the entire organization.
These targets are broken down for each activity center. The next
step is to set up various productivity or performance ratios a nd
finally target for each program activity is fixed. The targets are
compared with the actual results achieved. Thus the procedure for
the performance budgets include allocation of resources execution
of the budget and periodic reporting at regular interv als.
The budgets are finally compiled by the various agencies
such as Government Department, public undertakings etc.
thereafter these budgets move on to the authorities responsible for
reviewing the performance budgets. Once the higher authoritiesmunotes.in

Page 125

125decide about the funds, the amount sanctioned are communicated
and the work is started. It is the duty of these agencies to start the
work in time, to ensure the regular flow of expenditure, against the
physical targets, prevent over runs under spending and furn ish
report to the higher authorities regarding the physical progress
achieved.
Inthe final phase of performance budgetary process,
progress reports are to be submitted periodically to higher
authorities to indicate broadly, the physical performance to be
achieved, the expenditure incurred and the variances together with
explanations for the variances.
Check Your Progress:
1)Define the terms.
a)Budget
b)Budgetary Control
c)Zero Base Budget
d)Performance Budgeting
2)“Budgetary control is actually a means of Control.” Discuss.
3)Distinguish between Forecast and Budget.
6.5 FUNCTIONAL BUDGETS
The Functional Budgets are prepared for each function of
the organization. These budgets are normally prepared for a period
of one year and then broken dow nt oe a c hm o n t h .T h ef o l l o w i n g
budgets are included in this category.
i.Sales Budget: A Sales budget shows forecast of expected sales
in the future period and expressed in quantity of the product to
be sold as well as the monetary value of the same. A Sa les
Budget may be prepared product wise, territories/ area/ country
wise, customer group wise, salesmen wise as well as time like
quarter wise, month wise, weekly etc.
ii.Production Budget: This budget shows the production target
to be achieved in the yea ro rt h ef u t u r ep e r i o d .T h ep r o d u c t i o n
budget is prepared in quantity as well as in monetary terms.
Before preparation of this budget it is necessary to study the
principal budget or the key factor. The principal budget factor
can be sales demand or the pr oduction capacity or availability of
raw material. The policy of the management regarding the
inventory is also taken into consideration. The production
budget is normally prepared for a period of one year and broken
down on monthly basis. Production targe ts are decided by
adding the budgeted closing inventory in the sales forecast andmunotes.in

Page 126

126subtracting the opening inventory from the total of the same.
Production Cost Budget is prepared by multiplying the
production targets by the budgeted production cost per uni t.
iii.Material Purchase Budget: This budget of materials tot be
purchased during the coming year. For the preparation of this
budget, production budget is the starting point if it is the key
factor. If the raw material availability is the key factor, it
becomes the starting point. The desired closing inventory of the
raw materials is added to the requirement as per the production
budget and the opening inventory is subtracted from the gross
requirements. This budget is prepared in quantity as well as the
monetary terms and helps immensely in planning of the
purchase of raw materials. Availability of storage space,
financial resources, various levels of materials like maximum,
minimum, re -order and economic order quantity are taken into
consideration while preparing this budget. A separate material
utilization budget may also be prepared as a preparation of
material purchase budget.
iv)Cash Budget: a cash budget is an estimate of cash receipts
and cash payments prepared for each month. In this budget all
expected payments, revenue as well as capital and all receipts,
revenue and capital are taken into consideration. The main
purpose of cash budget is to predict the receipts and payments
in cash so that the firm will be able to find out the cash balance
at th e end of the budget period. This will help the firm to know
whether there will be surplus or deficit at the end of budget
period. It will help them to plan for either investing the surplus or
raise necessary amount to finance deficit. Cash budget is
prepar ed in various ways, but the most popular form of the
same is by method of Receipt and Payment method.
v.Master Budget: All the budgets described above are called as
‘Functional Budgets’ that are prepared for the planning of
individual function of the orga nization. For example, Budgets
are prepared for Purchase, Sales, Production, Manpower
Planning, and so on. A master budget which is also called as
‘Compressive Budget’ is a consolidation of all the functional
budgets. It shows the projected Profit and Loss account and
Balance sheet of business organization. For preparation of this
budget, all functional budget are combined together and the
relevant figures are incorporated in preparation of the projected
Profit and Loss Account and Balance Sheet. Thus Maste r
Budget is prepared for the organization and not for individual
functions.munotes.in

Page 127

1276.6 CAPITAL EXPENDITURE BUDGET
6.6.1 Fixed and Flexible Budgets:
The fixed and flexible budgets are discussed in detail in the
following paragraphs.
i.Fixed Budget: When a bu dget is prepared by assuming a fixed
percentage of capacity utilization, it is called as a fixed budget.
For example, a firm may decide to operate at 90% of its total
capacity and prepare a budget showing the projected profit or
loss at that capacity. This budget is defined by The Institute of
Cost and Management Accountants of [U.K.] as ‘the budget
which is designed to remain unchanged irrespective of the level
of activity actually attained. It is based on a single level of
activity’. For preparation of th is budget, sales forecast will have
to be prepared along with the cost estimate. Cost estimate can
be prepared by segregating the costs according to their
behavior i.e. fixed and variable. Cost predictions should be
made element wise and the projected prof it or loss can be
worked out by deducing the cost from the sales revenue.
Actually in practice, fixed budgets are prepared very rarely. The
main reason is that the actual output differs from the
budgeted output significantly. Thus if the budget is prepared
on the assumption of producing 50, 000 units and actually the
number of units produced are 40, 000, the comparison of actual
results with the budgeted ones will be unfair and misleading.
The budget may reveal the difference between the budgeted
costs and actual costs but the reason for the deviations may
not be pointed out. A fixed budget may be prepared when the
budgeted output and actual output are quite close and not much
deviation exists between the two. In such cases, maximum
control can be exercised between the budgeted performance and
actual performance.
ii.Flexible Budgets: aF l e x i b l eb u d g e ti sab u d g e tt h a ti s
prepared for different levels of capacity utilization. It can be
called as a series of fixed budgets prepared for different levels
of activity. For example, a budget can be prepared for capacity
utilization levels of 50%, 60%, 70%, 80%, 90% and 100%. The
basic principle of flexible budget is that if budget is prepared
for showing the results at say, 15, 000 units and actual
production is only 12, 000 units, the comparison between the
expenditures, budgeted and actual will not be fair as the
budget was prepared for 15, 000 units. Therefore it is
developed for a relevant range of production from 12, 000
units to 15, 000 units. Thus even if the actua l production is 12,
000 units, the results will be comparable with the budgeted
performance of 12, 000 units. Even if the production slips tomunotes.in

Page 128

1288,000 units, the manager has a tool that can be used to
determine budgeted cost at 8,000 units of output. The flex ible
budget thus, provides a reliable basis for comparison because
it is automatically geared to change in production activity.
Thus a flexible budget covers a range of activity, it is flexible
i.e. easy with variation in production levels and it facilitat es
performance measurement and evaluation.
iii.While preparing flexible budget, it is necessary to study the
behavior of cost and divide them in fixed, variable and semi
variable. After doing this, the costs can be estimated for a
given level of activity.
iv.It is also necessary to plan the range of activity. A firm may
decide to develop flexible budget for activity level starting to
plan the range of activity level from 50% to 100% with an
interval of 10% in between. It is necessary to estimate the
costs and asso ciate them with chosen level of activity.
v.Finally the profit or loss at different levels of activity will be
computed by comparing the costs with the revenues.
6.6.2 Preparation of Budget:
A budgetary control is extremely useful for planning and
contro lling as described above. However, for getting these benefits,
sufficient preparation should be made. For complete success, a
solid foundation should be laid down and in view of this the
following aspects are of crucial importance.
i. Budget Committee: fors u c c e s s f u li m p l e m e n t a t i o no f
budgetary control system, there is a need of a budget
committee. In small or medium size organizations, there may
not be carried out by the Chief Account himself. Due to the
size of organization, there may not be too many pro blems in
implementation of the budgetary control system. However, in
large size organization, there is a need of a budget committee
consisting of the chief executive, budget officer and heads of
main departments in the organization. The functions of the
budget committee are to get the budgets prepared and then
scrutinize the same, to lay down broad policies regarding the
preparation of budgets, to approve the budgets, suggest for
revision, to monitor the implementation and to recommended
the action to be ta ken in a given situation.
ii.Budget Centers: Establishment of budget centers is another
important pre -requisite of a sound budgetary control system. A
budget canter is a group of activities or a section of the
organization for which budget can be developed. F or example,
manpower planning budget, research and development costmunotes.in

Page 129

129budget, production and production cost budget, labor hour and
so on. Budget centers should be defined clearly so that
preparation becomes easy.
iii.Budget Period: Ab u d g e ti sa l w a y sp r e p a red prior to a defined
period of time. This means that the period for which a budget
is prepared is decided in advance. Thus a budget may be
prepared for three years, one year, six months, one month or
even for a week. The point is that the period for the functional
budgets like sales, purchase, production etc. are prepared for
one year and then broken down on monthly basis. Budgets
like capital expenditure are generally prepared for a period
from 1 year to 3 years. Thus depending upon the type of
budget, t he period of the same is decided and it is important
that it is decided well in advance.
iv.Preparation of an Organization chart: There should be an
organization chart that shows clearly defined authorities and
responsibilities of various executives. The organization chart
will define clearly the functions to be performed by each
executive relating to the budget preparation and his
relationship with other executives. The organization chart may
have to be ensure that each budget center is controlled by an
appropriate member of the staff.
v.Budget Manual :A budget manual is defined by ICMA as’ a
document whish sets out the responsibilities of the person
engaged in, the routine of and the forms and records required
for budgetary control’. The budget manual th us is a schedule,
document or booklet, which contains different forms to be used,
procedures to be followed, budgeting organization details, and
set of instructions to be followed in the budgeting system. It also
lists out detail of the responsibilities of different persons and the
managers involved in the process. A typical budget manual
contains the following.
1.Objectives and of authorities and managerial policies of the
business concern.
2.Internal lines of authorities and responsibilities.
3.Functions of the role of budget committee officer.
4.Budget period
5.Principal budget factor
6.Detailed program of budget preparation
7.Accounting codes and numbering
8.Follow up procedures.
vi.Principal Budget Factor: A principal budget factor is that factor
the extent of whose i nfluence must first be assessed in order to
prepare the functional budgets. Normally sales is the key factor
or principal budget factor but other factors like production,munotes.in

Page 130

130purchase, skilled labor may also be the key factors. The key
factor puts restrictions on the other functions and hence it must
be considered carefully in advance. So continuous assessment
of the business situation becomes necessary. In all conditions
the key factor is the starting point in the process of preparation
of budgets. A typical l ist of some of the key factor is given
below:
Sales :Consumer demand, shortage of sales staff, inadequate
advertising
Material :Availability of supply, restrictions on import
Labor :Shortage of labor
Plant :Availability of capacity, bottlenecks in key processes
Management: Lack of capital, pricing policy, shortage of efficient
executive, lack of faulty design of the product etc.
vii. Accounting Records: It is essential that the accounting
system should be able to record and analyze the transactio n
involved. A chart of accounts or accounts code should be
maintained which may correspond with the budget centers for
establishment of budgets and finally, control through budgets.
Check Your Progress:
1)Define the terms.
a)Functional Budget.
b)Prod uction Budget.
c)Cash Budget.
d)Master Budget.
e)Budget Committee
f)Budget Creators.
g) Budget Manual
2)Distinguish between Fixed Budget and Flexible Budget.
6.7 EXERCISES
1.Select the correct answer from the choices given in each of
followin g:-
1)A budget is A] an aid to management B] a postmortem
analysis C] a substitute of management.
2)The budgeted standard hours of factory is 12,000. The
capacity utilization ratios for April 2009 stood at 90% while
the efficiency ratios for the month came to 120%. The actualmunotes.in

Page 131

131production in standard hour for April 2009 was A] 10,800 B]
12,960 C] 14,400 D] 12,800
3)A budget is a projected plan of action in A] physical units B]
monetary terms C] physical units and monetary units.
4)Flexible budget are useful for A] Planning purposes only B]
Planning performance evaluation and feedback control C]
Control of performance only
5)The scarce factor of production is known as , A] Key factor
B] Linking factor C] Critical factor D] Production factor.
2.State whether the following statements are TRUE or FALSE.
1)Fixed budgets are concerned with acquisition of fixed assets.
2)Functional Budgets are subsidiary to master budget.
3)Budgeting is useful for planning and controlling.
4)Capital expenditures bud get is prepared generally for short
term.
5)Budgetary control is a technique of costing.

munotes.in

Page 132

132Unit-7
BUDGETARY CONTROL -II
Unit structure :
7.0 Objectives
7.1 Solved Problems
7.2 Exercise
7.0 OBJECTIVES
After studying the unit the students will be able to solve the
problems on budgetary control.
7.1 SOLVED PROBLEMS
Illustration 1
1.ZL t d . ,h a sp r e p a r e dt h ef o l l o w i n gs a l e sB u d g e tf o rf i r s tf i v e
months of 2011.
Inventory finished goods at the end of every month is to be
equal to 25 % of sales estimate for the next month. On 1st January
2011, there were 2,700 units of product on hand. There is no work
in progress at the end of any month.
Every unit product requires two types of materials in the
following quantities;
Material A: 4 Kg.
Material B: 5 Kg.
Materials equal to one half of the requirements of the next
month’s product ion are to be in hand at the end of every month.
This requirement was met on 1st January 2011.munotes.in

Page 133

133Prepare the following budgets for the quarter ending on 31st
marsh 2011
i)Production Budget -Quantity Wise.
ii)Materials Purchase Budget -Quantity wise.
Solu tion :
ZL t d .
Production Budget [In units] January -March, 2011
Materials Requirement Budget [Quantitative]
Material A -January -March, 2011
Materials Requirement Budget [Quantitative]
Material B -January -March, 2011
Working Notes:
1)Production for April. Sales 10,400 [ units] + Closing Stock 2,450
[units] = 12,850 [units] –Opening Sock 2,600 [units] = 10,250
[units].munotes.in

Page 134

1342)Material required for production in April: A :10,250 X 4 = 41,000
kg. B :10,250 X 5 = 51,250 kg
Illustration 2
ALtd. manufactures a single product P with a single grade of
labor. The sales budget and finished goods stock budget for the 1st
Quarter ending on 30thJune 2011 are as follows:
Sales: 1400 units
Opening finished units: 100 units
Closing finished units: 140 units
The goods are imported only when the production work is
complete and it is budgeted that 10% of finished work will be
scrapped.
The standard direct labor content of the product P is 3 hours.
The budgeted productivity ratio for direct is 80% only.
The company employs 36 direct operatives who are
expected to average 144 working hour each in the 1stquarter.
You are required to prepare,
i)Production Budget
ii)Direct Labor Budget
iii)Comment on the problem that your direct labor budget
reveals and suggest how this problem might be overcome.
Solution:
AL t d .
Production Budget
April –June 2011
munotes.in

Page 135

135Direct Labour Budget
Comments :From the Direct Labor Budget it can be seen that the
direct labor hours available are not sufficient and hence the re is
shortage of 816 Hours. Therefore it will be necessary to work
overtime, as well as improvement in the efficiency.
Illustration 3
Summarized below are the Income and Expenditure forecast
for the month March to August 2011.
You are given following further information
i.Plant Costing Rs. 16,000 due for delivery in June. 10% on
delivery and balance after three months .
ii.Advance Tax Rs. 8,000 is payable in March and June
iii.Period of credit allowed, Suppliers 2 months and Customers 1
month
iv.Lag in payment of manufacturing expenses half month
v.Lag in payment of all others expenses one month
vi.Cash balance on 1st May 2008 is Rs. 8,000
vii.Prepare Cash Budget for three months starting from 1st May
2010munotes.in

Page 136

136Solution :
Cash Budget
May -August 2010
There is delay of h alf a month for payment of Manufacturing
Expenses and wages and hence current month’s 50% and previous
month’s 50% are paid in the current month.
Illustration 4
Make out a cash flow chart for the budget period April -June,
1999 from the following informati on:
1) Actual and budgeted sales :
munotes.in

Page 137

1372) Actual and budgeted Purchases :
3) Actual and budgeted wages and expenses :
4)Special Payments
Advance Income Tax payable in May Rs. 4000
For Purchase of Plant in April Rs. 10,000
5)Rent Rs. 300/ -payable e ach month, not included in expenses.
6)10% of purchases and sales are on cash terms
7)Credit purchases are paid after 1 month and credit sales are
collected after 2 months. Time lag in wages and expenses ½
month.
8)Cash and Bank Balance on April 1, 197 1 Rs. 18,000.
ANSWERS
munotes.in

Page 138

138
Illustration 5:
A manufacturing company is currently working at 50%
capacity and produces 10,000 units at a cost of Rs. 180 per unit as
per the following details.
Materials: Rs.100
Labo ur: Rs.30
Factory Overheads: Rs.30 [4 0 %f i x e d]
Administrative Overheads: Rs.20 [50% fixed]
Total Cost Per Unit: Rs.180
The selling price per unit at present is Rs.200. At 60%
working, material cost per unit increases by 2% and selling pricemunotes.in

Page 139

139per unit falls by 2%. At 80% working, material cost per unit
increases by 5% and selling price per unit falls by 5%.
Prepare a Flexible Budget to show the profits/ losses at
50%, 60% and 80% capacity utilization.
Solution:
Flexible Budget
Illustration 6:
A company has three products and two sales divisions. The
annual sales and budgeted sales of the products for the year
ending 31 -12-97 are given below –
1)During the next year product A is expected to register an
increase in general demand of 15 percent and product B of 10%.
However prod uct C is expected to register a decrease in demand of
5%.munotes.in

Page 140

1402)By reason of the management policy to increase its price at an
overall 20% for all products, Division –I and Division –II and each
expected to show a decline in sales of 10%.
On the basis o f the above information prepare a sales
budget for the year ending 31st December 1998.
Solution :
Sales budget for the year ending 31st December 1980
Budgeted Sales is arrived at as follows
munotes.in

Page 141

141
Sales Budget for the year ending 31 -12-1998
Illustration 7
A company has four departments at Bombay, Delhi, Calcutta
and Madras. It sells two products, Product –A and Product –B.
The budgeted sales for the six months ending June 30, 1997 at
each place are as follows :
Bombay Product –A 1,00,000 Units at Rs. 8/ -each
Product –B 70,000 Units at Rs. 5/ -each
Delhi Product –B 1,10,000 Units at Rs. 5/ -each
Calcutta Product –A 1,50,000 Units at Rs. 8/ -each
Madras Product –A 90,000 Units at Rs. 8/ -each
Product -B 80,000 Units at Rs. 5/ -each
The actual sales during the same period were :
Bombay Product –A 1,25,000 Units at Rs. 8/ -each
Product –B 75,000 Units at Rs. 5/ -each
Delhi Product –B 1,25,000 Units at Rs. 5/ -each
Calcutta Product –A 1,55,000 Units at Rs. 8/ -each
Madras Product -A 1,00,000 Units at Rs. 5/ -each
Product -B 87,000 Units at Rs. 5/ -each
From the reports of the sales personnel it was considered
that the sales budget for the six months ending June 30, 1980
would be higher than budget in the following aspects.munotes.in

Page 142

142Bombay Product –A - 8,000 units
Product –B - 5,000 units
Delhi Product –B - 13,000 units
Calcutta Product –A - 10,000 units
Madras Product -A - 8,000 units
Product -B - 6,000 units
The management intends to undertake intensive sales
compaign in Delhi and Calcutta and expect that this would result in
additional sales of 25,000 units of Product –Ai nD e l h ia n d1 8 , 0 0 0
units of Product –B in Calcutta.
You are required to prepare a sales budget for the period
ending 30th June 1998.
Illustration 8:
X. Company manufactures two products A & B. A forecast
for the number of units to be sold in the first seven months of the
year is given below :
munotes.in

Page 143

143
It is anticipated that 1) there will be no work in progress at
the end of any month, 2) fini shed units equal to half the sales for
the next month will be in stock at the end of each month (including
previous December)
Prepare for the six months ending 30th June a production
budget for each month and summarized production cost budget.
MONTHLY PRODUCTION BUDGET
munotes.in

Page 144

144Production Cost Budget
Illustration 9:
A semi -variable expense revealed that the expenses are Rs.
4,400 Find the fixed and variable elements. Also estimate the
amount of semi -variable expense for 1,500 labour -hours.
Solution:
Thus for a change of 200 labour -hours, change in amount of
expense is Rs. 400.
Therefore,
Thus if there be 1,500 labour -hours in period 3, variable
expense is Rs. 3,000 and total semi -variable expense is Rs. 3,000
+ Rs. 2,00 = Rs. 5,000.
Illustratio n1 0 :
Shah Manufacturers product 4,000 units of a certain product
at 100% capacity. The following information is obtained from the
books of accounts.munotes.in

Page 145

145
The rate of production per hour is 10 units. Direct material
Cost per unit is Rs. 1 and direct wages per hour Rs. 4.
Your are required to -
a)compute the cost of production at 100 per cent, 80 per cent, 60
per cent capacity showing the variable, fixed and semi -fixed
expense under the flexible budget and
b)find the overhead absorption rate per unit at 80% capacity.
Solution :
a)Cost of production under Flexible Budget.
munotes.in

Page 146

146
[At 80%, variable portion is 80 Ö 10 = Rs. 800 and fixed
portion is Rs. 100]
b)Total overhead at 80%, is Rs. 7,450, i.e. Rs. 2.33 per unit
should be the overhead absorption r ate.
Illustration 11:
A company has a Flexible Budgeting Control System which
recognizes between fixed and variable costs. The following data
has been collected for budgeting together with the Variability rate
for each items of expense.
Prepare a fle xible budget Allowance for the levels of Activity
which may be attained they are :
a)60%, 70%, 80%, 90% to 100% and 120%.
b)The Normal budget is taken as 80% at which level the units sold
is 12,000 Numbers.
c)The selling prices have to be reduced as f ollows : At 60% the
selling price is Rs. 30, but at 80% it is Rs. 29, at 90% it is Rs. 28
and at 100 and 120% it is Rs. 27.
d)This reduction is necessary to each the wider cirele of
customers.
e)The time taken to produce on unit of the Product is 0.6 Hou rs.
a)Variable Expenses : Rs. per unit
Material cost 5.00
Director Labour Cost 8.00
Processing Expenses 2.00 (Direct Expense)
10.00
Sales commission will 0.20 (per unit sold)
b)Semi Variable Expense : Selling and Manufacturing
a)Selling Expenses. . . . Rs. 12.00 Fixed + 0.20 per unit sold)
b)Manufacturing Expense for normal level of activity. 80% Hrs.
Fixed and Variable Components are –munotes.in

Page 147

147
Budget for Fixed Expenses
Schedule showing the selling prices at the various levels of
Sales Activity Sc hedule A
munotes.in

Page 148

148
Example
Indirect Labour @ 60% -5400 x 0.400 + 600 = 2160 + 600
= Rs. 2760
Supervision @ 60% -5400 x 1,200 + 1000 = 6480 + 100 = 7480.
Flexible Budgeting
munotes.in

Page 149

149
7.2 EXERCISE
Illustration 1
Prepare a Cash Budget from the follow ing information for
ABC Ltd.
The company desires to maintain a cash balance of
Rs.15,000 at the end of each quarter. Cash can be borrowed or
repaid in multiples of Rs.500 at an interest rate of 10% p. a.
Management does not want to borrow cash more tha nw h a ti s
necessary and wants to repay as early as possible. In any event,
loans cannot be extended beyond a quarter. Interest is computed
and paid when principal is repaid. Assume that borrowing takes
place at the beginning and repayments are made at the end of the
quarter.
Illustration 2
A company manufactures two products, X and Y. A forecast
units to be sold in first 4 months of the year is given below.
munotes.in

Page 150

150Other information is given below :
There will be no opening and closing work in progress [WIP ]
at the end of any month and finished product [in units] equal to half
the budgeted sale of the next month should be in stock at the end
of each month[ including previous year]
You are required to prepare,
a)Production Budget for January to April and
b)Summarized production cost budget
Illustration 3
The monthly budget for manufacturing overheads of a
manufacturing company is given below.
You are required to,
i)Indicate which of the item are fixed, variable and semi variable
ii)Prepare a budget f or 80% capacity
iii)Show that total cost, both fixed and variable per unit and output
at 60%, 80%, and 100% capacity levels.
munotes.in

Page 151

151MODULE -IV
Unit -8
OPERATING COSTING -I
Unit Structure :
8.0 Learning objectives
8.1 Introduction
8.2 Meaning of Operating Costing
8.3 Transport Costing
8.4 Solved Problems of Operating Costing 3.5 Hospital Costing
8.6 Solve d Problems on Hospital Costing 3.7 Hotel Costing
8.8 Solved Problems on Hotel Costing
8.9 Exercise
8.10 Questions
8.0 LEARNING OBJECTIVES
After studying this chapter one should able to understand:
The meaning of operating costing.
Process to select c ost limit in operating costing.
Procedure in operating costing according to the procedure of a
transporter
Accounting procedure of a Hotel
Accounting procedure of a Hospital
8.1 INTRODUCTION
Operating Costing method is normally used in service sector.
When the service is not completely standardized, it is the cost of
producing and monitoring a service. It is a method of costing
applied to undertakings which provide service rather than
production of commodities. Service may be performed internally
and exte rnally. Services are termed as internal when they have to
be performed on inter -departmental basis in factory itself e.g.
Power house services, canteen service etc.
Services are termed as external when they are to be
rendered to outside parties. Public ut ility services like transport,
water supply, electricity supply, hospitals are the best example for
the service costing. Thus operating costing is a method of cost
accumulation which is designed to determine the cost of services.munotes.in

Page 152

152Operating costing is just a variant of unit or output costing.
Operating costs are collected periodically like process cost. The
cost of rendering the service for particular period is related to
quantum of services rendered during the particular period to arrive
at cost per unit of service rendered. So the principal of unit costing
is used in operating costing.
8.2 MEANING OF OPERATING COSTING
Operating costing is a method of ascertaining the cost of
providing or operating a service. It is also known as service costing
CIMA London , defines Operating Costing as “that form of operation
costing which applies where standardized services are rendered
either by an undertaking or by a service cost renter with in an
undertaking”.
8.2.1 Cost Unit:
Determining the suitable cost unit to be u sed for cost
ascertainment is a major problem in service costing. Selection of a
proper cost unit is a difficult task. A proper unit of cost must be
related with reference to nature of world and the cost objectives.
The cost unit related must be simple i.e . per bed in a hospital, per
cup of tea sold in a canteen and per child in a school. In a certain
cases a composite unit is used i.e. Passenger –Kilometer in a
transport company. The following are some of example of cost units
used in different organizati ons
Enterprises Cost per unit
Passenger transport Kilometer
Goods transport Ton–Kilometer
Hotel Per room per day
Hospital Per bed per day
Canteen Per item, per meal
Water supply Per 1000 liters
Electricity Per kilowatt
8.2.2 Collection of costing data:
After determining the cost unit, the cost relating to the
service is collected. The collected cost is a presented under the
heads suitable for control purpose i.e. fixed expenditure and
variable expenditure. The presentation of cost data under difficult
categories helps to improve managerial control over cost.
8.3 TRANSPORT COSTING
8.3.1 Meaning
Transport costing is method of ascertaining the cost of
providing service by a transport undertaking. This includes air,munotes.in

Page 153

153water, road and railways; motor transpor t includes private cars,
carriers for owners, buses, taxies, carrier Lorries etc. The objective
of motor transport costing may be summarized as follows:
to ascertain the operation cost of running a vehicle
to provide and accurate basis for quotation and f ixing of rates
to provide cost companion between own transport and
alternative e.g. hiring
to compare the cost of monitoring one group of vehicle with
another group
to determine the cost to be changed against departments using
the service
to ensure the cos t of maintenance and repairs is not excessive
8.3.2 Classification of costs:
Costs are classified into the following three heads:
1)Standing or Fixed Charges: These charges are includes
whether vehicle is operating or not. Insurance, tax, depreciation
and part of driver wages. Interest on capital, general supervision,
and salary of operating managers is items come under the category
of fixed or standing charges.
2)Maintenance charges: There are semi variable expenses in
nature and include wear on tires ,r e p a i r sa n do v e r h e a d sp a i n t i n g
etc.
3. Operating and running charges: Running costs are the cost of
operations. These charges vary more or less in direct proportion to
kilometers etc. These expenses are variable in nature because they
are dependent on d istance covered and trips made.
Though the above three classification is done, in practical it
is difficult to distribute. It depends basically on the circumstances of
each case e.g. if the salary paid to driver is on monthly basis then it
is a fixed char ged but if the same is limited to kilometer run then it
is a running cost.
8.3.3 Collection of Cost Data:
Each vehicle is given a separate unique number and all the
basic documents will contain the assigned number of the respective
vehicles. A separate da ily log sheet for each vehicle is maintains to
record the details of trips, running time, capacity, distance cover,
cost of petrol / diesel, lubricants, loading and unloading time etc on
daily basis. A specimen of log sheet is given below:munotes.in

Page 154

154Daily log she et Table
Vehicle No.: ……………………. Route No.: ……………….
Date of Purchase: …………….. Driver:
Make and Specification: …………………….
Time of Leaving: …………….
License No.: …………………….. Time of Returning: …………….
Supplies Worker’s time abnormal delays
Petrol / die sel…………… Driver …………… Loading / unloading …..
Oil………………… conductor ………………. Accident ………………
Grease ………… Cleaner ………… Traffic Delays …….. Others ……..
Format of transport operating cost sheet:
Operating cost sheet
Vehicle No. : ……………….. Period ………………..
Cost Unit :……………………….. No. of Cost units …………………
Note: Maintenance expenses can be shown separately also
depends on cases.munotes.in

Page 155

155Check Your Progress:
1)Give the format of Transport Operating cost -sheet
2)Give the specimen of log sheet
3)Give the Cost Unit of the following
a)Passenger Transport
b)Good Transport
c)Electricity
d)Hospital
e) Hotel
4)Explain the following terms
a)Standing or Fixed Charges
b)Maintenance charges
c)Operating and running charges
d)Transport costing
e)Operating costing
8.4 SOLVED PROBLEMS OF TRANSPORT COSTING
Illustration 1:
From the following information calculate fare for passenger
KM.
The bus will make 3 rounds trips for carrying on the average
40 passenger’s in each trip. Assume 15 % profit on takings. The
busw i l lw o r ko nt h ea v e r a g e2 5d a y si nam o n t h .munotes.in

Page 156

156Solution
Operating Cost Statement
Bus No.
Capacity : 40 persons
Working Note:
1)No. of Km run in a month : 3 x 2 x 20 x 25 = 3000 km
2)No. of passenger km per annum : 3000 x 40 x 12= 14,40,000
3)Diesel and oil : 3000 x 125 / 100 = Rs. 3750
4)Commission & Profits: Commission 10 % of taking + profit 15 %
of Taking total = 25 % of taking so the cost
Cost is only 75 %
Illustration 2 :
From the following data relating to two different vehicles A
and B, compute cost per running mile.munotes.in

Page 157

157
Charge interest @ 5 % p.a. on cost of vehicles. The vehicles run 20
miles per hour on an average
[M. Com .M a d u r a iK a m r a j ]
Solution :
Operating cost sheet (cost per mile)
Note :
1)Depreciation is linked with mileage so operating cost.
2)Driver wage is taken as operating since it is paid per hour.munotes.in

Page 158

1588.5 HOSPITAL COSTING
Hospitals comes under service sector, big companies also
maintain hospitals. For costing purpose the hospital serv ice can be
divided in two following categories .
1)Outpatient department
2)Wards
3)Medical service departments such as radio therapy ‘X’ ray etc.
4)General Services such as heating, lighting, catering laundry etc.
5)Other services such as transport, d ispensary, cleaning etc.
8.5.1 Cost Statement:
The expenses of hospital can be broadly divided into two
categories i.e. (1) Capital Expenditure and (2) Maintenance
Expenditure –this includes salaries and wages, provision, staff
uniforms clothing, medical and surgical appliances and equipments,
fuel light and power, laundry, water etc.
8.5.2 Format of a cost Sheet of a Hospital:
munotes.in

Page 159

1598.6 SOLVED PROBLEMS ON HOSPITAL COSTING
Illustration 3:
The following information is available from a intensive care
unit. Rent (including repairs) Rs. 10000 p.m.
The unit cost consists of 25 beds and 5 more beds can be
accommodate when the occasion demands. The permanent staff
attached to the unit is as follows:
2 supervisors each at a salary or Rs. 2000 per month.
4n u r se each at a salary of Rs. 1500 per month.
2 ward boys each at a salary of Rs. 1000 per month.
Though the unit was open for the patients all the 365 days in
a year, security of accounts of 2008 revealed that only 150 days in
ay e a rt h eu n i th a dt h ef u l l capacity of 25 patients per day and for
another 80 days it had on an average 20 beds only occupied per
day. But there were occasions when the beds were full, extra beds
were hired from outside at a charge of Rs. 10 per bed per day and
this did not come to more than 5 beds extra above the normal
capacity any one day. The total hire charges for the whole year
were Rs. 4000.
The unit engaged expert doctor from outside to attend on the
patients and the fees were paid on the basis of number of patients
attended at time spent by them on an average worked out to Rs.
2000 per month in 2008. The other expenses for the year were as
under.
1)If the unit recovered an overall amount of Rs. 200 per day on an
average from each patient what is the profit per patient d ay
made by the unit in 2008.
2)The unit wants to work out a budget for 2009, since the number
of patients is very uncertain, annuity the same revenue and
expenses prevail in 2009, work out the number of patient days
required break -even.munotes.in

Page 160

160Solution :
Statem ent of cost and profit
8.7 HOTEL COSTING
Hotel industry is a service industry and covers various
activities such as provision for food and accommodation. It also
provides other comforts like recreations, business facilities,
shopping areas etc. Th e expenses incurred in a hotel are fixed or
variable. Fixed expenses comprises of staff salaries, repairs,
interior decoration, laundry contract cost, sundries and depreciation
on fixed assets. The variable expenses incurred are lighting,munotes.in

Page 161

161attendants’ salar ies, power etc. To find out room rent to be charged
from customers a notional profit is added with the cost and divided
by the number of rooms available. The number of rooms available
is calculated after for considering availability of suits and
occupancy.
Rooms rent may be different from season to season.
Sometime besides accommodation they also provide food. Then
the cost of meals, other direct and indirect costs are considered to
work out the costs to be charged from customers.
Operating cost sheet of aH o t e l :
CHECK YOUR PROGRESS
1)Give the formats of the following:
a)Operating Cost Sheet of a Hospital
b)Operating cost sheet of a Hotel
2)Enlist the categories of Hospital services.
3)Which expenditures are included in Maintenance Expenditure in
case if hospital costing?
4)Find out if the following expenses are Fixed expenses or
variable expenses in case of Hotel costin:.
a)Staff salaries
b)Repairs
c)Interior decoration
d)Laundry contract cost
e)Sundries
f)Depreciation on fixed asset s
g)Lighting
h)Attendants’ salaries
i)Powermunotes.in

Page 162

1628.8 SOLVED PROBLEMS ON HOTEL COSTING
Illustration 4:
A company runs a holiday home for this purpose it hired a
building at a rent of Rs. 10,000 per month along with 5% of total
takings. It has three types of suites for its customer’s viz. single
room, double room and triple rooms.
Following information is given:
Types of suite Number Occupancy
percentage
Single rooms 100 100 %
Double rooms 50 80 %
Triple rooms 30 60 %
The rent of double room’s suite is t ob ef i x e da t2 . 5t i m e so f
the single room and that of triple rooms at twice of the double room
suite.
The other expenses for the year 2009 are as follows:
Rs.
Staff salaries 14,25,000
Room attendants wages 4,50,000
Lighting heating and powers 2,15,000
Repairs and renovations 1,23,500
Laundry charges 80,500
Interior decoration 74,000
Sundries 1,53,000
Provide profit @ 20 % on total takings and assume 360 days
in a year. You are required to calculate the rent to be charged for
each type of suite
[C. A. PE II]
Solution:
Calculation of room occupancy
Calculation of equalant single room suits occupancy
36,000 x 1 + 14400 x 2.5 + 6480 x 5 = 104400munotes.in

Page 163

163Calculation of Total Cost :
Rent for a single room = 3521333 ÷ 104400 = Rs. 33.73
Rent for a double room = 33.73 x 2.5 = Rs. 84.325
Rent for a triple room = 84.325 x 2 = Rs. 168.65
8.9 EXERCISE
Objective Type
Choose the correct answer for the multiple choice questions
1)Classification and accumulation of costs by fixed and variable
costs is a distinctive feature of
a) Process costing b)Unit costing
c) Operating costing d)Batch costing
2)Composite unit is distinctive feature of
a) Single or output costing b)Process costing
c) Job costing d)Operating costing
3)Electricity generating company should employ
a) Unit costing b)Process costing
c) Operating costing d)Multiple costing
4)Cinema houses must adopt
a) Operating costing b)Job costing
c) Batch costing d)Contract costing
5)For a library the best method of costing suitable is
a) Output costing b) Operating costing
c) Process costing d) Multiple costing
6)For an educational institutes the right method of costing is
a) Output costing b) Job costing
c) Operating costing d) Process costingmunotes.in

Page 164

1647)Hospitals must make use of
a)Operating costing b)Batch costing
c)Process costing d)Multiple costing
8)For hotels the best method of costing is
a)Single or output b)Contract costing
c)Process costing d)Operating costing
9)Air India Co. must make use of
a)Job cost ing b)Operating costing
c)Batch costing d)Process costing
10)Indian Railways must adopt
a)Operating costing b)Unit costing
c)Batch costing d)Multiple costing
11)Public utility undertakings must invariably adopt
a)Operating costing b)Output costing
c)Contract costing d)Multiple costing
12)Karnataka Electricity Board must make use of
a)Single or output costing b)Job costing
c)Process costing d)Operating cost
13)The method of costing used in case of a gas company is t ermed
as
a)Job costing b)Process costing
c)Operating costing
14)Mines `A’ and `B’ are at a distance of 10 kms and 15 kms from
the factory. The cost per tone -km in case of mine A is Rs. 3
while it is R. 2.5 in case of mine B. The factory should p rocure
coal from
a) Mine A only b) Mine B only
c) Both from mines A and B in the ration of 3 : 2
15)In case of steam company, the cost per unit is calculated on the
basis of
a)Total quantity of lbs. produced
b)Total quantity of kwh. generated
c)Total quantity of tones produced.
Answers: 131(c), 2(d), 3(c), 4(a), 5(b), 6 (b), 7(a), 8(d), 9(b), 10(a),
11(d), 12(a) (c), 14 (a), 15(a)munotes.in

Page 165

1658.10 QUESTIONS
Simple Questions
1.Define operating cost
2.Define operating costing
3.Distinguish between operating co st and operating costing
4.What do you mean by a composite unit?
5.List out any eight undertakings which makes use of operating
costing.
6.Give the composite unit of the following undertakings :
a)Roadways carrying passenger
b)Railways carrying goods
c)Hospit al and
d)College.
7.Mention the basis of classifying the cost under operating
costing.
8.Mention the basis of classifying the costs under transport
costing.
9.What is a log sheet?
10.What do you mean by cost summary performance statement?
11.What do you mean by absol ute tone -kilometer?
12.What do you mean by a commercial tone -kilometer?
13.Distinguish between absolute tone -kilometer and commercial
tone kilometer.
14.What do you mean by “kilometer run”?
15.What do you mean by “cost per hour” under operating
costing?
State whether each of the following statement is `True’ or
`False’
1.Operating costing is used in case of service undertaking.
2.Log sheet is prepared in case of power house costing.
3.The unit of cost for production of steam may be per lb.
4.Per man show cost is calculated in case of Canteen costing. 5.
Fare in case of taxis is generally based on cost per passenger,
km
Answer : (1) True (2) False (3) True (4) False (5) Falsemunotes.in

Page 166

166Long answer type.
1.What is service costing? Mention the types of business in which
the system would be suitable. Describe briefly a system of
service costing which you would recommend for use by a
passenger taxi service.
2.What are the main objects of motor transport costing? A
company owns a fleet of vans and wishes to examine the cost
of (a) each van, (b) the fleet as a whole. Prepare a report on the
accounting arrangements that are needed and draft specimen of
the forms that you recommend for presentation to the directors.
Show separate rates for fixed and variable expenditure and
state how these should be used.
3.Draw up a proforma cost statement for a canteen serving 1,000
workers in a factory. The canteen is subsidized by the factory.
4.What is “Operating Costing”? State the industries where it is to
be used?
5.What is a “Log sheet”? Give its proforma.
6.Your cl ient running a canteen tends to introduce costing system
in his organization. How should he classify his costs for the
purpose of preparing an Operating Cost Statement?
munotes.in

Page 167

167Unit -9
OPERATING COSTING -II
Unit Structure :
9.0 Objectives
9.1 Problems and Solution
9.2 Exercise
9.0 OBJECTIVES
After studying the unit the students will be able to solve the
problems on transport costing, Hospital costing and Hotel costing.
9.1 PROBLEMS AND SOLUTION
Illustration 1:
SAITRAVELS o wns a bus and operates a tourist service on
daily basis. The bus starts from New City to Rest village and
returns back to New City the same day. Distance between New city
and Rest village is 250 kms. This trip operates for 10 days in a
month. The bus also plies for another 10 days between New city
and Shivapur and returns back to New city the same day, distance
between these two places is 200 kms. The bus makes local
sightseeing trips for 5 days in a month, earning a total distance of
60 kms per day.
The f ollowing data are given.
Cost of bus Rs. 3,50,000
Depreciation 25 5
Driver’s salary Rs. 1,200 p.m.
Conductor’s Salary Rs. 1,000 p.m.
Part time clerk’s salary Rs. 400 p.m.
Insurance Rs. 1,800
Diesel consumption 4 kms per Rs. 8p e rl i t r e
litre @
Token tax Rs.2,400 p.m.
Permit fee Rs. 1,000 p.m.
Lubricant oil Rs. 100 for every 200 kms
Repairs and maintenance Rs. 1,500 p.m.
Normal capacity Rs. 50 personsmunotes.in

Page 168

168While playing to and for Rest village, the bus occupies 90%
of the capacity and 80% when it plies between New city to Shivapur
(both ways). In the city the bus runs full capacity passenger tax is
20 % of net takings of the “Travels” firm.
Calculate the rate to be charged to Rest village and Shivaupr
from New city, per passenger, if the profit required to be earned is
33 % of net taking of firm.
[I.C.W.A., Intermediate]
Solution:
Operating cost statement for the month
Charges per passenger:
a)to Rest village from New city : 250 x 0.161 i.e. Rs. 40.25
b)to Shivapur from New city : 200 x 0.161 i.e. Rs. 3 2.20
*t o t a lk m sc o v e r e dp . m .
Rest village and back 2 x 250 x 10 days 5,000
Shivapur and back 2 x 200 x 10 days 4,000
Local trips @ 60 kms for 5 days 300
9,300
**Total effective passenger –km per month :
Rest village 2 x 250 x 90 % of 50 x 10 days = 2,25,000
passenger km
Shivapur 2 x 200 x 80 % of 50 x 10 days =1,60,000
Local Trips 5 x 60 x 50 =15,000
4,00,000munotes.in

Page 169

169Illustration: 2:
(Service costing –use own / company cars or hire cars)
A company is considering three alternative proposals fo r
conveyance facilities for its sales personal who have to do
considerable travelling, approximately 20,000 kilometers every
year. The proposals are as follows :
1)Purchase and maintain its own fleet of cars. The average cost of
car is Rs. 1,00,000.
2)Allow the executive use his own car and reimburse expenses at
the rate of Rs. 1.60 paise per kilometer and also bear insurance
costs.
3)Hire cars from an agency at Rs. 20,000 per year per car. The
company will have to bear costs of petrol, taxes and tyres.
The following further details are available :
Petrol Re. 0.60 per km.
Repairs and maintenance Re. 0.20 per km
Tyre Re. 0.12 per km
Insurance Rs. 1,200 per car annum;
Taxes Rs. 800 per car per annum
Life of a car : 5 years with Annual milage of 20,000 kms . Resale
value: Rs. 20,000 at the end of the fifth year.
Work out the relative costs of three proposals and rank them
[C.A., Inter]
Solution :
Alternative proposals
munotes.in

Page 170

170
Decision II alternating i.e., use of own car will be the best
alternative from compan y’s point of view. III alternative i.e. hiring
the card is 2nd best alternative. I alternative i.e. maintaining the
fleet will be costliest alternative.
Rs. 1,200 ~ 20,000 kms = Re. 0.06; (Rs. 800 ~ 20,000 kms) = Re.
0.04 @ Rs. 20,000 ~ 20,000 kms = Re. 1 /-
Illustration 3:
The Union Transport Company has been given a twenty
kilometer long route to ply a bus. The bus costs the company Rs.
1,00,000. It has been insured at 3 % per annum. The annual road
tax amounts to Rs. 2,000. Garage rent is Rs. 400 per mo nth.
Annual repair is estimated to cost Rs. 2,360 and the bus is likely to
last for five years.
The salary of the driver and conductor is Rs. 600 and Rs.
200 per month respectively in addition to 10% of the taking as
commission to be shared equally by the m. The managers salary is
Rs. 1,400 per month and stationery will cost Rs. 100 per month.
Petrol and oil will cost Rs. 50 per 100 kilometers. The bus will make
three round trips per day carrying on an average 40 passengers in
each trip. Assuming 15% profit on takings and that the bus will ply
on an average 25 days in a month.
Prepare operating cost statement on a full year basis and
also calculate the bus fare to be charged from each passenger per
kilometer
[C.A., Inter]munotes.in

Page 171

171Solution:
Union Transport Company
Statement showing operating cost of the bus per annum
Calculation of bus fare to be charged:
Effective passenger kilometers:
(2 x 20 km x 3 trips x 40 passengers x 25 days x 12 months) = 14,
40,000
Rate to be charged per km from each passenger
Rs. 1, 03,680 ~ 14,40,000 = Re. 0.072
Calculation of total distance covered
(20 km 2 x 3 x 25 x 12) = 36,000 km per annum
Illustration 4: (Transport Costing)
Prakash Automobiles distributes its goods to a regional
dealer using a single lorry. The dealers’ pre mises are 40 kilometers
away by road. The lorry has a capacity of 10 tons and makes the
journey twice a day fully loaded on the outward journeys and empty
on return journey. The following information is available for a four
weekly period during the year 19 90.munotes.in

Page 172

172Petrol consumption 8 km per liter
Petrol Cost Rs. 13 per liter
Oil Rs. 100 per week
Driver’s wages Rs. 400 per week
Repairs Rs. 100 per week
Garage Rent Rs. 150 per week
Cost of Lorry (excluding tyres) Rs. 4,50,000
Life of Lorry 80,000 kilometers
Insurance Rs. 6,500 per annum
Cost of tyres Rs. 6,250
Life of tyres 25,000 kilometers
Estimated sale value of lorry at end of its Rs. 50,000
life
Vehicle license cost Rs. 1,300 per annum
The lorry operates on five day week Rs. 41,600 per annum
Required:
a)Astatement to show the total cost of operating the vehicle for
four-weekly period analyzed into running costs and fixed costs.
b)Calculate the vehicle cost per kilometer and per ton kilometer
[C.A., Inter]
Solution:
a)Before computing the total cost, it is necessary to find out the
basic data s under :
1)Distance travelled in 4 week period; 40 km one way x 2
(return) x 2 trips x 5 days x 4 weeks = 3200 km
2)For tone km working = empty on return and as such for tone
km = 3200 ÷2=1 , 6 0 0
3)Total cons umption in weeks = 3,200 km ÷8k m=4 0 0l t
4)Tyre cost = (Rs. 6,250 ÷25,000 km) x 3,200 km = Rs. 800
5)Depreciation of lorry in 4 weeks
= (Rs. 4, 50,000 –Rs. 50,000 km) ÷80,000 x 3,200 =
Rs. 16,000
Operating cost statement f a lorry of M/s. Prakash Automobiles
(for the 4 week period)munotes.in

Page 173

173
(b) Cost per tone –km = Rs. 28,800 ~ (1600 x 10 tons) Rs. 1.80
Illustration 5 :
A company presently brings coal to its factory from a nearby
yard and the rate paid for transportation of coal from the yard
located 6 kms. Away to factory is Rs. 50 per ton. The total coal to
be handled in a month is 24,000 tones.
The company is considering proposal to buy its own trucks
and has the option of buying either a 10 ton capacity or a 8 ton
capacity trucks.
The follo wing information is available:
Each truck will daily make 5 trips (to and fro) on an average
for 24 days in a month. Cost of diesel Rs. 15/ -per liter. Salary ofmunotes.in

Page 174

174driver Rs. 3,000/ -,p . a .m o n t h .T w od r i v e r sw i l lb er e q u i r e dp e r
truck. Other staff expens es Rs. 1,08,000 p.a.
Present a comparative cost sheet on the basis of above data
showing transport cost per ton of operating 10 ton and 8 ton Truck
at full capacity utilization.
[C.A. Final]
Solution :
Comparative statement of operating cost sheet :
Conclusion : A comparison of cost per ton by using 10 ton trucks
is more economical. The cost paid for bringing coal per ton
presently viz. Rs. 50/ -is the highest.
Working Note :
munotes.in

Page 175

175
Illustration.6:
You are required to calculate a suggested fare per
passenger –km from the following information for a mini bus.
i)Length of route 30 km
ii)Purchase price Rs. 4,00,000.
iii)Part of above cost meet by loan, annual interest Rs. 10,000 p.a.
iv)Other annual charges : Insurance Rs. 15,000, Garage Rent Rs.
9,000, Road Taxes Rs. 3,000, Repairs and Maintenance Rs.
5,000. Administrative charges Rs. 5000.
v)Running expenses : Driver & Conductor Rs. 5000 p.m., Repairs
/ Replacement of tyre tube Rs. 3600 p.a. Diesel and Oil cost per
Km Rs. 5/ -
vi)Effective life of vehicle is estimated at 5 years at the end of
which it will have a scrap value of Rs. 10,000.
vii)Mini Bus has 20 seats and is planned to make six two way trips
for 25 days / p.m.
viii)Provide profit @ 20 % of total revenue.
[C.A., Final]
Solution :
munotes.in

Page 176

176
Rate per passenger km :
Rs. 36937.50 / 1,80,000 passenger km = 0.4274305 or 0.43 paise
Workings:
Total distance travelled by mini bus in 25 days = 60 km x 6 trips x
25 days = 9000 km
Total passenger km = 9000 km x 20 seats = 1,80,000 passengers
km
Illustration 7 :
Krishna Transport Ltd. Charges Rs. 150 per ton for its 10 ton
lorry load from city A to city B. the charges for the return journey
are Rs. 140 per ton. No concession is made for any delivery of
goods at intermediate station ’C’ in January 2008 . The truck made
10 outward journeys for city B with full load of which 2 ton were
unloaded twice at city ‘C’. The truck carried a load of 12 ton in its
return journey for 4 times but once caught by police and Rs. 1500
was paid as fine. For the remaining t rips it carried full load out of
which all the goods on load were unloaded once at city ‘C’. The
distance from city A to city A and city ‘B’ are 150 km and 250 km
respectively. Annual fixed cost are Rs. 1,20,000 and maintenance
cost is Rs. 15,000. Running charges spent during January 2008 are
Rs. 3500.
Calculate the cost per tone -kilometer and the profit for
January 2008.munotes.in

Page 177

177Solution :
Operating Cost and Profit Statement of Krishna Transport Ltd.
Illustration 8 :
Mr. Sampath owns a fleet of taxies a nd the following
information is available from the records maintained by him.
1)Number of Taxis –10
2)Cost of each Taxi –Rs. 2,00,000
3)Salary of manager Rs. 6000 p.m.
4)Salary of Accountant Rs. 5000 p.mmunotes.in

Page 178

1785)Salary of cleaner Rs. 3000 p.m.
6)Salary of Mechanic Rs. 4000 p.m.
7)Garage Rent Rs 7000 p.m.
8)Insurance premium 5 %
9)Annual Tax Rs. 6000 per taxi
10)Drivers Salary Rs. 4000 p.m.
11)Annual Repairs Rs. 15,000 per taxi
Total life of a taxi is about 2,00,000 kms. A taxi runs in all
3000 kms. in a month of which 25 % its runs empty. Petrol
consumption is one liter for 10 kms @ Rs. 40 per liter. Oil and other
sundries are Rs. 10 per 100 kms.
Calculate the cost of running a taxi per km.
Solution:
Operating cost sheet
Illustration 9:
A lodging home is being run in a small hill station with 50
single rooms. The home offers concessional rate during six off
season months in a year. During this period, half of the full room
rent is charged. The management profit margin is targe ted at 20%
of the room rent. The following are the cost estimates and othermunotes.in

Page 179

179details for the year ending 31st March, 1996 (assume a month to
be of 30 days)
a)Occupancy during the season is 80%, while in the off season is
40% only.
c)Annual depreciation is to be provided for building at 5% and on
furniture and equipments at 15% on straight line basis.
d)Room attendants are paid Rs. 5/ -per room -day on the basis of
occupancy of the rooms in a month.
e)Monthly lighting charges are Rs. 120 per room, expec ti nf o u r
months of winter when it is Rs. 30 per room and this cost is on
the basis of full occupancy for a month and
f)Total investments in the home are Rs. 100 lakhs of which Rs.
80 lakhs relate to buildings and balance for furniture and
equipments.
You are required to work out the room rent chargeable per
day both during the season and the off -season months, on the
basis of the foregoing information.
[I.C.W.A., Intermediate
Solution:
Total estimated costs for the year ending 31.03.1996
During sea son room rent is Rs. 197 and during off -season room
rent is Rs. 98.50munotes.in

Page 180

180*Attendant’ salary
For 10,800 room days @ Rs. 5 per day = Rs. 54,000
**Total light bill
Light bill during 8 months at Rs. 120 per month or 120 ÷30 =
Rs. 4 Per room day.
Light bill during 4 months of winter at Rs. 30 per month or 30 ÷
30 = Re. 1 per Room day.
Total light bill for full one year Rs.
During season @ Rs. 4 for 7,200 days 28,800
During 2 months of off -season
@ Rs. 4 for 1,200 days (2 ÷6x3 , 6 0 0 ) 4,800
During 4 month so fw i n t e ra tR e .1
For 2,400 days (4 ÷6x3 , 6 0 0 ) 2,400
Total 36,000
*** Number of room days in a year :
Seasons occupancy for 6 months@80% (50 x 0.8 x 6 x 30) =
7,200 room days Off season’s occupancy for 6 months @ 40 % (50
x0 . 4x6x3 0 )=3 , 6 0 0 r oom days
Total room days during the Year 10,800
Total full room days in terms of rate
Season 7,200
Off Season (in terms of 50 % rate on 3,600 days) 1,800
Total Full room days 9,000 per annum
Illustration 10:
Elegant Hotel has a capacity of 100 single rooms and 20
double rooms. It has a sports centre with a swimming pool which is
also used by persons other than residents of the hotel. The hotel
has a shopping arcade at the basement and a specialty restaurant
at the roof top. The following information is available:
1)Average occupancy : 75 % for 365 days of the year
2)Current costs are :
Variable cost Fixed cost
Single room 400 200
Double room 500 250
3)Average sales per day of restaurant Rs. 1, 00,000; contribution
is at 30 %. Fixed cost Rs. 10, 00,000 per annum.
4)The sports centre / swimming pool is likely to be used by 50 non
–residents daily; average contribution per day per non resident is
estimated at Rs. 50; fixed cost is Rs. 5,00,000 per annum.
5)Average contribution per month from the s hopping arcade is Rs.
50,000; fixed cost is Rs. 6, 00,000 per annum.munotes.in

Page 181

181You are required to find out:
a)Rent chargeable for single and double room per day, so that
there is a margin of safety of 20 % on hire of rooms and that the
rent for a double room shou ld be kept at 120 % of a single
room.
b)Evaluate the profitability of restaurant, sports centre and
shopping arcade separately.
[C. A. Final]
Solution:
(a) Statement for calculating the rent chargeable for single and
double room per day.
Rent per day of single room 9in Rs.) 756 (approx)
(Refer to working note 2)
(Rs. 2, 56,64,062 / 33,945)
Rent per day of double room (in Rs.) 907 (approx)
(Rs. 756 x 1.2 times)
munotes.in

Page 182

182Working Note :
1)Single room occupancy days in a year = 100 room x 365 days x
75 % = 27,375
Double room occupancy days in a year = 20 rooms x 365 days x
75 %
=5 , 4 7 5
2)In terms of single room total room occupancy days in a year
=2 7 , 3 7 5+1 . 2 0%x5 , 4 7 5=2 7 , 3 7 5+6 , 5 7 0
=3 3 , 9 4 5
Illustration 11:
Following are the in formation given by an owner of a hotel.
You are requested to advice him that what rent should be charge
from his customers per day so that he is able to earn 25 % on cost
other than interest.
1)Staff salaries Rs. 80,000 per annum
2)Room attendant’s sala ry Rs. 2 per day. The salary is paid on
daily basis and services of room attendant are needed only
when the room is occupied. There is one room attendant for one
room.
3)Lighting, heating and power. The normal lighting expenses for a
room if it is occupie d for the whole month is Rs. 50. Power is
used only in winter and normal charge per month if occupied for
a room is Rs. 20.
4)Repairs to building Rs. 10,000 per annum
5)Linen etc. Rs. 4,800 per annum
6)Sundries Rs. 6,600 per annum
7)Interior decoration and furnishing Rs. 10,000 annually
8)Cost of building Rs. 4,00,000; rate of depreciation 5 %
9)Other equipments Rs. 1,00,000; rate of depreciation 10 %
10)Interest @ 5% may be charged on its investment of Rs.
5,00,000 in the building and equipment
11)There are 100 rooms in the hotel and 80 % of the rooms are
normally occupied in summer and 30 % of the rooms are busy
in winter. You may assume that period of summer and winter is
six month each. Normal days in a month may be assumed to be
30.munotes.in

Page 183

183Solutio n:
Operating cost sheet
Rent per day
9.2 EXERCISE
Practical problems
Illustration 1 :
AM i n e r a li st r a n s p o r t e df r o mt w om i n e s –“A” and “B” and
unloaded at plots in a Railway Station. Mine A is at a distance of
10kms. And B is at a distance of 1 5kms. from the mines. Records
reveal that the lorries average a speed of 30 kms. per hour, when
running and regularly take 10 minutes to unload at the railhead. Atmunotes.in

Page 184

184mine “A” loading time averages 30 minutes per load while at mine
“B” loading time averages 2 0m i n u t e sp e rl o a d .
Drivers’ wages, depreciation, insurance and taxes are found to cost
9 per hour operated. Fuel, oil, tyres, repairs and maintenance cost
1.20 per km.
Draw up a statement, showing the cost per tonne -kilometer of
carrying mineral from ea ch mine.
(M.Com . Oct. 01)
(Ans.: Cost per tonne Km. Mine A: Rs. 0.72, Mine B: Rs.0.66)
Illustration 2 :
A transport company maintains a fleet of bus as follows :
Number of Buses Carrying Capacity
20 50 passengers each
10 40passengers each
Each bus makes 5 trips a day, covering a distance of 10 Km.
in each trip. On an average 80% of the seats are occupied in each
trip and 5 buses are under repair every day. Assuming that the
company operates its fleet daily, ascertain the o perating cost per
passenger -Km. from the following :
Wages of 30 Drivers `3,000 each per month
Wages of 30 Cleaners `1,000 each per month
Petrol `20,000 per month
Oil, Grease etc. `5,000 per month
Tyres, Tubes etc. `2,000 per month
Repairs `30,000 p er year
Garage Rent `40,000 per year
Road Licences `20,000 per year
Taxes `5,000 per half year
Permit Fee `25,000 per year
Salary of Operating Manager `5,000 per month
Office Overheads `10,000 per year
(M.Com , Oct 2000)
Ans: (Total Operating Cost: Rs. 19,59,000, Cost per passenger
Km.: Rs.0.1 15)
Illustration 3 :
A company presently brings coal to its factory from a nearby
yard which is located 6 kms. away to factory and the rate paid ` 50
per ton for transportation. The total coal to be handled in month in
24,000 tons.munotes.in

Page 185

185The company is considering proposal to buy its own trucks
and has the option of buying either a 10 ton or a 8 ton capacity
trucks.
The flowing information is available :
Each Truck will daily make 5 tr ips (to and fro) on an average for 24
days in a month.
Cost of Diesel `16 per litre.
Salary of Drivers `3,000 per month and two drivers will be required
for a Truck.
Other staff expenses `1,08,000 p.a.
Present a Comparative Cost Sheet on the basis of a bove
data showing transport cost per ton of operating 10 ton and 8 ton
Truck at full capacity utilization.
(M.Com .M a r .0 2 ,a d a p t e d )
Ans: (Total Operating Cost: 10 Ton Truck: Rs. 49,635 8 Ton Truck
Rs.41,381, Cost per Ton: 10 Ton Truck: Rs. 41.36, 8 Ton Truck:
Rs.43.1 1)
Illustration 4 :
The following were the expenses incurred by CALL and
MALL Company in operating two lorries (for the conveyance of Raw
Materials) and a bus (for the conveyance of Staff) during the month
of Fe bruary, 2006 :munotes.in

Page 186

186
The above vehicles carried the following Raw materials and
Passengers during the month :
Lorry C 100 Tonnes of Raw Material
Lorry M 120 Tonnes daily for 25 days
Respective mileage of the vehicles during the month :
Lorry C 3,000
LorryM 4,500
Bus 2,000
From the above statistics prepare an Operating Cost Sheet
in summary for the three vehicles. Also explain the unit of costing
selected.
(M.Com ., April 06, adapted)
Ans: (Total Operating Cost: Lorry C: Rs. 1,178, Lorry M: Rs. 1,135,
Bus: Rs. 955,Total Tonnes or Passenger Miles: Lorry C: 12,000,
Lorry M: 21,600, Bus: 50,000)
Illustration 5:
An entrepreneur owns a bus which runs from Mumbai to
Pune and back for 25 days in a month. The distance from Mumbai
to Pune is 170 kms. The bus completes the trip from Mumbai to
Pune and back on the same day. Calculate the fare to be charged
tothe following further information is available :munotes.in

Page 187

187
The bus usually runs full upto 90% of its capacity both ways. Permit
fee is payable on the cost of bus at 10% p.a.
(M.Com .A p r i l0 5 )
Ans: (Total Operating Cost: Rs. 3,08,295, Total passenger Km.:
45,90,000)
Illustration 6 :
KKK Automobiles distributes its goods to a regional trader
using a single lor ry. The trader’s premises are 40 kms away by
road. The lorry has a capacity of 10 tonnes and makes the journey
twice a day fully loaded on the outward journeys and empty on
return journeys.
Your are given data for 4 weekly periods during the year
2003.
Petrol consumption 8 kms per litre
Petrol cost Rs. 13 per litre
Oil Rs. 100 per week
Driver’s wages Rs. 400 per week
Repairs Rs. 100 per week
Garage rent Rs. 150 per week
Cost of lorry Rs. 4,50,000 (excluding tyres)
Life of lorry 80,000 kms.
Insurance Rs. 6 ,500 p.a.
Cost of tyres Rs. 6,250
Life of tyres 2,500 kms
Estimated Scrap value of lorry at the end of its life `50,000
Vehicle licence cost Rs. 1,300 p.a.
Other overhead cost Rs. 41,600 p.a.
The lorry operates on a Five -day weekmunotes.in

Page 188

188Required :
a)A stateme nt to show the total cost of operating the vehicle for
the 4 weekly periods analysed into running costs and fixed
costs.
b)Calculate vehicle cost per km. and per tonne km.
(M. Com ,O c t .0 4 ,a d a p t e d )
Ans: (Total Operating Cost : Rs. 28,800, Effective Km -Tonne
.:16,000 Effective Km. 1600)
Illustration 7 :
A person owns a bus which runs between Delhi and
Chandigarh and back for 10 days in a month. The distance
between Delhi and Chandigarh is 150 kms. The bus completes the
trip from Delhi and Chandigarh and back on the same day.
The bus goes to Agra for another 10 days. The distance
between Delhi and Agra is 120 kms. The trip is also completed on
the same day. For the rest 4 days of its operation, it runs in Delhi.
The daily dist ance covered is 40 kms.
Calculate the charges to be made if a profit of 33(1/3)% is to
be earned on his takings.
The other available information given to you is :
Cost of the bus Rs. 60,000.
Depreciation 20% p.a.
Salary of Driver Rs. 350 p.m.
Salary of C onductor Rs. 350 p.m.
Salary of Cleaner Rs. 160 p.m.
Insurance Rs. 1,680 p.a.
Diesel consumption is 4 kms per litre. Diesel costs Rs. one per litre.
The token tax is Rs. 600 p.a.
Lubricants Rs. 10 per 100 kms; repairs and maintenance Rs. 300
p.m.; permit fee Rs. 284 p.m. and the normal capacity is 50
persons.
The bus generally has 90% of its capacity occupied when it goes to
Chandigarh, 80% when it goes to Agra. It is always full when it runs
within the city. Passenger tax is 20% of his net takings.
(M. Com ,O c t .0 4 ,a d a p t e d )
Ans: (Total Operating Cost per month: Rs. 4,580, Cost per
passenger Km. Rs.0.034)
Illustration 8 :
A person owns a bus that runs between Mumbai and
Lonavala and back, for 10 days in a month. The distance frommunotes.in

Page 189

189Mumbai to Lonavala is 150 kms. The bus completes the trip from
Mumbai to Lonavala and return in the same day. The bus goes
another 10 days in a month towards Alibagh. The distance from
Mumbai to Alibagh is 120 kms. The trip is also completed on the
same day. For the rest 4 days of its operation in a month it runs
locally in Mumbai, covering daily distance of 40 kms. Calculate the
rate that the person should charge from passenger when he wants
to earn the profit of 25% on his takings and also calculate the
charge per passenger for both the out -station trips. The other
information is given as follows :
Cost of the bus (Depreciation @ 20% p.a.; Normal Capacity :
50 persons) 6,00,000
Salary : Driver 5,000 per month
Salary : Conductor 5,000 per month
Fixed Office Overheads 2,000 per month
Insurance 7,200 per month
Fuel (Consumed @ 4 kms/litre) 35 per litre
R.T.O. tax 600 per annum
Lubricant Oil 10 per 100 kms
Repairs and Maintenance 500 per month
Permit Fee 300 per month
Passenger tax is 20% of the net tak ings. The bus is occupied 90%
of its capacity while on Lonavala trip and 80% of its capacity while
on Alibagh trip, but is fully occupied in its local journey.
(M. Com ,A p r i l0 8 ,a d a p t e d )
Ans: (Total Operating Cost: Rs. 72,656, Total Passenger Km.
2,39.000, Cost per passenger Km.: Rs.48.64)
Illustration 9
A transport company supplies the following details in respect
of a truck of 5 tonne capacity which carries goods to an from the
city covering a distance of 50 kms each way.
munotes.in

Page 190

190While going to the city, freight is available for a full load of
the truck and on its return journey it can fetch freight only upto 20
percent of its capacity.
On the assumption that the trucks runs on an average 25
days a month, you are required to det ermine the following :
i)Operating cost per tone -km,
ii)Rate per tone per trip that the company should charge if profit if
50 percent on cost is to be earned, and
iii)What freight should the company charge if one wants to engage
the truck for one day f or a trip to the city and back?
(M.Com ,A p r i l0 9 ,a d a p t e d )
Ans: (Total Operating Cost: Rs. 8,250, Cost per Tonne Km.:
Rs.1.100)
Illustration 10 :
From the following information relating to a Hotel, calculate
the room rent to be charged to give a profit of 25% on cost
excluding interest charged on Loan for the year ended 31st March,
2008 :
1)Salaries of office staff Rs. 50,000 per month.
2)Wages of the room attendant: Rs. 20 per day per room when
the room is occupied.
3)lighting, Heating and Power :
a)The normal lighting expenses for a room for the full month is
Rs. 500, when occupied.
b)Power is used only in winter and the charges are 200 for a
room, when occupied.
4)Repairs to Beds and other furniture: Rs. 30,000 per annum.
5)Repairs to Hotel building: Rs. 50,000 per annum.
6)Licence fees: Rs. 12,400 per annum.
7)Sundries: ` 10,000 per month.
8)Interior decoration and furnishing: Rs. 1,00,000 per annum.
9)Depreciation @ 5% p.a. is to be charged on Building costing
20,00,000/ -and @ 10% p.a. on Equipments.
10)There are 200 rooms in the Hotel, 80% of the rooms are
generally occupied in summer, 60% in winter and 30% in rainy
season.munotes.in

Page 191

191The period of summer, winter and rainy season may be
considered to be of 4 months in each case. A month may be
assumed of 30 days of an average
(M. Com .O c t .0 8 ,a d a p t e d )
Ans: (Total Earnings: Rs. 33,18,000, Total Room Days:
40,800.Cost per Day: Rs.81 .32)
Illustration 11:
Relax Hotel has a capacity of 100 si ngle rooms and 20
double rooms. The average occupancy of both single and double
rooms is expected to be 80% throughout the year of 365 days. The
rent for the double room has been fixed at 125% of the rent of the
single room. The costs are as under:
Variab le Costs :Single rooms Rs. 220 each per day
Double rooms Rs. 350 each per day
Fixed Costs :Single rooms Rs. 120 each per day
Double rooms Rs. 250 each per day
Calculate the rent chargeable for single and double rooms
per day in such a way that the hotel earns on overall profit of 20%
on hire charges of rooms.
(M. Com .A p r i l0 9 ,a d a p t e d )
Ans: (Total Earnings: Rs. 1,67,90,000, Total Room Days: Single
room: 29,200 Double room : 5,840.)
Illustration 12:
A hospital is run by a Company. For this purpose it has hired
a building at a rent of Rs. 5,000 per month plus it would bear the
repair charges also.
The hospital is having 25 beds and 5 more beds can be
accommodated when the need arises.
The staff o f the hospital is as follows :
2 Supervisors each at a salary of Rs. 500 per month
4 Nurses each at a salary of Rs. 300 per month
2 Ward boys, each at a salary of Rs. 150 per month
Although the hospital is open for patients all the 365 days in
ay e a r ,r e c ords for the year 2004 disclose that only for 120 days in
the year, the unit had the full capacity of 25 patients per day and
when the beds were full, extra beds were hired at a charge of ` 5
per bed per day and this did not come to more than 5 beds extra
above the normal capacity on any one day. The total hire charges
for the extra beds incurred for the whole year were Rs. 2,000.munotes.in

Page 192

192The Unit engaged expert doctors from outside to attend on
the patients and the fees was paid on the basis of the number of
patients attended and time spent by them which on an average
worked out to Rs. 10,000 per month in 2004.
The other expenses for the year were as under:
Repair and Maintenance Rs. 3,600
Food supplied to patients Rs. 44,000
Sanitary and Other services for patie nts Rs. 12,500
Laundry Charges Rs. 28,000
Medicines supplied Rs. 35,000
Cost of oxygen, X -ray, etc. other than directly borne for
treatment of patients Rs. 54,000.
General Administration Charges allocated to hospital Rs.
49,550.
If the hospital recovere d an amount of Rs. 100 per day on an
average from each patient, compute the profit per patient –day
made by the hospital as per operating cost sheet for the year 2004.
(M. Com .O c t . 0 6 ,a d a p t e d )
Ans: (Total Earnings: Rs.61 ,350 , Total Number of Patient days:
5000.)
munotes.in